You are on page 1of 147

1.

CURRENT ELECTRICITY
We know that charge flows from higher potential to lower potential. Here charge means positive
charge. Negative charge flows from lower potential to higher potential.
Charge in motion constitutes electric current.
Flow of electric charge is a direct measure of electric current. Suppose a collection of charges is
moving perpendicular to a surface of area A, as shown in figure. The electric current is defined as the
rate at which charges flow across any cross-sectional area. If an amount of charge  Q passes
through a surface in a time interval t , then the average current I avg is given by
I

Q
I avg  …… (1)
t

Q dQ
In the limit t  0 , the instantaneous current I may be defined as I  Lt 
t 0 t dt
Current is a scalar quantity. It is a macroscopic quantity like the mass of a body or volume of a
container. In SI Units, current is one of the fundamental physical quantities. It is dimensionally
denoted by [I] or [A].
The SI unit of current is ampere (A).
Ampere: If one coulomb of charge passes through a cross- section of the conductor per second then the
current is one ampere.
coulomb(C)
Ampere(A)= =1.67x10-19 electrons/sec.
second(s)
The conventional direction of current is the direction of flow of positive charge or applied field. It is
opposite to direction of flow of negative charge or electrons.

+- +-

CONDUCTOR:
In some materials, the outer electrons of each atoms or molecules are only weakly bound to it.
These electrons are almost free to move throughout the body of the material and are called free
electrons. They are also known as conduction electrons. When such a
material is placed in an electric field, the free electrons move in a direction opposite to the field.
Such materials are called conductors.
INSULATOR
Another class of materials is called insulators in which all the electrons are tightly bound to their
respective atoms or molecules. Effectively, there are no free electrons. When such a material is
placed in an electric field, the electrons may slightly shift opposite to the field but they can’t

Dr.K.K.R Gowtham 1 Current Electricity


leave their parent atoms or molecules and hence can’t move through long distances. Such
materials are also called dielectrics.
SEMICONDUCTOR
In semiconductors, the behavior is like an insulator at low levels of temperature. But at higher
temperatures, a small number of electrons are able to free themselves and they respond to the
applied electric field. As the number of free electrons in a semiconductor is much smaller than that
in a conductor, its behavior is in between a conductor and an insulator and hence, the name
semiconductor. A freed electron in a semiconductor leaves a vacancy in its normal bound position.
These vacancies also help in conduction.
ELECTRIC CURRENT IN DIFFERENT CASES:
a) Current Constitutions in solid conductors. In solid conductors like metals, the valence electrons
of the atoms do not remain attached to individual atoms but are free to move throughout the volume
of the conductor. Under the effect of an external electric field, these free electrons move in a definite
direction causing electric current in the conductors. Thus free electrons are the current
constituents in solid conductors.
b) Current in electrolytes: Liquid conductors are called electrolytes. In an electrolyte like NaCl
solution etc., there are positively and negatively charged ions (Na +, Cl–). These are forced to move
in definite directions under the effect of an external electric field, causing electric current. Thus in
electrolytes, the current is due to positively and negatively charged ions.
c) Current in gases: Generally, the gases are insulators of electricity. But they can be ionized by
applying a high potential difference at low pressures or by their exposure to X rays etc. The ionized
gas contains positive ions and electrons. Thus positive ions and electrons constitute current in
gases.
d) Current in semi conductors: In semi conductors the flow of current is due to holes and free
electrons.
Remarks:
a) Current is a scalar quantity even though it has a direction I1
because it does not follow Laws of vector addition i.e, if two I

currents I1 and I2 reach a point we always have I = I1 + I2 ,
whatever be the angle between I1 and I2 as shown I2
I  I1  I 2
b) As charge is conserved and current is rate of flow of charge, the charge entering at one end per
sec of a conductor is equal to the charge leaving the other end per sec,This in turn implies that
i) For a given conductor current does not change with change in its area of cross section.
ii) Conductor remains uncharged when current flows through it.
c) Types of current: According to its magnitude and direction we categories the current in two
types.
i) Direct Current (DC) : If the magnitude and direction of current does not vary with time, it is
said to be direct current(DC). This type of current is provided by a Cell, a battery or DC dynamo.
ii) Alternating Current (AC) : If a current is periodic (with constant amplitude) and has half
cycle positive and half negative, it is said to be alternating current(AC). This type of current is
generally sinusoidal in nature. An AC dynamo provides this type is current.

Dr.K.K.R Gowtham 2 Current Electricity


Alternating current (ac) Direct current (dc)
(i) (i) (Pulsating dc) (Constant dc)
i
+
i i

– t

t t
Magnitude and direction both varies with
dc  Inverter  ac
time

ac  Rectifier  dc

(ii) Shows heating effect only (ii) Shows heating effect, chemical effect
and magnetic effect of current
+ –
(iii) It’s symbol is ~ (iii) It’s symbol is

DIFFERENT SITUATIONS PRODUCING CURRENT:


A. DUE TO TRANSLATORY MOTION OF CHARGES
a) If n particles, each having a charge q, pass through a given area in time t, the current is given by
Q nq
I 
t t
x
b) If n particles (each having a charge q) pass per second per unit area, q
v V
A
Q
the current associated with cross-sectional area A is I   nqA V t
t

c) If there are n particles per unit volume in the conductor ( each having a charge q) moving with
Q x
velocity  , then current through cross-sectional area A is I  nqA  nq A
t t

B. DUE TO ROTATORY MOTION OF CHARGE


If a point charge q is moving in a circle of radius r with speed  , then its time
 2 r 
period T    So through a given hypothetical cross-section
  
q q q q
( perpendicular to motion), the current is I   qf  
t T 2 r 2

Where  is the angular velocity of the charge.


C. DUE TO CONVECTIONAL MOTION OF CHARGE

Dr.K.K.R Gowtham 3 Current Electricity


It is possible that a charged body is transported from one place to another . The convectional
current is the current which is developed due to the transportation of charge or the mechanical
transfer of a charge.
If in a discharge tube n1 protons are moving from left to right in t seconds and n2 electrons are
moving simultaneously from right to left in t seconds, then the net current in any cross section of the
discharge tube is

I=
 n1  n2  e (from left to right) here e is the magnitude of charge of electron (or) proton.
t
AMOUNT OF CHARGE FLOWING THROUGH A CROSS-SECTION:
a)If the current is constant with time t, then the charge flowing in a time interval ‘t’ is q=It
t2
b) If the current is varying with time t,then the charge flowing in a time interval t 1 to t2 is q   Idt
t1
Ex:1 : The current in a wire varies with time according to the relation I  a  bt ,where current I is in
2

ampere and time t is in second and a  4 A, b  2 As 2 .


a) How many coulomb pass a cross-section of the wire in the time interval between
t  5s and t  10s ?
b) What constant current could transport the same charge in same time interval?
10 10
Solution a) q   I dt    4  2t 2  dt
5 5

q 603.33
10
2 2
 q  4t  t 3  4 10  5  1000  125  603.33C b) I e    120.67 A
3 5 3 t 10  5
Ex:2
In the Bohr model of hydrogen atom, the electron is pictured to rotate in a circular orbit of radius
5  10 11 m at a speed of 2.2  106 ms 1 . What is the current associated with electron motion?
Solution The time taken to complete one rotation is
2 r q e e 1.6 1019  2.2 106
T Therefore, the current is   I   1.12mA
 t T 2 r 2  3.14  5 1011
Ex:3 n1 electron/sec passes through a given cross-section towards right with velocity v1 and n2
proton/sec passes through the same cross-section with velocity v2 . Find the current through a
i1
i1
given cross-sectional. Put n1  1.5 10 and n2  10
10 10

q N1q1 dN1
Solution : i1    q1
t t dt
i
dN
i2  2 q2  i  i1  i2
dt i0
 dN   dN 
  1   e    2  e  i   n2  n1  e t
 dt   dt  T

 1.5 1010  11010 1.6 1019  0.5 109 amp

Dr.K.K.R Gowtham 4 Current Electricity


Ex:4 : A current i varies with time in a coil of resistance R as shown in the graph. Find the (a) total
charge flown(b) average current(c) heat dissipated in the resistor.
Solution : (a) The total charge flow is
q  idt
i
 Area under i  t graph i0
1 iT
 Ti0  0
2 2 T
t
q iT i
b) The average current  iav   0 / T  0
T 2 2
ur
CURRENT DENSITY J : ()
Current density at a point is defined as a vector having magnitude equal to current per unit
area. The direction of J is in the direction of current.
If a current  I passes normally to an area  s as shown, then the current density at a point will be
ds
 ds
  J
I J
n̂ I

ur I dI
J  Lt  nˆ
 s 0 s ds
If the normal to the area makes an angle  with the direction of the current, then the current
I ur uur ur uur
density is J 
 s cos 
, dI  Jds cos (or) dI  J . ds 
i.e, I  J . ds


SI Unit of J is A m2 ,Dimensional formula of J is  AL2 
 
 
Note:1. Though both current I and current density J have directions ,by definition current density J is a
vector while current is a scalar.
2. In terms of current density, Current is the flux of current density.
3. In case of uniform flow of charge through a cross-section normal to it as I=nqv
 I r  r r
So J  nˆ   nqv  n or J  nqv  v ch arg e
S
i.e., current density at a point is equal to the product of charge density with velocity of charge
distribution at that point.
 
4. If current density J is uniform for a normal cross-section S :
r uur r uur r ur 
I   J .dS  J . dS  J .S  JS cos 00  JS (as J =constant)

From this result it is clear that;


i)If current is constant, j 1/ S ,i.e., for a given current, lesser the
area, greater will be the current density and vice-versa.(fig.A)

Dr.K.K.R Gowtham 5 Current Electricity


ii)If current density is constant, I  S, i.e., for a given current density,
smaller the area of cross-section, lesser will be the current.(fig.B)

iii)If cross –section is constant ,I  J, i.e., for a given cross-sectional


area, greater the current density, larger will be the current.

J
r
Ex-1 The current density J  2r. Find the electric current flowing dA
radially away of the spherical region of radius R=1m R

Solution : Since, dA  dA r and J  2r


r r
we have iÑJ   2rˆ . dArˆ 
 .dA  Ñ
 dA  2  4 R   8 1  25.14amp
 2Ñ 2 2

Ex-2 ˆ / m2 . Find the current passing through the area A  2m2 .


The current density J  iA

A
1
 iˆ .2.  1amp
r r
Solution : i  J . A  JA cos 600 A
2 600

 
J  1A / m 2
2
Ex-3: The current density at a point is J  2 10 j Jm .
4
300


Find the rate of charge flow through a cross sectional area S  2i  3 j cm2 
Solution

 J .d S   2 10   j. 2i  3 j  10 A  6 A


4 4
The rate of flow of charge = current = I =
Ex-4:
A potential difference applied to the ends of a wire made up of an alloy drives a current through
it. The current density varies as J =3 +2r. Where r is the distance of the point from the axis. If R
be the radius of the wire, then the total current through any cross section of the wire.
Solution
Consider a circular strip of radius r and thickness dr

dI  J .d S   3  2r  2 rdr  cos 00  2 3r  2r 2 dr 
R
 3r 2 2 3   3R 2 2 R 3 
0 2  3r  2r 
R 2
dr  2   r   2    units
 2 3 0  2 3 
DRIFT VELOCITY
In a conductor, the atoms are quite close to each other and are
strongly bound to one another. However, valence electrons in an
atom of a conductor do not remain attached to a particular atom.
B'
These electrons are free to move through the lattice of positive ions in B
the conductor. Such valence electrons in a conductor are called free
electrons or conduction electrons. The free electrons in a conductor
are always in a continuous random motion (figure.) This random A A' E

Dr.K.K.R Gowtham 6 Current Electricity


motion of free electrons is due to thermal energy of the conductor. Likewise, the velocity of free
electron due to thermal energy of the conductor is termed as its thermal velocity and is usually very
large.
Further, the number of free electrons in a conductor is very large. Due to their random motion, the
free electrons keep on suffering collisions with the positive ions in the conductor so frequently that
the net flow of electrons in any particular direction is zero. In other words, average thermal velocity
of free electrons in a conductor is zero.
r r r r
If u1 , u 2 , u 3 ,...........u n are random thermal velocities of n electrons in a conductor, then their
average thermal velocity i.e.
r r r r
u1 + u 2 + u 3 + ........... + u n
= 0 .......... (1)
n
However, when some potential difference is applied across the two
E
ends of a conductor, an electric field is set up. If V is potential + F eE -
-
difference applied and, l the length of the conductor (figure.), then
V
magnitude of the electric field set up is given by E=
l v
The direction of electric field set up in the conductor is from
positive end of the conductor to its negative end as shown in the figure. Since charge on an electron
r ur
is -e , each free electron in the conductor experiences a force F = - eE ........
(2)
The direction of force is opposite to the direction of electric field. If m is mass of the electron, then
ur
r eE
acceleration produced is given by a= - ......... (3)
m
Under the effect of electric field, the free electrons accelerate and acquire a velocity component in a
direction opposite to the direction of electric field in addition to their thermal velocities. However,
the gain in velocity of an electron due to electric field takes place only for a very short time. It is
because, as the electron accelerates, it gets scattered or deflected on suffering collision against the
positive ion in the conductor. The electron, then starts afresh with a random thermal velocity.
The short time, for which a free electron accelerates before it undergoes a collision with the positive
ion in the conductor, is called relaxation time
r
Therefore, if an electron having random thermal velocity u1 accelerates for a time t 1 (before it
suffers a collision), then it will attain a velocity
r r r
v1 = u1 + a t 1
r
Where a the acceleration, given by equation (1). Similarly, velocities acquired by the other
electrons in the conductor will be
r r r r r r r r r
v2 = u 2 + a t 2 , v3 = u 3 + a t 3 ,......vn = u n + a t n
The drift velocity is defined as the average velocity with which free electrons in a conductor
get drifted under the influence of an external electric field applied across the conductor which
r
is responsible for current through it.. It is denoted by v d . Thus ,
r r r r r r r r r r
r
vd =
v1 + v2 + v3 + ........ + vn
=
(u + a t )+ (u
1 1 2 ) (
+ a t 2 + ...... + u n + a t n )
n n

Dr.K.K.R Gowtham 7 Current Electricity


r r r
r æu1 + u 2 + ......u n ö
÷ r æt + t + ......t ö
= Þ vd = çç ÷+ a çç 1 n÷
çç ÷
÷ çè
2
÷
÷
è n ÷
ø n ø
t 1 + t 2 + ......t n
Now, is called average relaxation time and is denoted be t . As obtained in
n
r r r
u1 + u 2 + ......u n
equation is equal to zero. Therefore, the above equation become
n
r r r
vd = 0 + a t = a t
ur
r eE
Using equation (1), we have vd = - t .........(4)
m
The equation (4) gives drift velocity of free electrons in a conductor in terms of electric field and
average relaxation time.
RELATION BETWEEN CURRENTDENSITY AND DRIFT
VELOCITY: J
To relate current, a macroscopic quantity, to the microscopic
motion of the charges, let’s examine a conductor of cross- vd
sectional area A, as shown in figure. q


Let the total current through a surface be written as I  J . dA
A x  vd t
Where J is the current density (the SI unit of current density
2
are Am ). If q is the charge of each carrier, and n is the number
of charge carriers per unit volume (also called “Charge
Carrier Density”), the total amount of charge in this section is
then Q  q  nAx  .
vd
Suppose that the charge carriers move with a speed  d , then
the displacement in a time interval t will be x  d t .
So, by definition of average current, we get
Q E
I avg   nqAd …(2)
t Motion of electron in a conductor
The speed  d at which the charge carriers are moving is
known as the drift speed. Physically,  d is the average speed of the charge carriers inside a
conductor when an external electric field is applied. Actually an electron inside the conductor does
not travel in a straight line. Instead, it path is rather erratic, as shown in figure. From the above
equations (1) and (2), the current density J can be written as J  nqd …(3)
Thus, we see that J  nqd and  d point in the same direction for positive charge carriers, in
opposite directions for negative charge carriers.
Note : a) A current carrying conductor do not create electric field as it is neutral. But it has an
electric field in it
b) When electrons are drifted in a conductor, they will have net average velocity but they will have
zero average acceleration.
c) In the absence of external electric field, there will be no current in the conductor.
d) The drift velocity of electrons is of the order of 10-4 ms–1 very small as compared to thermal
speed of electrons at room temperature (106 ms–1).

Dr.K.K.R Gowtham 8 Current Electricity


e) The direction of drift velocity for electrons in a metal is opposite to that of applied field
 
E (i.e., current density J )
f) Drift velocity depends on electric field i.e, v d µ E . So, greater the
electric field, larger will be the drift velocity.
(g) When a steady current flows through a conductor of non-uniform vd 2
cross-section drift velocity varies inversely with area of cross-section i A1 < A2
v d1
 1 so vd1  vd 2
v d   i
 A A1 A2

If a steady current flows in a metallic conductor of non uniform cross section


(i) Along the wire I is same (ii) Current density and drift velocity depends on area
I 2  I 2 . A1  A2  J1  J 2 , E1  E2 , vd1  vd2

(h) If diameter (d) of a conductor is doubled, then drift velocity of


electrons inside it will not change.
+
V – + V –

Less – d More – d
Same – vd Some – vd

i)When a current flows through a metallic conductor of non-uniform area of cross section, then at
any cross section
1) constants : charge, current 2) variables :current density, drift velocity, electric field.
MOBILITY (µ) : Mobility (µ) of a charge carrier (like electron) is defined as the average drift velocity
resulting from the application of unit electric field strength.
drift velocity v
= \ m= d
electric field E
Mobility is positive for both positive current carriers and negative current carriers, although their
drift velocities are opposite to each other. SI unit of mobility is m2s–1V–1 or ms–1 N–1C
Mobility depends on pressure and temperature.
FACTORS AFFECT THE DRIFT VELOCITY OF ELECTRON:
Relaxation time () : The time interval between two successive collisions of electrons with the positive
mean free path 
ions in the metallic lattice is defined as relaxation time   r.m.s. velocity of electrons  v .
rms
With rise in temperature vrms increases consequently  decreases.
l
Drift velocity depends on the nature of metal, i.e., relaxation time t . And as t =
v rms
with l =mean free path. With rise in temperature due to increase in v rms and decrease in l ,
relaxation time and hence drift velocity will decrease.
Note: Even though the drift velocity is of the order of 10–4m/s, an electric bulb at home glows
immediately after it is switched on. How is it possible?
When we close the circuit, the electric field is set up in the entire closed circuit instantly with the
speed of electromagnetic wave which causes electron drift at every portion of the circuit due to

Dr.K.K.R Gowtham 9 Current Electricity


which the current is set up in the entire circuit instantly. The current so set up does not wait for the
electrons to flow from one end of the conductor to other end. It is due to this reason, the electric
bulb glows immediately when switched on.
RELATION BETWEEN CURRENT AND DRIFT VELOCITY:
Consider a conductor of length l and of uniform area of cross - section A. Volume of the conductor = Al.
If n is the number of free electrons per unit volume of the conductor, then the total number of free
electrons (N) in the conductor is N = Aln
If e is the charge of each electron, then total charge on all the free electrons in the conductor is
q = N . e = Al ne l
A
Suppose a constant potential difference V be applied across the ends vd
of the conductor with the help of a battery. The electric field set up
across the conductor is given by E = V/l (magnitude). Due to this E
field, the free electrons present in the conductor will begin to move
with a drift velocity vd towards the left hand side as shown in Fig. I I

 Time taken by the free electrons to cross the conductor is  


q A l ne I
t = l / vd Hence current, I = = I = A nevd \ vd =
t l / vd nAe
This equation gives the relation between the current flowing through the conductor and drift
velocity of the electrons.
eE nAe2
Also we know that drift velocity is related to relaxation time τ as v d = t I E
m m

Ex-: Find the electric current in a conductor (copper) of cross- section A  1mm2 , conduction electron
density n  8.69 10 / m and drift speed vd  1cm / s
28 3

Solution :i  nevd A  8.69 10 1.6 1019 102 1104  8.69 1.6 105 amp
28

ur
r eE ne 2
OHM’S LAW: Using J  nqd v =-
and d
t  J= E  J   E ----(a)
m me
Where  depends only on the material of the conductor and its temperature. This constant is called
the electrical conductivity of the material. Equation (a) is known Microscopic Ohm’s Law. The
1
resistivity of a material is defined as 

Ohm’s law tells us that the conductivity (or resistivity) of a material is independent of the electric
field existing in the material. This is valid for conductors over a wide range of field.
“Macroscopic” version of the Ohm’s law:
Consider a conductor of length l and uniform cross sectional area A. Let a potential difference V
between the ends of the conductor. The electric field inside the conductor is
E = V/l .If the current in the conductor is I, the current density is J=i/A.
Ohm’s law J   E then becomes
i V 1 l l l
 or V  i   i Or V=Ri
A l  A A A
vd

Dr.K.K.R Gowtham 10 Current


E Electricity

I I

 
l
Where R=  is called the resistance of the given conductor.
A
1
The reciprocal the resistance is called conductance, denoted by G G =
R
The equation V=iR is “Macroscopic” version of the Ohm’s law:
which is widely used in circuit analysis.
“For a given conductor, at a given temperature the strength of electric current through it is directly
proportional to the potential difference applied across it”.
Let V be the potential difference applied across the conductor and I be the current flowing through
it. According to Ohm’s law,
1
I  V i.e  I  V Where R is electrical resistance of the conductor.
R

RESISTOR: An object of conducting material, having a distance of desired value, is called a


resistor.
RESISTANCE :The property by virtue of which a conductor opposes the flow of charge in it is known
as electric resistance
Definition: The resistance of a conductor is defined as the ratio of the potential difference ‘V’
across the conductor to the current ‘i’ flowing through the conductor.
V
Re sis tan ce R = Units of resistance: volt/ampere (or) ohm, denoted by the symbol 
i
The resistance of a specimen is said to be one Ohm if one Volt potential difference across it
causes a current of one Ampere to flow through it.
1volt
1ohm  Resistance of a conductor is a scalar quantity and is characteristic of the
1ampere
specimen as a whole. It depends on the nature of the material of the specimen, dimensions (length,
area of cross section) of the specimen, and physical conditions like temperature, pressure and
impurities. Resistance is the bulk property of the conductor.
The below figures shows the symbols of fixed resistor and variable resistor.

(a) Fixed resistor (b) Variable resisotr


Cause of Resistance in a metallic conductor: When a potential difference is applied across a
conductor, then electric field is set up in which free electrons gets accelerated. As a result, they
collide against the ions and atoms and their motion is thus opposed. This opposition offered by ions
and atoms due to collisions is termed as the resistance of the conductor.
CONDUCTANCE: The inverse of resistance (R) is called conductance of a conductor, i.e
Conductance, G  1 . The S.I unit of conductance is mho or siemen(S).
R

RESISTIVITY (  ):
As we know, that the resistance of the conductor is directly proportional to its length and inversely
ρ
proportional to its area of cross section, we can write R R
A A

Dr.K.K.R Gowtham 11 Current Electricity


where ρ is specific resistance or resistivity of the material of the conductor.
Now, let us define the specific resistance of the conductor.If = 1 m, A = 1 m2, then  =R.
The resistance of a conductor of unit length and unit area of cross section is called specific
resistance or resistivity of the material of the conductor. SI unit of specific resistance: ohm-m
A perfect conductor would have zero resistivity, and a perfect insulator would have an infinite
resistivity. Metals and alloys have the smallest resistivities and are the best conductors. The
resistivities of insulators are greater than those of the metals by an enormous factor, of the order
of 10 22 .
CONDUCTIVITY (  ):Conductivity is the measure of the ability of a material to conduct electric
1 l
current through it. It is reciprocal of resistivity.  = 
 RA

S.I. unit: siemen / m ; (Sm–1) or mho metre 1  1m 1 
For Insulators  = 0 For perfect conductors,  is infinity.
Electrical Circuits:
The devices which deliver electricity are called the 'sources' (cells, battery , generator )and lamps,
electric motors, etc. which consume electricity are called 'loads' or ‘electric appliances’. Switches
are used to switch the loads on and off whenever necessary. Hence, the source, the load, and the
switch are connected together using conducting wires to form electrical circuits.
The circuit is called 'closed' when the current flows in the circuit, and when the current does not
flow, it is called an 'open circuit'. The switch or key helps to make or break the circuit, i.e. switches
on or switches off the current.
GALVANOMETER
A galvanometer is an instrument for detecting electric current. (flow of electric
charge) through an electric circuit. It is always connected in series in the circuit and
has very low resistance. Its needle deflects as long as current flows through it.
AMMETER
An ammeter (from Ampere Meter) is a measuring instrument used to measure the current in
a circuit. Electric currents are measured in amperes (A), hence the name. Instruments used to
measure smaller currents, in the milliampere or microampere range, are designated
as milliammeters or microammeters.Analog ammeters move a pointer across a scale in proportion
to the current in the circuit; digital ammeters give a numerical display of
current by use of an analog to digital converter.

A ammeter in a circuit diagram is represented by the letter A in a


circle.
It is always connected in series in the circuit and has very low
resistance. Because of finite resistance the reading shown by it is
slightly less than actual current. The resistance of an ideal
ammeter is zero.

Dr.K.K.R Gowtham 12 Current Electricity


VOLTMETER
A voltmeter is an instrument used for measuring electrical potential difference between two points
in an electric circuit or P.D across a electric device. Analog voltmeters move a pointer across a scale
in proportion to the voltage of the circuit; digital voltmeters give a
numerical display of voltage by use of an analog to digital converter.
A voltmeter in a circuit diagram is represented by the letter V in a circle.
For measuring potential difference across the device or
potential difference between two points in an electric circuit,
voltmeter is always connected parallel to the device or
parallel to the points. To measure the actual P.D the volte
meter has infinite resistance. If resistance is infinite
measurement of P.D is not possible, since no current enters
into it. Hence voltmeter has very large resistance. Because of
finite large resistance, the reading shown by it is slightly less than actual P.D.
The resistance of an ideal voltmeter is infinite.
RHEOSTAT :
Rheostat is a variable resistor, which is used to control the flow of electric current by manually
increasing or decreasing the resistance. The electric current flowing through an electrical circuit is
determined by two factors: the amount of voltage applied and the total resistance of the electrical
circuit. If we reduce the circuit resistance, the flow of electric current through the circuit will be
increased. On the other hand, if we increase the circuit resistance, the flow of electric current
through the circuit will be decreased.
By placing the rheostat in the electrical circuit, we can control (increase or decrease) the flow of
electric current in the circuit. Rheostat reduces the electric current flow to certain level. However, it
does not completely blocks the electric current flow. To completely block the electric current flow,
we need infinite resistance. Practically it is not possible
to completely block the electric current.
Generally, rheostats are made by winding the
Nichrome wire around an insulating ceramic core.
The ceramic core of the rheostat acts as the
insulating material to the heat. Hence, the ceramic
core does not allow heat through it.

Symbol of rheostat

Applications of rheostat
 Rheostats are used in dim lights to change the intensity of light. If we increase the resistance of
the rheostat, the flow of electric current through the light bulb decreases. As a result, the light
brightness decreases. In the similar way, if we decrease the resistance of the rheostat, the flow of
electric current through the light bulb increases. As a result, the light brightness increases.
 Rheostats are used to increase or decrease the volume of a radio and to increase or decrease the
speed of an electric motor.

Dr.K.K.R Gowtham 13 Current Electricity


CIRCUIT SYMBOLS:
Sl.No. Components Symbols

01 An electric cell

02 A battery or a combination of cells

03 Plug key or switch (open)

04 Plug key or switch (closed)

05 A wire joint

06 Wires crossing without joining

07 Electric bulb

08 A resistor or resistance R

09 Variable resistance or rheostat

10 Ammeter

11 Voltmeter

galvanometer
12

VERIFICATION OF OHM'S LAW:


Ohm’s law states that at a constant temperature, current I through a conductor between two
points is directly proportional to the potential difference or voltage, V, across the two points.
Thus, the ratio V: I is constant, and is called the resistance, R, of the conductor.
In other words, V/I = constant = R, or V = IR, where R is a constant for the given metallic wire at
a given temperature and is called its resistance.

Dr.K.K.R Gowtham 14 Current Electricity


Material required :
A resistor of about 5 Ω, an ammeter ( 0 - 3 A), a voltmeter (0 - 10 V), four dry cells of 1.5 V
each with a cell holder (or a battery eliminator), a plug key, connecting wires, and a piece of sand
paper.
Precautions :
 All the electrical connections must be neat and tight.
 Voltmeter and Ammeter must be of proper range.
 The key should be inserted only while taking readings.

Circuit Diagram:

Procedure :

1. Draw the circuit diagram as shown above.


2. Arrange the apparatus as per the circuit diagram.
3. Clean the ends of the connecting wires with sand paper and make them shiny.
4. Make the connections as per circuit diagram. All connections must be neat and tight. Take care to
connect the ammeter and voltmeter with their correct polarity. (+ve to +ve and -ve to -ve).
5. Adjust the rheostat to pass a low current.
6. Insert the key K and slide the rheostat contact to see whether the ammeter and voltmeter are
showing deflections properly.
7. Adjust the rheostat to get a small deflection in ammeter and
voltmeter.
8. Record the readings of the ammeter and voltmeter.
9. Take atleast six sets of readings by adjusting the rheostat gradually.
10. Plot a graph with V along x-axis and I along y-axis.
11. The graph will be a straight line which verifies Ohm's law.
12. Determine the slope of the V-I graph. The reciprocal of the slope
gives resistance of the wire.

VALIDITY (OR) LIMITATIONS OF OHM’S LAW:


a) Ohms law is valid only for linear conductors or ohmic conductors such as metals and alloys of
metals. (In other words, the resistance of the device is independent of the magnitude and polarity of
the applied potential difference).
For linear or ohmic conductors, I versus V(potential difference) graphs are straight lines passing
through origin as shown in A and B. Y
I 1
A) slope = tan  = = = G ,at the given temperature.
V R I
Y q
V X
B) slope = tan  = = R , at the given temperature V V
I
q
X
I

Dr.K.K.R Gowtham 15 Current Electricity


Resistance of a ohmic conductor never be negative.
b) Ohm’s law is not universal as it does not hold good in case of gases, crystal - rectifiers, thermionic
valves, transistor, carbon, mica, vacuum tubes, thermistor and most of the electrolytes. The devices
or substances which do not obey ohms law (i.e. the resistance is not constant, but changes in a way
that depends on the voltage across it.) are called non-ohmic or non linear conductors.
In this case V versus I graph is not a straight line, but has some curvy shape as shown for different
materials.. Such devices do not have a constant value of resistance and the resistance is called
dynamic resistance because it is constantly changing.

Y
I
1.5 mA
I
i
-2 0.2 V
V
A
O X
V
Thermistor Semi-conductor Vacuum tube r
Note : V = IR is applicable for all linear and nonlinear conductors but, ohm’s law is not applicable for
nonlinear conductors.
1. In general, electrolytes does not obey Ohm’s law. But copper sulphate solution with copper
electrodes obey Ohm’s law even though it is an electrolyte.
Static and Dynamic Resistance:
Static Resistance is the normal ohmic resistance in accordance with Ohm's Law. It is the ratio of
voltage and current and is a constant at a given temperature.
Static Resistance = Voltage / Current=V/I
Dynamic Resistance:
Dynamic Resistance is a concept of resistance used in non-ohmic conductors or devices. Dynamic
resistance refers to the change in current in response to a change in voltage at a specific region of
the VI curve.
Incase of non-ohmic conductors, the resistance depends on P.D across it. The value of resistance at
a given P.D (V) is called dynamic resistance.
When the voltage is changed, the current may not change proportionally. Examples of such devices
are tungsten filament (bulb), diode, thermistor etc...
1 v dv
 lim  =inverse slope of tangent to I-V curve at a given voltage V or current I
r v  0 i di

 r = slope of tangent to V-I curve at a given voltage V or current I


Note:1. Resistance of a ohmic conductor(static resistance) never be negative.
2. Dynamic resistance may be positive or negative

Dr.K.K.R Gowtham 16 Current Electricity


Dynamic resistance at
At A - infinite
At B -ve
At C - infinite
At D +ve
At E zero

COLOUR CODE FOR CARBON RESISTANCES:


For different electrical and electronic circuits we require resistors of different values. Resistors
may be divided into two groups; wire wound resistors and carbon resistors. In a wire wound resistor
a resistance wire (of magnanin, constanton or nichrome) of definite length according to value of
resistance is wound two fold over insulating cylinder to make it non inductive.
To make carbon resistor, carbon with a suitable binding agent is moulded into a cylinder. Wire leads
are attached to this cylinder for connecting it to an electrical circuit. The value of resistance is
indicated by four coloured bands marked on the cylinder (Fig.) and meaning of different colours are
given in table.
Colour bands on a resistor Colour Digit Multiplier Tolerance
Black 0 1
Brown 1 10
Red 2 102
Orange 3 103
Yellow 4 104
T Green 5 105
he Blue 6 106
table may be memorised by the mnemonic Violet 7 107
BB ROY of Great Britain has a Very Good
Gray 8 108
Wife.
Capital letters represent the first letter of colour. White 9 109
The colour bands are formed from Gold 5%
Silver 10%
No colour 20%
left to right. The first three bands  A, B and C  give value of resistance. The colours of first and
second bands indicate the first and second significant digits while the colour of third bond gives
decimal multiplier (e.e., the number of zeros which follow the first two digits). The colour of fourth
band represent its tolerance. Absence of any colour means a tolerance of 20%.

A BC D
R  AB  10c 
Tolerance (in%)
e.g.1 if A is red, B is green, C is grey and there is no colour at D, then
R  25 108  20%(25 108 )
 25 108  5 108 
Its resistance is between 20  10  to 30 10 
8 8

Dr.K.K.R Gowtham 17 Current Electricity


Ex- Suppose the colours on the resistor as shown in Figure are brown, yellow, green and gold as read
from left to right. Using the table, find the resistance of the resistor
Brown Yellow Green Gold

 5 
   (1.4  0.07)10   (1.4  0.07)M 
6
 14 105 1 
 100 
Carbon resistances find their wide use in electronic circuits at low voltages due to following
reasons.
a) They may have values ranging from few ohms to 100M .
b) They are made up of small handy sizes.
c) They are quite cheap.
FACTORS AFFECTING THE RESISTANCE OF A CONDUCTOR
1) The resistance of the conductor is directly proportional to the length (l) of the conductor
R1 R 
i.e R  l (or)  1
For small changes in the length, 
R2 2 R
2.The resistance of a conductor is inversely proportional to the area of cross-section (A)
1 R1  A2   r2 2 
i.e, R  1 (or) R  2    
A r R2  A1   r12 
R A 2 r
For small changes in area (or) radius we have  
R A r
l 2
l d V  m
2
Special Cases:. The alternate forms of resistance is R      2  2
V m A d A
Where d is density of material of conductor V is volume of the conductor m is mass of the
conductor.
2. Stretching of Wire:
If a conductor is stretched or elongated or drawn or twisted, then the volume of the conductor is
l 2 V 1 1
constant. Hence a) R  R  l2 b) R  2
 R 2
 4
V A A r
2
l m m
c) In terms of mass of the wire R  and R  2  4
m A r
R l R A r
3) For small changes in the length or radius during the stretching 2  2  4
R l R A r
4) The resistance between end faces of hollow cylindrical tube of length l, inner radius ri and outer
l
radius ro is given by R 
  r02  ri 2 
h
F
5) Resistance of a conducting body is not unique but depends on it’s C
length and area of cross-section i.e. how the potential difference is b B
A
applied. In case of a solid cuboid of dimensions l ´ b´ h is D
E l

Dr.K.K.R Gowtham 18 Current Electricity


l
Resistance across AB, RAB 
b h
b h
Resistance across CD, RCD  Resistance across EF, REF 
lh l b
l h
If l > b > h, then Rmax  Rmin 
b h l b
1
Ex1: Find the resistance of a conductor of length l  m area of cross-section A  103 m2 , having a
2
current density J  100 A / m when an electric field of 2 volt/m is applied inside the conductor.
2

 2  
1

Solution : R 
V
, Where V  El and i  JA Then, R 
El
 2  10
i JA  1 
100   
 1000 
Resistance of a Conductor of Non-uniform Cross Section:
Ex2: Find the resistance of the conductor of conductively  between the points 1 and 2.

r1 r2
1
l 2

Solution : The potential difference across the thin strip is r1 r r2


x dx
dV  E.dx

l
Integrating dV, the potential difference between the terminals 1 and 2 is V   E.dx …(i)
0
The total current passing through the conductor is i   J  r 2    E   r 2
r2  r1 r r
2
 
...  ii 
l
Putting r  r1  x, we have i    E  r1  2 1
x
l 0
 l 
v
From eqs. (i) and (ii) , putting v and i in Ohm’s law R  , we have
i
l

R

0
E dx
r2  r1 
2
l 
0  E  r1 
 l
x

Since E is uniform in the conductor and  is a constant pull, then out of the integral to obtain
1 l dx l l
R 
   r2  r1  
0 2
Or R 
 r1r2  r1r2
r1  x
 l 

Dr.K.K.R Gowtham 19 Current Electricity


Ex:3 (Resistance of cylindrical conductor).
Find the resistance of the hollow cylindrical conductor between r1 1 2
the points 1 and 2 . Assume  =conductivity of the conductor . r2

l
Solution : The potential difference across the thin cylindrical shell is dV  E dr

Integrating dV, the potential across the given cylinder is


...  i 
r2
V   E.dr
r1 J
E
The current density at any point of the thin cylindrical shell is
r dr
i r1
J
2 rl
Then , the electric field in thin cylindrical shell is
J i
E  ...  ii 
 2 rl
i r2 dr V 1 r
2 l 
Substituting E from eq.(ii) in eq.(i), we have V  Then, R   ln 2
r1 r i 2 l r1
Ex-4
Figure shows a conductor of length l carrying current I and having
a circular cross – section. The radius of cross section varies
linearly from a to b. Assuming that (b –a) << l. Calculate
current density at distance x from left end.

Solution
Since radius at left end is a and that of right end is b. therefore increase in radius over length l is
ba
(b- a). Hence rate of increase of radius per unit length =   increase in radius over length
 l 
ba ba
   Since radius at left end is a so radius at distance x, r  a   
 l   l 
2
 ba 
Area at this particular section A   r    a  2
 x
  l  
1 1 1
Hence current density J   
A r 2
 x b  a  
2

 a  
 l 
TEMPERATURE DEPENDENCE OF RESISTIVITY:
Resistance of Pure Metals
m l
We know that R 
ne2 A
For a given conductor, l, A and n are constant, hence R  (1/)
If  represents the mean free path (Average distance covered between two successive
collisions) of the electron and vrms, the root-mean-square speed, then,
 vrms
 Hence R Now,
vrms 
(a)  decreases with rise in temperature because the amplitude of vibrations of the +ve ions of
the

Dr.K.K.R Gowtham 20 Current Electricity


metal increases and they create more hindrance in the movement of electrons and,
(b) (i) vrms increases because vrms  T. Therefore, Resistance of the metallic wire increases
1
with rise in temperature. As  R and  hence resistivity increases and

conductivity decreases with rise in temperature of the metallic of the metallic wires.
As the temperature of a conductor is increased, the thermal agitation increases and the collisions
become more frequent. Hence, the average time between the successive collisions decreases and so
drift speed decreases. Thus the conductivity decreases and the resistivity increases as the
temperature increases. Hence for small temperature variations we can write
r t = r 0 (1+ a t + bt 2 + .......) The values of  and  very from metal to metal.
If temperature t is not sufficiently large (as in most practical cases), then equation may be expressed
as(Neglecting higher orders) r t = r 0 (1+ a t )
where  is a constant, called temperature coefficient of resistance (or resistivity).
Since resistance is proportional to resistivity for a given conductor we can write


Rt  R0 1   t   t 2  ....... Neglecting higher orders Rt = R0 (1+ a t )t
Resistance of semiconductors
(i) There are certain substances whose conductivity lies in between that of insulators and
conductors, higher than that of insulators but lower than that of conductors. These are called as
semiconductors,
e.g., silicon, germanium, carbon etc.
(ii) The resistivity of semiconductors decreases with increase in temperature i.e.,  for
semiconductors is -ve and high.
(iii) Though at ordinary temperature the value of n (no. of free electrons per unit volume) for these
materials is very small as compared to metals, but increases very rapidly with rise in temperature
(this happens due to breaking of covalent bonds). Though t decreases but factor of n dominates.
m l
Therefore, the resistance R  goes on decreasing with increase in temperature.
ne2 A
Graph shows the variation of resistivity with temperature for
conductors , semi conductors and for alloys like manganin and Y rs
cto
constantan. Since the resistivity of manganin and constantan ndu
co
remains constant with respect to change in temperature, these for
Resistivity

materials are used for the bridge wires and resistance coils. for manganin and constantan
for
The resistivity of managing and constantan is almost s em
ico
independent of temperature. ndu
cto
rs
TEMPERATURE COEFFICIENT OF RESISTANCE:
O X
The resistance of a conductor varies with temperature t as Temperature
Rt  R0
Rt = R0 (1+ a Vt ) Þ  av   R
R0Vt Rt
If R0 = 1 and t = 10 C, then ;  = Rt – R0.
Thus temperature coefficient of resistance is equal to change in resistance of a wire of resistance
one ohm at 00C when temperature changes by 1oC.   lim R = 1 dR
t
Vt ® 0 Rt R dt
If the resistance of a wire at temperature t10C is R1 and at t 02C is R 2 , then

Dr.K.K.R Gowtham 21 Current Electricity


R1 1  t1
R1  R0 (l+  t1 ) and R 2  R0 (1   t2 ) On dividing 
R2 1  t2
R2  R1
So that  ( per 0C )
R1t2  R2t1
 is positive for metals or linear conductors.
 is negative for semi conductors or non linear conductors.
The resistivity of some materials like carbon, silicon, germanium etc. decrease with increase of
temperature that is temperature coefficient of resistivity for these materials is negative.
Ex-1 Resistance of a wire at temperature t0C is R = R0(1 + at + bt2) Here, R0 is the temperature at 00C.
Find the temperature coefficient of resistance at temperature t .
1 dR 1  a  2bt 
Sol :  .  [ R0 (a  2bt )]    2 
R dt R0 (1  at  bt 2 )  1  at  bt 
Ex-2 The current –voltage graphs for a given ohmic resistor at two different temperature are as shown.
Compare T1 and T2.
1 1
As R2  , R2  and tan  1 > tan  2
tan  2 tan  2
R2  R1
 T2  T1
Ex-3 V- I graph of a conductor at temperature T1 and T2 are shown in the figure (T2 – T1) is
proportional to V
T2
Sol : Slope of line gives resistance T1

So, R1  tan   R0 1  T1 


R2  tan(90  )  cot   R0 1  T2  cot   tan   R0 T2  T1  or
I

cos  sin  cos 2


or   R0  T2  T1  R0  T2  T1   T  T  cot 2
sin  cos   sin 2  2 1
2
 1  d
Ex-4: The temperature coefficient of resistivity  is given by     , where  is the resistivity
   dT
at temperature T.
a
a) Assume that  is not constant and is given by    , where T is the absolute temperature
T
b
and a is a constant, show that the resistivity  is given by   n , where b is another constant.
T
 
1
b) Using the values   3.5 105 m and   5 104 C 0 for graphite at 293K , determine
a and b.
c) Using your result from part (b), determine the resistivity of graphite at 196 C . (Remember to
0

express T on absolute scale).


1 d d dT
Solution a)      dT  a
 dT  T

Dr.K.K.R Gowtham 22 Current Electricity



d
T
dT
Let   0 at T  T0 , then 
 
 a 
T0 T
0

   T 
a
T 
     0T0a 
1 b
 log e    a log e    log e  0   a Here, b   0T0a
 0  T 
a
 T0  T T

 C
1
b) Given that 0  3.5 105 m and  0  5 104 0
at T0  293K
 a
a   0T0 as      a  0.15
 T
And b  0T0a  3.5kt105    293 0.15
 8 105 mK 0.15 {be careful while taking units of a and b
b 8 105
c) Now at T  196  273  77 K ,    4.3 105 m
T a  77 0.15
8 105
Similarly at T  300  273  573K   3 105 m
 573
0.15

Ex5: A rod of length L and cross-section area A lies along the x-axis between x=0 and x=L. The
material obeys Ohm’s law and its resistivity varies along the rod according to,
  x   0e x/ L
The end of the rod at x=0 is at a potential V0 and it is zero at x=L
a) Find the total resistance of the rod and the current in the wire.
b) Find the electric potential V(x) in the rod as a function of x.
Solution
x
x=0 dx x=L
a) Resistance of elementary section dx at x=x is,
x
  x  dx  l 0e dx
L
dR   R   dR 
A  A A
Since all such elements are in series, hence
L
0 L x
0 L 0 L  1
R   dR 
A  e L dx  A
1  e 
1
 R 1  
A  e
0 0

V0 V0 A  e 
Current in the wire is given by I   
R 0 L  e  1 
b) dV  IdR
V0 A  e  0e x / L V0  e   x / L
dV  dV    e dx
0 L  e  1  A
.dx
L  e 1 
V  x
V0  e   x / L
x
 
V0
dV 
L  e  1  0
  e dx

 e  V0  e x / L  e1 
V  x   V0  V0   1  e  V  x 
 x/ L
 
 1 e  1  e1

Dr.K.K.R Gowtham 23 Current Electricity


THERMISTOR:
It is heat-sensitive resistor (semiconducting material) such that its resistance varies appreciably
with temperature. The temperature coefficient of resistance of a thermistor is very high.
Thermistors of type 1 have a negative temperature coefficient of resistance i.e., resistance
decreases with increase of temperature.
Thermistors of type 2 have a high positive temperature coefficient of resistance i.e., resistance
increases with increase of temperature.
A mixture of oxides of manganese and nickel are used in the fabrication of thermistors.
Important applications of thermistor are :
1) The thermistor with a high negative temperature coefficient of resistance is used in resistance
thermometer for very low temperature measurement of the order as 10 K.
2) Thermistors can be used to detect even very small changes in temperature of the order as small
as 10–3 K.
3) In a radio circuit, there will be several heater elements in series. A sudden and large surge of
current through the circuit will damage the device. To prevent such a sudden surge of current, we
place a thermistor with a high negative temperature coefficient of resistance, in series in the circuit.
Initially, the thermistor is in a relatively cold state and hence has a very high resistance. This
prevents the current to a moderate level. Later, as the thermistor gets heated, its resistance decreases
and it allows normal flow of current through the heater elements thereby preventing initial surge.
4) A thermistor can also be used to serve as a thermostat.
5) Thermistors are used in temperature-control units.
6) Thermistors can be used for the detection of excessive temperature in an industrial equipment.
7) Thermistors are widely used in measuring the rate of energy flow in microwave beams. The
beam falls on a thermistor and heats it. A relatively small rise in temperature results in a very large
change in resistance because, for a thermistor a is very high. By measuring the change in
resistance, we can accurately measure the microwave power.
CLASSIFICATION OF ELECTRICAL MATERIALS:
All the substances according to their resistivity and its variation with temperature are divided into
the following four classes:
1) CONDUCTORS: These have large number of free electrons ( 1028 / m3 ) and small resistivity
( 108   m ).The resistivity of ideal conductor is zero and it increases with rise in temperature if
metals(e.g.,Cu,Al,Ag,P….) and decreases in non metals(e.g.,graphite).In case of alloys of metals such
as nichrome, manganin or constantan resistivity is more than that of constituent metals but varies
slowly with temperature than metals.
2) SEMI-CONDUCTORS: These have conduction electrons ( 1016 / m3 ) and greater resistivity
( 3X103   m ) as compared to metals. In these electrical conduction is due to motion of
electrons or holes or both. The resistivity of semi-conductors decreases with rise in temperature .At
absolute zero a semi-conductor is an ideal insulator. Silicon (Si) and Germanium (Ge) are most
common examples of semi-conductors.
3. INSULATORS:These have practically no free electrons and very high resistivity( 1016   m ).The
resistivity of ideal insulator is infinite and it decreases with rise in temperature .Examples:
Mica,rubber,glass porcelain(china clay) and fused quartz ,diamond.
4. SUPERCONDUCTORS :
We see that for some materials below a certain temperature resistivity suddenly becomes zero. This
temperature is called critical temperature for this transition. The material in this state is called

Dr.K.K.R Gowtham 24 Current Electricity


superconductor and the phenomenon is called superconductivity. It was observed for mercury in
1911 by Kamerleigh Onnes. Critical temperature for mercury is 4.2 K. The critical temperature
depends on nature of the material. For example, the resistance of mercury becomes zero at 4 K.

Resistivity
Resistivity normal metal
super conductor

Temperature Temperature
From the above graphs, above the critical temperature, the resistivity of the super conductor follows
the trend of a normal metal.
If an electric current is set up in a super-conductor, it can persist for long time even for months and
years after removing the applied potential difference. Superconductivity exists at very low
temperatures which are difficult to obtain. Scientists are trying to prepare compounds and alloys
which would be superconducting at room temperatures (300K). Superconductivity at around 125 K
has already been achieved and efforts are on to improve upon this. Superconductors are used to
construct very strong magnets. Possible applications of superconductors are ultra fast computer
switches and transmission of electric power through superconducting power lines.
Critical temperature for Copper is 43 K,
For Aluminium 1.19 K, for Indium 0.14 K, for Mercury 4.2 K.
NOTE;
1. Resistivity is the specific property of a material but Resistance is the bulk property of a
conductor
2. Resistivity is independent of dimensions of the conductor such as length, area of the cross
section.
3. Resistivity depends on the nature of the material of the conductor, temperature and impurities.
4) Resistivity of metals increases by the addition of impurities. Resistivity of any alloy is more
than the resistivity of its constituent elements.
For example,constantan (alloy of Copper and Nickel) ,manganin (alloy of Copper,Manganese and
Nickel) and nichrome (alloy of Nickel and Chromium) have high resistivities as compared to their
constituent metals.
 insulator   alloy   semi - conductor   conductor
(Maximum for fused quartz) (Minimum for silver )
5) Silver, Gold, Copper ,Aluminium and Iron have very low values of resistivity, so they are used
in the manufacture of electric cables and connecting wires.
Electrical Conducting Materials for Specific Use:
1) The filament of electric bulb is made up of tungsten which has low resistivity and high melting
point(3300K).However, at this high temperature tungsten reacts with air forming oxides so bulb
containing tungsten filament is filled with inert gases like nitrogen or organ.
2) The elements of heating devices are made up of Nichrome(alloy of Nickel and Chromium)
which has high resistivity and high melting point. Nichrome also
Y s
does not react with air. tor
d uc
3) Graph shows the variation of resistivity with temperature for n
co
for
conductors, semi conductors and for alloys like manganin(alloy of
Resistivity

for manganin and constantan


Copper, Manganese and Nickel) and constantan(alloy of Copper
for
and Nickel). Since the resistivity of manganin and constantan s em
ico
remains constant with respect to change in temperature, these ndu
cto
rs
materials are used for the bridge wires and resistance coils.
The resistivity of manganin and constantan is almost independent O Temperature X
of temperature.

Dr.K.K.R Gowtham 25 Current Electricity


4) Fuse wire is made of tin-lead alloy. It should have low melting point, low resistivity. It is used in
series as a safety device in an electric circuit and is designed so as to melt and thereby open the
circuit if the current exceeds a predetermined value due to some fault. The function of a fuse is
independent of its length. Safe current of fuse wire relates with it’s radius as i  r 3/ 2 . In the absence
of radiation loss, the time in which a fuse will melt does not depend on its length but varies with radius
as t  r 4 .
5) The connection wires are made of copper because it has low resistance and resistivity.
BATTERY AND EMF:
A battery is a device which maintains a potential difference between its two terminals A and B.
Figure shows a schematic diagram of a battery.
Some internal mechanism exerts forces on the
charges of the battery material. This force drives the
positive charges of the battery material towards A
and the negative charges of the battery material
towards B. We show the force on a positive charge q
as Fb .As positive charge accumulates on A and negative charge on B, a potential difference
develops and grows between A and B. An electric field E is developed in the battery material from
A to B and exerts a force Fe  qE on a charge q. The direction of this force is opposite to that of
Fb .In steady state, the charge accumulation on A and B is such that Fb = Fe. No further
accumulation takes place.
If a charge q is taken from the terminal B to the terminal A , the work done by the battery force
Fb is W = Fb d where d is the distance between A and B. The work done by the battery force per unit
W Fb d
charge is E= 
q q
This quantity E is called the emf of the battery. The full form of emf is electromotive force. The
name is misleading in the sense that emf is not a force, it is work done/charge. We shall continue to
denote this quantity by the short name emf. If nothing is connected externally between A and B,
Fb = Fe =Eq W = Fb d =qEd =Qv
Fb d
where V = Ed is the potential difference between the terminals. Thus, E = V
q
Thus, the emf of a battery equals the potential difference between its terminals when the terminals
are not connected externally.
Potential difference and emf are two different quantities whose magnitudes may be equal in certain
conditions. The emf is the work done per unit charge by the battery force F b which is non-
electrostatic in nature. The potential difference originates from the electrostatic field created by the
charges accumulated on the terminals of the battery.
A battery is often prepared by putting two rods or plates of different metals in a chemical solution.
Such a battery, using chemical reactions to generate emf, is often called a cell.
The emf is measured in the units of joule/coulomb (JC-1), which is ‘volt’. Thus emf has the same
units as that of voltage or potential difference.
Now suppose the terminals of a battery are connected by a
conducting wire as shown in above figure. As the terminal A is at a
higher potential than B, there is an electric field in the wire in the
direction shown in the figure. The free electrons in the wire move in
the opposite direction and enter the battery at the terminal A. Some
electrons are withdrawn from the terminal B which enters the wire
through the right end. Thus, the potential difference between A and

Dr.K.K.R Gowtham 26 Current Electricity


B tends to decrease. If this potential difference decreases, the electrostatic force Fe inside the battery
also decreases. The force Fb due to the battery mechanism remains the same. Thus, there is a net
force on the positive charges of the battery material from B to A. The positive charges rush towards
A and neutralise the effect of the electrons coming at A from the wire. Similarly, the negative
charges rush towards B. Thus, the potential difference between A and B is maintained.
For calculation of current, motion of a positive charge in one direction is equivalent to the motion of
a negative charge in opposite direction. Using this fact, We can describe the above situation by a
simpler model. The positive terminal of the battery supplies positive charges to the wire. These
charges are pushed through the wire by the electric field and they reach the negative terminal of the
battery. The battery mechanism drives these charges back to the positive terminal against the
electric field existing in the battery and the process continues. This maintains a steady current in the
circuit.
Current can also be driven into a battery in the reverse direction. In such a case, positive charge
enters the battery at the positive terminal, moves inside the battery to the· negative terminal and
leaves the battery from the negative terminal. Such a process is called charging of the battery. The
more common process in which. the positive charge comes out of the battery from the positive
terminal is called discharging of the battery.
ENRGY TRANSFER IN AN ELECTRIC CIRCUIT:
Figure shows a simple circuit in which a register having resistance R
is connected to a battery of emf E through two connecting wires CA
and DB. The connecting wires are assumed to have negligible
resistance. This ensures that potential difference across AC and DB
are zero even when there is a current through them. The potential
difference across the resistor is same as that across the battery. If the
current in the circuit is i, this potential difference is
V=VA –VB = VC –VD = iR.
Thermal Energy Produced In the Resistor:
In time t a charge q=it goes through the circuit. As the charge moves from C to D the electric
potential energy decreases by U=qV=it(iR)=i2 Rt
This loss in electric potential energy appears has increased thermal energy of the resistor. Thus a
current ‘I’ for a time ‘t’ through a resistance R increases the thermal energy by i2 Rt.

KIRCHHOFF’S LAWS
Ohm’s law gives current - voltage relation in simple electrical circuits. But when the circuit is
complicated, it will be difficult to find current distribution by Ohm’s law. Kirchhoff in 1842
formulated the following two laws which enable us to find the distribution of current in complicated
electrical circuits or electrical networks.
(I) KIRCHHOFF`S FIRST LAW (JUNCTION LAW OR CURRENT LAW) : R4
It states that the sum of the currents flowing into a junction is
equal to the sum of the currents flowing out of the junction . I1
R1 I4
> >

< R3
>
Distribution of current at a junction in the circuit A I3

I in   I out I2 R2
I1 + I2 = I3 + I4 or I1 + I2 – I3 – I4 = 0
If we take currents approaching point A in Fig. as positive and that leaving the point as negative,
then the above relation may be written as
I1 + I2 +(–I3) + (–I4) = 0
Hence the first law may be stated in other words that “the algebraic sum of currents at a junction
is zero”. Kirchhoff’s first law tells us that there is no accumulation and supplying of charge at any

Dr.K.K.R Gowtham 27 Current Electricity


point, if steady current flows in it. The net charge coming towards the point should be equal to that
going away from it in the same time. I  0 ........ (1)
Note : Thus, Kirchhoff’s first law is accordance with law of conservation of charge, since no charge
can accumulate at a junction.
Before moving on to the statement of Kirchhoff’s second law, we state some conventions to be
followed in circuit analysis:
(1) Direction of conventional current is from high potential to low potential terminal.
(2) Current flows from high potential node A to low potential node B. if we traverse from point A to
B, there is drop of potential; similarly from B to A, there is gain of potential.
(II) KIRCHHOFF`S SECOND LAW (LOOP LAW OR POTENTIAL LAW): Kirchhoff’s second
law states that the algebraic sum of potential differences along a closed loop in a circuit is zero.
In mathematical form, the loop theorem (Kirchhoff’s second law) can be expressed as S V = 0 .
In terms of potential drops and emfs, the law is expressed as S (iR)+ S E = 0
Sign conventions:
Any Potential Drop Encountered Is Taken To Be Positive And Any Potential Rise Is Taken To Be
Negative. R I B A R
A B
(a) The change in potential in traversing a > > > >
resistance in the direction of current is +IR while
in the opposite direction -IR as shown in the figure. VA - IR = VB VB + IR = VA
Or VB –VA = -IR VA - VB = IR VA - VB = IR
E E
(b) The change in potential in traversing an emf
A I I B A I I B
source from negative to positive is -E while in < < < <
the opposite direction + E irrespective of the
direction of current in the circuit as shown in the VA - E = VB VB + E = VA
figure. Or VB –VA = - E VA - VB = E VA - VB = E
Note:
1) Second law implies that electric energy given to the charge by a source of emf is lost in passing
through the resistance.
2) This law represents “conservation of energy” as if the sum of potential changes around a closed
loop is not zero, unlimited energy could be gained by repeatedly carrying a charge around a loop.

PROBLEM SOLVING TECHNIQUE : APPLYING KIRCHHOFF’S RULES


In this chapter we have seen how Kirchhoff’s rules can be used to analyze multiloop circuits. The
steps are summarized below:
a) Draw a circuit diagram, and label all the quantities, both know and unknown. The number of
unknown quantities is equal to the number of linearly
independent equations we must look for.
b) Assign a direction to the current in each branch of the circuit.
(If the actual direction is opposite to what you have assumed,
your result at the end will be a negative number)
c) Apply KCL to each junction. Often, it is found more
convenient to apply KCL while marking the current in the
branches. This reduces the number of unknowns to be solved. For example, if we have assumed
currents in the branches AO and OB as I1 and I 2 , we need not mark the current in branch OC as
I 3 . Instead we apply KCL and say that the current in branch OC is  I1  I 2  .

Dr.K.K.R Gowtham 28 Current Electricity


d) Mark the polarity of the voltage across each element. In a
resistor, the polarity depends upon the assumed direction of
current. The end into which the current enters I marked
positive.

However, the polarity of the voltage (or emf) across a battery E E


A B A B
does not depend upon assumed direction of current.
I I

In both cases, terminal A is positive, though in the first case current enters the end A, and in the
second case the current leaves the end A.
e) Apply the loop rule to the loops until the number of independent equation obtained is the same as
the number of unknowns. For example, if there are three unknowns, then we must write down
three linearly independent equations in order to have a unique solution.
The same equation is obtained whether the closed loop is traversed clockwise or
counterclockwise. Try to take as many equations as the number of variables.
Start from any point and mentally go around the loop in the designated direction ,to reach the
same point again.
f) If necessary, chose other loops and repeat above steps until you get as many independent
equations as the unknowns. The set of equations obtained will be
independent provided each new loop equation contains a voltage I 1 b  I1  I 2 
change not included in a previous equation. The easiest way to P
ensure the independency of equation is to select all meshes (just like I2 Q
the panes of a window) as the loops for writing KVL. You may have a c
to elect loops as per the number of variables to be calculated.
Another thing we must keep in mind while selecting loops in that we R S
I I  I I  I  I 
1 2
must select the oops independently. Suppose we require three loops 2
d
to be selected and we elected a bigger loop along with two loops
(these two loops collectively from the bigger loop) and write three h g
equations. We will notice that actually we may not be getting three E
equations but instead we shall be getting only to equations. So avoid
taking such type of loops and try to identify and take independent loops. Suppose, in this circuit,
we take the currents as shown. Hence we observe that at least three equations are required to get
I , I1 and I 2 .
Please be careful and take loops as abda, bcdb (or abcda) because abcda = abda+bcdb. So these
are just two loops. The third loop must be abcgha (or adcgha or adbcgha or abdcgha).
So, while taking the loops, keep in mind to take unique loops. Avoid taking big loops made from
smaller loops (or which equations have already been taken) because the big loop made from the
combination of small loops will again fetch you the identical equations which cannot be used as
such.
g) Solve the simultaneous equations to obtain the solutions for the unknowns.
h) If there are ‘n’ meshes in a circuit, the number of independent
equations in accordance with loop rule will be (n – 1). I 1 b  I1  I 2 
Please be careful and take loops as abda, bcdb (or abcda) because P I2 Q
abcda = abda+bcdb. So these are just two loops. The third loop a c
must be abcgha (or adcgha or adbcgha or abdcgha).
So, while taking the loops, keep in mind to take unique loops. Avoid
R S
taking big loops made from smaller loops (or which equations I I  I  I 
I  I2 1 2
have already been taken) because the big loop made from the d
combination of small loops will again fetch you the identical
equations which cannot be used as such. h g
E

Dr.K.K.R Gowtham 29 Current Electricity


g) Solve the simultaneous equations to obtain the solutions for the unknowns.
Ex-1Calculate the currents through each resistance in the given circuit. Also calculate the potential
difference between points a and b.
5 a 10
Sol : Let the current through R1 be x and through R3 be y, as shown.
Applying KCL at junction a, the current through R2 will be
6V 8V
 x  y . 4
According to the assumed direction of currents, we mark the polarity
of voltage drops across different resistances. b

Writing KVL for loop I , se get


5 x a 10  x  y 
 5 x  10  4 y  6  0 y R2
xR1  E3  yR3  E1  0 R1
 5 x  4 y  4 E3 10V
Writing KVL for lop II, we get 6V II E2 8V
E1 I R
 x  y  R2  E2  yR3  E3  0 3 4
  x  y 10  8  4 y 10  0
 10 x  14 y  2 b
Solving equations (1) and (2), we get
24 5 1
x A, y   A and x  y  A
25 11 55
The signs indicate that the directions of x and y were assumed incorrectly, whereas the assumed
direction of current x-y through R2 was correct.Potential difference between a and b,
Vab  Potential fo a with respect to b is Va  Vb . So, Va  Vb  Potential rise when we go from b to a
5 90
 Va  Vb  yR3  E3    4  10  V
11 11
4 2
Ex-2: Find currents in different branches of the electric A B C
circuit shown in figure. Also find the potential difference
between points F and C. 2V 4V 6V

Sol In this Illustration there are three wire segments EFAB, BE F E D


2 4
and BCDE and two junctions at B and E. Therefore we have
three unknowns I , I1 and I 2 and hence we
require three equations. One equation will be A
H L B I1 H L C
obtained by applying Kirchhoff’s Junction 4 I2 2
Law (either at B or at E) and the remaining H L H
2V 4V
two equations, we get from the Second Law L Ph L 6V
(Loop Law). Consider three loops ABEFA ,
F D
ACDFA and BCDEB . But we have to I 2 E I1 4
choose any two of them. Further, we can
choose any arbitrary directions of I , I1 and I 2 .
Applying Krichhoff’s First Law ( Junction Law) at junction B, I  I1  I 2 ….(1)
Applying Krichhoff’s Second Law in Loop ABEFA, 4I  4  2I  2  0 …(2)
 I1  6 A
Applying Kirchhoff’s Second Law in Loop BCDEB,

Dr.K.K.R Gowtham 30 Current Electricity


10 5 5
2I1  6  4I1  4  0  I1   A
Using (1) ,we get
I1   A
6 3 3
Here negative sign of I1 implies that current I1 is the direction opposite to that assumed by us in
the figure i.e., it is from C to B ( and not from B to C) To find the potential difference between any
two points of a circuit let us reach from one point to the other via any path of the circuit. It is
advisable to choose a path in which we come across the least number of resistors preferably a path
which has no resistance. Let us reach from F to C via A and B,
VF  2  4I  2I1  VC  VF  VC  4I  2I1  2
5 4
Substituting , I  1A and I1   A, we get VF  VC   V
3 3
Here negative sign implies that VF  VC or F is at a lower C
potential than C. I1 I3 I2

<

<

<
Ex3:Consider the network as shown in Fig. Current is
supplied to the network by two batteries as shown. Find 5 2 3
the values of currents I1, I2, I3. The direction of the
A + I2V +B
currents are as indicated by the arrows. – – 6
Sol :
Applying Kirchhoff`s 1st law to junction C, we get D
I1 + I2 – I3 = 0 .... (1)
Applying Kirchhoff`s IInd law to the closed meshes ACDA and BCDB, we get
– 5I1 – 2I3 + 12 = 0 5I1 + 2I3 = 12 ..... (2) –3I1 – 2I3 + 6 = 0
3I2 + 2I3 = 6 ..... (3) On solving;
I1 = 1.548 A. I2 = 0.58A and I3 = I1 + I2 9V 1
=
4 3V 4
2.3128 A. > <
i2
< >
A i1 i2 i1 B
>

Ex4:For the circuit shown in the figure, potential i1 + i2


difference between points A and B is 16V. Find the current passing
2
through 2 .
Sol : VA – VB = 16V \ 4i1 + 2(i1 + i2 )- 3 + 4i1 = 16V .......... (1)
Using Kirchhoff’s second law in the closed loop, we have 9 – i2 – 2(i1 + i2) = 0 .......... (2)
Solving eqs (1) and (2), we get i1 = 1.5 A and i2 = 2A
Current through resistor 2 = 2 + 1.5 = 3.5A
Ex5. Find the potential at point A
Sol. Let potential at A = x, applying Kirchhoff’s current law at
junction A

x  20  10 x  15  20 x  5  50 x  30
   0
1 2 2 1
2 x  60  x  35  x  45  2 x  60
 0
2
5
 6x + 10 = 0   x
3

Dr.K.K.R Gowtham 31 Current Electricity


5
Potential at A  V
3

Ex6. Find the current in every branch?

Solution
Let we assume x potential at the top junction & zero potential at lower junction as from KCL, net
current on a junction is O.
i1 + i2 + i3 =0
x  5 x  10 x  20
  0
2 2 2
35
3x = 35  3 x  35  x 
3
35
5
3 10
 i1   A
2 3
5 25
Similarly, i2  A ; i3  A
6 6
Ex7. Find the current in every branch?

Sol. Assume x potential at the upper junction & zero potential at the lower junction. By KCl, we
know that net current on a junction is zero.
 i1  i2  i3  i4  0
x4 x2 x2 x4
   0
2 4 4 2
2x  8  x  2  x  2  2x  8  0
8
6 x  16  0  x V
3
2 1 7 2
 i1  A, i2   A, i3  A, i4   A
3 6 6 3
Ex8. Find the current in every branch?

Sol. The above question could be solved by assuming potential x & y at the top junctions & zero
potential at lower junctions

Dr.K.K.R Gowtham 32 Current Electricity


At the junction 1 applying KCL,
i1  i2  i3  0
x4 x2 x y
  0
2 2 2
 3x  y  6 …(1)
At the junction 2 applying KCL,
i4  i5  i6  0
yx y2 y
  0
2 2 2
 3y – x = 2
Solving (1) & (2) 9 x  3 y  18  3 y  x  2
5 3
 8x = 20 x  ,y
2 2
Just put the values of x & y & then the evaluate the current in every branch
Ex9. Find current in the circuit

Sol.  all the elements are connected in series


 current in all of them will be same
let current = i

Applying kirchhoff voltage law in ABCDA loop

3i-30+2i+15+i -20+4i+10 = 0
10 i = 25  i = 2.5 A
Ex10: Find the current in each wire applying only Kirchhoff’s
voltage law

Sol. Applying kirchhoff voltage law in loop ABEFA


i1  30  2  i1  i2  10  0
3i1  2i2  20  0 …(i)
Applying kirchhoff voltage law in BCDEB
30  2  i1  i2   50  2i2  0
4i2  2i1  80  0
2i2  i1  40  0 …(ii)
Solving (i) and (ii)
3 40  2i2   2i2  20  0

Dr.K.K.R Gowtham 33 Current Electricity


120  4i2  20  0
i2 = - 25A and i1 = 10A  i1 + i2 = 15A
Current in wire AF = 10A from A to E
Current in wire EB = 15A from B to E
Current in wire DE = 25 A from D to C
COMBINATIONS OF RESISTORS:
1. SERIES COMBINATION
Consider a number of resistors connected in series by joining them end to end such that same
current passes through all the resistors. In Fig. three resistors of resistances R1, R2 and R3 are
shown connected in series. The combination is connected to a battery at ends A and D. Let a current
I flow through the series combination when it is connected to a battery of voltage V. Potential
difference V1, V2 and V3 are developed across R1, R2 and R3 respectively due to this current I
then
A B C D
V1 = IR1, V2 = IR2 and V3 = IR3. But sum of V1, V2 and V3
R1 R2 R3
is equal to V i.e. I I

>
>
V = V1+ V2 + V3 V = IR1 + IR2 + IR3
+ –
V
Series combination of resistance
If equivalent resistance of this series combination is R, then
V = IR = I(R1 + R2 + R3) or R = R1 + R2 + R3
This arrangement may be extended for any number of resistors. R = R1 + R2 + R3 + R4 +.....
N
Raq  R1  R2  ....   Ri
i 1

Thus, equivalent resistance of a series combination of resistors is equal to sum of resistances of all
resistors.
Hence, the equivalent resistance of series combination is greater than the greatest
resistance in the combination.
In Series combination of resistors ,
1) Since same current flows in all resistors
I total = I1  I 2  I3  ...........
2) The total pd is V total = V1  V2  V3  ...........
3) From ohms law V = IR and V µ R
V1 : V2 : V3 :...........  R1 : R2 : R3 :...........
4) When two resistors R1 and R2 are connected in series then
R1 R2
>

é R ù é R ù I
V1  Vtotal ê 1 ú ; V2  Vtotal ê 2 ú
êR1  R2 ú êR1  R2 ú
ë û ë û
V V
I=
R1 + R2
5) ‘n’ identical resistors each of resistance R are connected in series, then effective resistance of
the combination is nR.

Dr.K.K.R Gowtham 34 Current Electricity


1 1 1 1
6) Net conductance (G) is   
Gs G1 G2 G3
VOLTAGE DIVIDER CIRCUIT
Consider a voltage source Vin that is connected in series to two
resistors, R1 and R2 . The voltage difference, Vout across resistor R2 will R1
be less than Vin . This unit is called a voltage divider. From the loop rule, Vin
Vout
Vin  IR1  IR2  0 R2
Vin
So the current in the circuit is given by I 
R1  R2 Voltage divider

Thus the voltage difference, Vout , across resistor R2 is given by


R2
Vout  IR2  Vin
R1  R2
Note that the ratio of the voltages characterizes the voltage divider and is determined by the resistors
Vout R2

Vin R1  R2
2. PARALLEL COMBINATION
I1 R1
Connect the resistors in parallel by joining their one end at one point and > >
other end at another point. In parallel combination same potential I2 R2
A >
difference exists across all resistors.
Fig. shows the parallel combination of three resistors of resistances I3 R3
> >
R1, R2 and R3. Let the combination be connected to a battery of I I
>

>
voltage V and draw a current I from the source. V
+ –
Parallel Combination of Resistance
The main current divides into three parts. Let I1, I2, I3 be the currents flowing through resistors
R1, R2, R3 respectively, then I1 = V/R1, I2 = V/R2 and I3 = V/R3.
The main current is the sum of I1, I2 and I3
V V V
i.e. I = I1+ I2+ I3 or I =  
R1 R2 R 3
If the equivalent resistance of combination is R, then V = IR or I = V/R
V V V V ; 1  1  1 + 1
R R1 R2 R3 R R 1 R 2 R 3
The process may be extended for any number of resistors so that,
N
1 1 1 1 1 1 1 1 1 1
     ........     .....  
R R 1 R2 R 3 R 4 Req R1 R2 R3 i 1 Ri

From this we infer that reciprocal of equivalent of resistance of parallel combination is equal to sum
of reciprocals of individual resistances.
For two resistors in parallel
1  1  1 or R  R1R2
R R1 R2 R1  R2

Dr.K.K.R Gowtham 35 Current Electricity


Hence, the equivalent resistance of parallel combination will be less than the least resistance value
in the combination.
In Parallel combination of resistors
1. V total = V1  V2  V3  ...........

2. I total = I1  I 2  I3  ...........
1 1 1
3. I1 : I 2 : I 3 :...........  : : :...........
R 1 R2 R3
4. ‘n’ identical resistors each of resistance R are connected in parallel, then effective resistance of the
R
combination is .
n

5. Effective conductance G p is the sum of conductance of individual resistors i.e., G p  G1  G2  G3

6. When one resistance R1 is much smaller than the other resistances, then the R1

equivalent resistance Req is approximately equal to the smallest resistor R1 . In the


case of two resistors,
R2

R1 R2 RR
Req   1 2  R1
R1  R2 R2 E

This means that almost all of the current that enters the node point will pass through the branch
containing the smallest resistance. So, when a short develops across a circuit, all of the current
passes through this path of nearly zero resistance. Hence SHORT CIRCUITING a branch implies
taking the resistance of that branch to be zero or close to zero.

Shown here is an electric circuit whose two points are directly connected by a conducting wire. Here
we say that the resistance R1 in the circuit is short circuited. Under such condition both points are at
same potential and hence the potential difference across R1 is zero. Hence , no current will flow
E
through R1 and the current through R2 is I 
R2

CURRENT DIVIDER CIRCUIT:


DIVISION OF CURRENT IN RESISTORS CONNECTED IN PARALLEL:
Let the two resistors of resistances R1 and R2 be connected in
parallel between points A and B I1 R1
(Fig. ). The main current I be divided into two parts >
I1 and I2 flowing through R1 and R2 respectively. I A B I
> >
The main current I is sum of I1 and I2 I2 R2
i.e I1 + I2 = I. ............. (1) >

Division of current in resistance


Potential difference across R1 and R2 is same

Dr.K.K.R Gowtham 36 Current Electricity


R2 R1
Therefore I1R1 = I2R2...... (2) from (1) and (2) I1  I , Similarly, I2  I .
R1  R2 R1  R2

The division of current in the branches of a parallel circuit is inversely


proportional to their resistances. I1 R1

I  I1  I 2  I 3 I I2 R2 I

I1R1  I 2 R2  I 3 R3
I3 R3
R1 R2 R3
And Req 
R1 R2  R2 R3  R3 R1

 1   1 
     
R2 R3 R1  I   R1  I
I1  I   1
 R1 R2  R2 R3  R3 R1    1  1   1 
R R R  R 
 1 2 3   p

 1   1 
     
R1 R3 R2  I   R2  I
I2  I   1
 R1 R2  R2 R3  R3 R1    1  1   1 
R R R  R 
 1 2 3   p

 1   1 
     
R1 R2 R3  I   R3  I
I3  I   1
 R1 R2  R2 R3  R3 R1    1  1   1 
R R R  R 
 1 2 3   p

It is easy to remember the expressions for I1 , I 2 and I 3 just by noticing which resistance is missing
in the term at the numerator. I =10 P
> I2
Ex1:For a circuit shown in Fig find the value of resistance R 2 and current I
>
>

1
+

I2 flowing through R2 – 50V R 10


1
R2

Sol : If equivalent resistance of parallel combination of R 1 and R2 is R, then Q


R1 R2 10 R2 V
R  According to Ohm’s law, R = .
R1  R2 10  R2 I
50 10 R2
R  5   5  R2  10
10 10  R2
The current is equally divided into R1 and R2. Hence I2 = 5A.

Dr.K.K.R Gowtham 37 Current Electricity


Ex2: 7
In the given network, determine the current through
6 resistor.
60V 12 6 12

Solution
By using Kirchhoff’s Laws, we can determine not only the current through 6 resistor, but also the
currents through other resistors. But for this we will have to solve three simultaneous equations,
which is quite a difficult task.

However, if our aim is to determine only the current through 6 resistor, we can do it easily by
using current divider concept.

7 7 I I1 7 I

60V 6 12 12 60V 6 6 60V 3

(A) (B) (C)


We first calculate the current I supplied by the battery. For this we rearrange the network and
simplify it step by step as follows:
a) We interchange the position of 6 and 12 resistors. This will not make any difference, as
the two resistors are in parallel [Fig.(A)]
b) We then combine the two 12 resistors to give a single resistor of 6 [Fig.(B)]
c) We again combine the two 6 resistors to give a single resistor of 3 [Fig.(C)]
d) The current I is clearly given as
60
I  6A
73
e) Using Fig(B), and the current divider concept, we get the current through 6 resistor as
 6 
Ix  I    3A
66

Ex3.Find current which is passing through battery.


30
Sol. i  15 A
2

Req=1 s (2p3p6)

Req = 1 + 1 = 2 

Ex4.Find equivalent Resistance

Dr.K.K.R Gowtham 38 Current Electricity


Here all the resistance are connected between the terminals A and B. so, modified circuit is

R
So Req 
3

Ex5. Find the current in Resistance P if voltage supply between A and B is V volts
3R
Req 
5

5V
I Modified circuit
3R

5V
R
Current in P  3R  2V
1.5R  R 3R

Ex6. Find the current in 2  resistance.

Sol. 2, 1 in series = 3


18
3, 6 in parallel =  2
9
2, 4 in series = 6

Dr.K.K.R Gowtham 39 Current Electricity


6, 3 is parallel = 2
Req = 4 + 4 + 2 = 10
120
i  12 A
10

8
So current in 2 Resistance = A
3
4 8
2 A A
3 3

Ex7. In the circuit shown in figure(a).


Find the current flowing through the 100  resistor connecting points U and S.

Sol. Figure (b) shows simplified circuit. The battery is directly attached to resistor 90  hence current in
it is 2 A, see figure (c), The total resistance of second branch is also 90  , hence current divides equally.
Now current through 45  resistor is 2 A and it is a combination of two equal 90  resistors. Once
again current divides equally. 90  resistor is a series combination of 40  and 50  , hence current
through them is equal, i.e., 1 A. As 50  resistor is a parallel combination of two equal 100  resistors,
they must have the same current i.e., 0.5 A
Note :
1. If ‘n’ wires each of resistance ‘R’ are connected to form a closed polygon, equivalent resistance
 n 1 
across any two adjacent corners is Reff   R
 n 
2. If a wire of resistance R is bent in the form of regular polygon of n sides, effective resistance between
 n 1 
any two adjacent corners is Reff   2 
R
 n 
3. If a uniform wire of resistance R is, stretched to m time its initial length and bent into a regular
polygon of n sides
a) Resistance of the wire after stretching is R1  m 2 R ( R  l 2 )
m2 R
b) Resistance of each side R2 
n

Dr.K.K.R Gowtham 40 Current Electricity


n 
 2 R2  m2 R
c) Resistance across diagonally opposite points (if n is even) R0     R0 
 2  4
 
(n  1) (n  1)m R
2
d) Resistance across one side R3  R2 
n n2
4. A wire of resistance R is cut into n identical pieces and those are connected in parallel. Then
R
equivalent resistance is
n2
5. If equivalent resistance of R1 and R2 in series and parallel be Rs and Rp respectively then
1 1
R1  Rs  Rs2  4 Rs R p  and R2   Rs  Rs2  4 Rs R p  .
2  2 
6. If n identical resistances are first connected in series and then in parallel, the ratio of the equivalent
Rp n2
resistance is given by  .
Rs 1
7.Using n conductors of equal resistance, the number of possible combinations is 2 n – 1.
8. If the resistance of n conductors are totally different, then the number of possible combinations will be 2n.

Ex8:P and Q are two points on a uniform ring of resistance R. The equivalent resistance between P and Q
is
P S
P Q

O O
Q
T
R R
Sol : Resistance of section PSQ R1  . r 
2r 2
Rr  2  
Resistance of section PTQ R2 
2r
R  2  
R2 
2
As R1 and R2 are in parallel
R1 R2 R  2    
So, Req  
R1  R2 42

Ex9: Find the equivalent resistance across AB of the circuit given.


5 C 5

25 20
10 5

AB
Solution
The circuit is redrawn to make it easily evaluable.
The left block is equivalent to 15 and 25 in parallel, and its resistance is given as

Dr.K.K.R Gowtham 41 Current Electricity


25 15
R1   9.375
25  15
The right block is equivalent to 10 and 20 in parallel, and its resistance is given as
5 5
C
10 5

A 25 20 B
10  20 200
R2    6.667
10  20 30
9.375 6.667
A R1 C B
R2
The circuit now reduces to two resistors in series, as shown.
Hence , the total resistance , R  R1  R2  9.375  6.667  16.042
Ex10: Two wires of equal diameters of resistivities 1 and 2 and length l1 and l2 respectively are
joined in series. Find the equivalent resistivity of the combination.
1 1 
Sol :Resistance, R1  ; R2  2 2
A1 A2
As the wires are of equal diameters A1  A2  A .
l1 l x
R1  , R2  2 ; R 
A A A
where l = l1 + l2; R = R1 + R2
l 1l1 2l2
  ; l  1l1  2l2
A A A
(l1  l2 )  1l1  2l2 [ l  l1  l2 ]
l1 l
+ 2
 l  l 1 s1 s 2 (l + l )s s
  1 1 2 2 also = or s = 1 2 1 2
l1  l2 s l1 + l2 l1s 2 + l2s 1
1  2 2s 1s 2
If l1= l2 ,then   and s =
2 s1 + s 2
Ex11: Two wires of equal lengths of resistivities 1 and 2 and area A1 and A2 respectively are joined
in parallel. Find the equivalent resistivity of the combination.

1 1 
Resistance, R1  ; R2  2 2
A1 A2

As the wires are of equal lengths l1  l2  l .


l l l l
R1  , R2  ; R 
A1 A2 A A1  A2

Dr.K.K.R Gowtham 42 Current Electricity


R1 R2
where A= A1 + A2; R 
R1  R2

(A1 + A2 )r 1r 2 A11  A22


r= ALSO 
A1r 2 + A2r 1 A1  A2

212 s + s2
If A1=A2,then   and s = 1
1  2 2
Ex.12: Two resistors with temperature coefficients of resistance 1 and  2 have resistances R01
and R02 at 0 0C . Find the temperature coefficient of the compound resistor consisting of
the two resistors connected, (a) in series (b) in parallel.
Solution
In series:

At 0 0C R01 R02 R0  R01  R02


At t 0C R01 1  1t  R02 1   2t  R0 1   t 
R01 1  1t   R02 1  2t   R0 1  t 
 R01 1  1t   R02 1  2t    R01  R02 1  t 
 R01  R011t  R02  R022t  R01  R02   R01  R02 t
R011  R02 2
 
R01  R02
Note: a conductor wire and a semiconductor wire are connected in series, if the resistance of the
combination at all temperatures remains the same, then
R011  R02 2
  0  R011  R02 2  0  R011  R02 2 (Numerically, as  2 is negative)
R01  R02
Two resistors having resistances R1 and R2 at 00 C are connected in series. What is the condition for the
effective resistance in series is same at all temperatures.
R1 + R2 = R1¢+ R2¢
R1 + R2 = R1 (1+ a1t )+ R2 (1+ a 2t )
R1a 1 = - R2a 2
In parallel
R01 R01 R02
R
R01  R02

R02
1 1 1
o
At t C ,  
R0 1   t  R01 1  1t  R02 1   2t 
R01  R02 1 1
  
R01R02 1   t  R01 1  1t  R02 1   2t 

Dr.K.K.R Gowtham 43 Current Electricity


Using the Binomial expansion, we have
1 1 1 1
1   t   1   t   1  1t   1   2t 
R02 R01 R01 R02
 1 1  1 
 t    t 2 t
 R01 R02  R01 R02
 R   2 R01
   1 02
R01  R02

Ex13:

Consider a material of resistivity  in a shape of a truncated cone of altitude h, and radii a


and b, for the right and the left ends, respectively, as shown in figure.

b a

A Truncated Cone

Assuming that the current is distributed uniformly throughout the cross-section of the cone,
what is the resistance between the two ends?

Solution

Consider a thin disk of radius r at a distance x from the left end. From the figure shown, we have

br ba x
  r  a  b  b
x h h

b
r a 
x

l
Since resistance R is related to resistivity  by R  , where l is the length of the conductor
A
and A is the cross section, the contribution to the resistance from the disk having a thickness dy is

 dx  dx h
dR   
 r2 b  a  b x 
2
 ab
 
 h 

Straight forward integration then yields

Dr.K.K.R Gowtham 44 Current Electricity


h
 dx h
R 
 b  a  b x 
2
 ab

0

 h 

h
 R
 ab
where we have used

du 1
  u    2

  u   

 h l
Note that if b  a, we get R  
 a2 A
Ex14:

Consider a hollow cylinder of length L and inner radius a and outer radius b, as shown in
figure. The material has resistivity  .

a) Suppose a potential difference is applied between the ends of the cylinder and produces a
current flowing parallel to the axis. What is the resistance measured?

b) If instead the potential difference is applied between the inner and outer surfaces so that
current flows radially outward, what is the resistance measured?

a 

b
A hollow cylinder

Solution

a) When a potential difference is applied between the ends of the cylinder, current flows parallel to
 
the axis. In this case, the cross-sectional area is A   b 2  a 2 and the resistance is given by

L L
R 
A   b2  a 2 

b) Consider a differential element which is made up of a thin cylinder of inner radius r and outer
radius r  dr and length L. Its contribution to the resistance of the system is given by

Dr.K.K.R Gowtham 45 Current Electricity


 dl  dr
dR  
A 2 rL

where A  2 rL is the area normal to the direction of current flow. The total resistance of the
system becomes
b
 dr  b
R  log e  
a
2 rL 2 L a
INFINITE LADDER AND GRID:
Some networks make a ladder (or a grid) and extend to infinity. To reduce such networks we use
the following steps.
Step 1 : Let us assume the total resistance of the infinite network to be X (say)
Step 2: Now just retain one basic repetitive unit and we observe the remaining circuit to be the same as
the original circuit. So, resistance of this left out circuit must be X.
Step 3: Now the equivalent circuit, is the combination of basic unit and original repetitive circuit of
resistance X, such that the net resistance of the entire circuit is X.
The following illustrations are done on the basis of these three steps.
Ex1: a) Find effective resistance between points A and B of an infinite chain of resistors joined as
shown in Fig.(A)
b) For what value of R0 in the circuit shown in Fig.(B) will the net effective resistance is independent
of the number of cells in the chain?
R1 R1 R1 R1
R1 C R1 C
A
R2 R2 R2 To infinity R2 R2 R0
B
D D
Solution
a) Suppose the effective resistance between A and B is X .Applying the steps discussed, we get
R1 C R1 R1 R1
C A C

R2 R2 To infinity x R2 x

D D
D B
R2 X
 X  R1   R2 || X   R1 
 R2  X 
 X 2  R1 X  R1 R2  0

 X
1
2 
R1  R 1
2
 4 R1 R2  
And as resistance cannot be negative , we have
R1  4 R2 
X 1  1  
2 R1 
However, if R1  R2  R we get

X
R
2

1  5  1.6 R 
Dr.K.K.R Gowtham 46 Current Electricity
b) Suppose there are n sections between points A and B and the network is terminated by R0 with
equivalent resistance X. Now , if we add one more to the network between the C and D , the equivalent
resistance of the network X will be independent of number of cells if the resistance between points C and
D still remains R0 (or X) . So, the circuit reduces to an equivalent circuit as shown.
R1 R1 R1 C R1
C
R2 R2 R0 R2 R0 = X

D D
R2 R0
 R1  R
 R2  R0  0
1 4R 
 X  R0  1  1  2 
2 R1 
If R1  R2  R then we get
,
X  R0 
R
2
 
1  5  1.6 R

Ex2:Find the equivalent resistance between A & B ?

Sol. As moving from one section to next one, resistance is increasing by k times. Since the network is
infinitely long, removal of one section from the chain will bring a little
change in the network.
The effective resistance between points C & D would be kR E (where
RE is the effective resistance)
Effective R between A & B.
R  KRE 
RE  R 
KRE  R
On solving we get

2kR  R   R  2kR 2  4kR 2


RE 
2k
Ex3:The circuit diagram shown consists of a large number of elements ( each element has two resistors
1
R1 and R2 ) . The resistances of the resistors in each subsequent element differ by a factor of k 
2
from the resistances of the resistors in the previous elements.
Find the equivalent resistance between A and B shown in figure.

Dr.K.K.R Gowtham 47 Current Electricity


R1 kR1 k2R1 k3R1 k4R1
A
R2 kR2 k2R2 k3R2 k4R4 
B
Solution
When each element of circuit is multiplied by a factor k then equivalent resistance also becomes
k times. Let the equivalent resistance between A and B be X.
R1 kR1 k2R1 kR1 k2R1 k3R1
A A A

R2 kR2 k2R2 kR2 k2R2 k3R2

B B B

X kX

So, equivalent circuit becomes


A
R1
x R2 kx

For
1
k 
 R  R2  
X 1
R12  R22  6 R1R2
2 2
SHORT AND OPEN CIRCUITS:
When two points of a circuit are connected together by a conducting wire, they are said to be short
circuited. The connecting wire is assumed to have zero resistance. No voltage can exist across a short,
secondly current through it is very large (theoretically infinity)

Remaing Short
circuit circuit I scVAB  0
I sc  I AB  
Two points are said to be open circuited when there is not direct connection between them, a break in the
continuity of circuit exists. Due to this break the resistance between the two points is infinite and there is
no flow of current between the two points.

Remaing Short
circuit circuit
I AB  0
R AB  
Please not that the entire applied voltage is felt across the open, i.e., across terminals A and B, so,
VAB  100V .

Dr.K.K.R Gowtham 48 Current Electricity


R1
A
Open
100V=V circuit V 100V
B VAB=100V
R2
In the circuit shown if B and D are shorted, then
Short
circuit

R1 R2 R3
A D
B C

V
V V
Req  R1  I  and VB  VD
Req R1
Bulb B1 acts as open circuit, bulb B1 will not glow. However, other bulb B2 remains connected across
the voltage supply, it will operate normally.
B1 B2
220V
open filament

Short circuit across R3 shorts R1 and R2 as well , short across one branch in parallel means short across
all branches . There is no current in shorted resistors. The shorted components are not damaged, they will
function normally when short circuit is removed.
A

V R1 R2 R3 Short
circuit
B
R4 B
d) In figure shown, short circuit across R3 may
short out R2 but not R1 , because it is protected by R4 .
R1 R2 R3 Short
circuit
C
PRINCIPLE OF SYMMETRY:
If a network is symmetrical on both sides of a line, all the points lying on the line will have same
potential. Hence, no current will flow in a resistance connected between two such points. Such
resistances can be ignored while calculating the equivalent resistance. This simplifies the network. Or,
else all the points on this line may be treated as shorted.
SYMMETRICAL CIRCUITS
I Method: When a circuit is symmetrical about a line ( By symmetry we mean that two parts are mirror
images of each others), then the potential and current must also be symmetrical . Therefore, currents in

Dr.K.K.R Gowtham 49 Current Electricity


ab and ad are same. Currents in dc same. Potential of the points b, e and d are same. The equivalent
2
circuit is redrawn, the equivalent resistance is R. Note that there is no current in branches be and ef .
3
b
R R
R
R b R
A a c B
R R R e R
R A B
R R R f R
d
(b)
(a)
Another symmetry is visible along line bd . The current flow is not a mirror image in branches ab and
bc because the flow is in same direction. This is called asymmetric condition. The special thing about
this asymmetry is that current incoming at b is equal to outgoing current, similar situation exists at b and
d also. Thus resistors in branches be and de are ineffective.
II Method: FOLDING SYMMETRY
The potential difference in R between (B, C) and between (B, D) is same VC = VD
Hence the point C and D are same hence circuit can be simplified as

This called folding.


Now , it is Balanced Wheatstone bridge

Note:In II Method it is not necessary to know the currents in CA and DA


Ex1:In figure there is asymmetry along the xy . The current reaching O  I 2  is equal to outgoing current
that means there is no mingling of current from upper branch and lower branch into middle branch.
X
R

R R R 2r
R I I2 I1
I1 I R
2 O I2 I1 I 1 I2
A B
I3 R I3 I3 I3
R R
R R

R
Y (b)
(a)

Dr.K.K.R Gowtham 50 Current Electricity


4R
The resulting circuit is simple enough, the equivalent resistance is .
5 12V A
F
Ex2:
Find the current in the branch AB of the circuit. 2 6
Also find the current in the branches BF and EA. 4

C G D
4 6 2

B 12V E
Solution
Referring to figure we see that symmetry demands that current only circulates in outer branch.
Points A and B are at the same potential because the circuit is symmetrical. Therefore, no current
can go across the resistors in that branch. The current through BF and EA is 2A. R
A B
Ex3: R R
Calculate effective resistance between points
A and C for the networks shown. R R
O
R
R
D C
Solution R
The two sides of the line BOD , are symmetrical . Thus , if the R
points A and C are connected to a source of emf, points B, O A B
and D of the network will have same potential . Therefore, no
R
current flows through RBO and RDO . These resistances can be
ignored. The network then reduces to that show in figure.
R R
O
Thus, between points A and C, we now have three branches in R
parallel, each having resistance of 2R.
2R D C
 Req 
3 R
Alternately, since the points B,O and D are at same potential , we can
treat the shorted.
R
A B
R
R R R
O R
R
R BO R
D A C
R C R D R
R R 2R
 Req    B
3 3 3

Ex4: Calculate the effective resistance R R R


between points A and C, by applying symmetry principle. R R
D
A C
R

Dr.K.K.R Gowtham 51 Current Electricity


Solution
R
Break the branch AC into two resistors in series, each , and consider the dotted line passing
2
through B,D and E. The network on the two sides of this line is symmetrical. Hence, one can short-circuit
the points B,D and E and calculate Req .   AE

R R R R
R
RD R
R R
A E C B D
R/2 R/2 A C
R/2 E R/2
R
Now, we find that resistances R,R and are in parallel across A and E. The parallel combination of R
2
R R R R
and R gives . This in parallel with given . Hence ,
2 2 2 4
 Req  AE  R4   Req  AC  2  Req  AE  2  R4   R2
INDENTICAL POTENTIAL POINTS
In some networks, you may not find symmetry on the two sides of a line. But, you may find that the
network contains some set of points having identical potential . Such a set of points can be joined
together to make the network simple.
D C
Ex1:
Twelve equal wires of resistances R each are joined up to form the A
edges of a cube, as shown in figure. The cube is connected into a B
circuit across the diagonal AG. Find the equivalent resistance of
the network.
H G
Solution
Let us search the points of identical potential. Since the three E F
edges of the cube from A, viz., AB,AD and AE are identical in all respects in the circuit, the points B,D
and E are at the same potential. Similarly, for the point G, the sides GC, GH and GF are symmetrical and
the points C,H and F are at the same potential.
Next, to simplify the circuit, we bring together the points, B,D and E and also C,H and F.
Now, it becomes obvious that
A D C G
B H
E F
the resistance between A and D =R/3
the resistance between D and C=R/6
the resistance between C and G=R/3
Thus, the circuit is equivalent to three resistances of value. R/3,R/6 and R/3, in series, and hence the net
resistance between A and G is
R R R 5R
AAG    
3 6 3 6

Dr.K.K.R Gowtham 52 Current Electricity


Ex2:
Consider the circuit shown in figure. For a given
R1 R1
resistance R0 , what must be the value of R1 so that
the equivalent resistance between the terminals is equal to R0 ? R1
Solution
The equivalent resistance, R ' , due to the three resistors on the right is
1 1 1 R0  2 R1 R  R  R1 
    R'  1 0
R ' R1 R0  R1 R1  R0  R1  R0  2 R1
Since R ' is in series with the fourth resistor R1 , the equivalent resistance of the entire configuration
becomes
R1  R0  R1  3R12  2 R1R0
Req  R1  
R0  2 R1 R0  2 R1
R0
If Req  R0 , then R0  R0  2R1   3R12  2R1R0  R02  3R12  R1 
3
DELTA TO STAR OR DELTA –STAR TRANSFORMATION
Suppose we are given three resistances R12 , R23 and R13 connected in delta fashion between
terminals 1,2 and 3 as in figure(a). So far as the respective terminals are concerned , these three
given resistances can be replaced by the three resistances R1 , R2 and R3 connected in star as shown
in figure(b).
These two arrangements will be electrically
1
equivalent because resistance as measured
between any pair of terminals is the same in both R1
the arrangements. 1

R12 R13 R13 R12 R3 R2


R1 
R12  R23  R13 R23 2
3 2 3
R23 R12
R2 
R12  R23  R13 (a) (b)
R13 R23
And R3 
R12  R23  R13
HOW TO REMEMBER ? R1
Resistance of each arm of the star is given by the product 1
of the resistances of the two delta sides the meet at it s end R13
divided by the sum of the three delta resistances. R12
3 2
R23 R
R3 2
Ex1:
Find the input resistance of the circuit between the points A and B of figure.

Dr.K.K.R Gowtham 53 Current Electricity


4 C
A
4 8
6
D E

8 4
B
Solution
For finding RAB , we will convert the delta CDE of figure into its equivalent star as shown in
figure.

4 16 6 24
RCS  8   ; RES  8   
18 9 18 9
4
A C
16 / 9
4
A C
S
12 / 9 24 / 9 16/9

D E
35/9
8 4

B B
35
The two parallel resistances between S and B can be reduced to a single resistance of 
9
As seen from figure
 16   35  87
RAB  4        
9  9  9
STAR TO DELTA OR STAR –DELTA TRANSFORMATION
Suppose we are given three resistances R1 , R2 and R3 connected in star fashion between terminals 1,2
and 3 as shown in figure(a). So far as the respective terminals are concerned, these three resistances can
now be replaced by the three resistances R12 , R23 and R13 connected in the delta network as shown in
figure(b).
These two arrangements will be electrically equivalent because resistance as measured between
any pair of terminals is the same in both the
1
arrangements.
R1
R1 R2 1
R12  R1  R2  N
R3 R3 R2 R13 R12
RR
R23  R2  R3  2 3 and 2 R23
R1 3 2
3
RR
R13  R1  R3  1 3 (b)
R2 (a)

Dr.K.K.R Gowtham 54 Current Electricity


HOW TO REMEMBER R1
The equivalent delta resistance between any two terminals 1
R13
is given by the sum of star resistances between R12
3 2
R23 R
R3 2
Those terminals plus the product of these two star resistances divided by the third star resistance.
Ex2: A
A network of resistances is formed as follows:
AB  9; BC  1; CA  1.5 forming a delta and
AD  6; BD  4 and CD  3 forming a star. Compute the
6

9 1.5
network resistance measured between
a) A and B
b) B and C, and 4 D 3
c) C and A.
B C
Solution 1
The star of figure(a) may be converted (a)
into the equivalent delta and combined in A
parallel with the given delta ABC. The A
three equivalent delta resistances of the
given star become as shown in figure. 9 1.5 6 27 / 20
When combined together, the final circuit 18 13.5

it as shown in figure(c).
a) As seen ,there are two parallel paths 9
across points A and B. C B C
B 9 /10
1
i) First, directly from A to B having a
resistance of 6 , and (b) (c)
ii) the other via C having a total
resistance

 27 9  6  2.25 18
     2.25  RAB   
 20 10   6  2.25 11
9  27  27  9
6   6  
10  20  441 20  10  621
b) RBC    c) RCA   
9 27  550 9 27  550
 6   6 
 10 10   10 20 
THE PRINCIPLE OF SUPERPOSITION OR SUPERPOSITION THEOREM
Complex network problems can sometimes be solved easily by using
the principle of superposition. This principle essentially states that 21.6V 8
when a number of emf’s act in a network, the solution is the same as
the superposition of the solutions for one emf acting at a time , the
others being shorted. 24 16

4 28.8V
Dr.K.K.R Gowtham 55 Current Electricity
Figure shows a network with two loops. The currents in various branches can be calculated using
Kirchhoff’s Laws. We can get the same solution by considering only one battery at a time and then
superposing the two solutions. If a battery has an internal resistance, it must be left in place when the emf
of the battery is removed. Figure shows how the superposition principle can be applied to the present
problem. The current values in figure(a) and (b) are easily verified. For example when the 21.6V battery
24 16
alone is acting , the total resistance in the circuit is, 8   4  21.6
24  16
21.6V 8 8 1.2A 21.6V 8 1.8A
1A 0.4A
1A 24 16 24 16  24 16
0.8A 1.8A
0.4A 0.6A
4 4 4
28.8V 28.8V
(a) (b) (c)
21.6V
This makes the total current  1A. This current splits between 16 and 24 in the ratio 3:2.
21.6
24 12
Similarly, the total resistance when only the 28.8V battery is acting is, 16   24
24  12
28.8V
Therefore, the total current is  1.2 A
24
The superposition principle shows that there is no current in the 24 resistance. Only a current of 1.8A
flows through the outer loop. All these conclusions can also be verified by analyzing the circuit using
Kirchhoff’s Laws.

PROBLEMS SOLVING TECHNIQUES


a) If there are a number of emfs acting simultaneously in any linear network, then each emf acts
independent of the others, i.e., as if the other emfs did not exist. The value of current in any
conductor is the algebraic sum of the voltages which each emf would have produced while acting
singly .
b) This theorem is applicable only to linear networks where current is linearly related to voltage as
per Ohm’s law.
c) Please note that while applying the superposition theorem only the current sources (or batteries)
and the currents have to be superimposed. Do not superimpose the resistances, because then you
will not be getting accurate results.

Electrochemical Cell
An electrochemical cell is a device which, by
converting chemical energy into electrical energy,
maintains the flow of charge in a circuit. It consists of
two electrodes of different charge in a circuit. It
consists of two electrodes of different material and an
electrolyte. The electrode at higher potential is called
positive terminal or anode while at lower potential
negative terminal or cathode. When a cell is in use, i.e., discharging, current outside the cell is from
anode to cathode while inside it from cathode to anode as shown in fig.

Dr.K.K.R Gowtham 56 Current Electricity


(a) Electro Motive Force (EMF) E
The emf of a cell is defined as the work done by the cell in moving unit positive charge in the whole
circuit including the cell once. So if W is the work done by a cell in moving a charge q once round
a circuit including the cell
Emf E = (W/q)
SI unit of emf is
(joule/coulomb) and is called
volt. The emf of a cell
depends only on the nature of
electrodes and electrolyte and
is constant for a given type of
cell, e.g., emf of a ‘dry cell’ is 1.5 V whatever be its shape and size.
The emf of a cell in a circuit is taken to be positive if circuit current inside cell, is from negative to
positive terminal (i.e., cell is discharging otherwise negative as shown in fig.
Note : (i) The term electromotive force is misleading; it was introduced by Volta who thought it
to be a ‘force’ that causes the current to flow. Actually emf is not a force but work required to
lift unit charge from lower potential to higher potential inside the cell, as outside the cell, charge
moves by itself losing energy.
(ii) According to modern concepts, a source of emf (called seat of emf) converts some form of
energy such as chemical (cell), thermal (thermocouple), radiant (photo-cell) or mechanical
(generator) into electrical (potential) energy.
DISTINCTION BETWEEN E.M.F. OF A CELL AND POTENTIAL DIFFERENCE
emf of a cell Potential difference
1. The emf is the maximum
potential difference between 1. It is the difference of potentials
the electrodes of a cell, between two points in a closed
when the cell is in its open circuit.
circuit

2. The emf is independent of 2. The potential difference


the resistance of circuit. It depends upon the resistance
depends on the nature of the between the two points of the
electrodes and the nature of circuit and current flowing
the electrolyte of the cell. through the circuit.

3. It is used as a source of 3. It is measured between any


energy. two points of the electric circuit.

4. Potential difference is an
4. emf is a cause.
effect.

(b) Potential Difference (PD) V


In reference to an electric circuit potential difference between two points is defined as the work
done in moving unit positive
charge from one point to the other
in a specified part of the circuit
and is equal to the product of
current and resistance between the
points in that part, ‘e.g., if in the
external circuit of a cell anode is
connected to cathode through a resistance R and a current I flows through the circuit and W 1 work is
done in moving charge q from anode to cathode as shown in fig.
W1 I2 Rt
V  VA  VB    IR [as W1 = I2Rt and q = It]
q It

Dr.K.K.R Gowtham 57 Current Electricity


In calculating PD between two points we mentally traverse the circuit from one point (say A) to the
other (Say B) and take change in potential to be – IR if I is in the direction of travel and +IR if I is
opposite to the direction of travel, e.g., in fig B.
VA – IR = VB , i.e., V = VA – VB = IR
While in fig. (B)
VA + IR = VB , i.e., V = VB – VA = IR
(c) Internal Resistance r
As the charge q=it goes through the battery from the negative terminal B to the positive terminal A,
work is done by the nonelectrostatic battery force. This work is U1= qE = Eit. As the potential of A
is higher than the potential of B by an amount V, the electric potential energy increases by an
amount U2=Vit.
The remaining energy U1-U2 appears as thermal energy of the battery material. The fraction
appearing as thermal energy depends on the battery material and the battery mechanism. If no
thermal energy is developed as the charge goes through the battery, Eit =Vit Or E = V
Such a battery is called an ideal battery. The potential difference between the terminals of an ideal
battery remains equal to it emf even if there is a current through it.
An ideal cell is that which has no internal resistance. i.e., internal resistance r = 0 for an
ideal cell.
An ideal cell or battery is denoted by the symbol shown in figure . The potential
difference between the facing parallel lines is V= E ,the longer line being at the
higher potential.
A nonideal battery develops thermal energy as a current goes through it and
the potential difference between the terminals is smaller than emf. Such a
battery may be represented by the symbol shown in figure. This is a
combination of an ideal battery of emf E and a resistance r.
If there is a current i through the battery in the direction indicated in
figure, the potential difference between the terminals is
VA- VB =( VA- VC)- (VB- VC) = (E – ir)
The thermal energy developed in time t is i2rt. The addition of resistance r
accounts for the difference between E and V as well as for the thermal
energy developed in the battery. This resistance is called the internal resistance of the battery.
In case of a cell, the opposition of electrolyte to the flow of current through it is called internal resistance
of the cell.
The internal resistance of a cell is the resistance of the electrolyte between the anode and cathode.
The magnitude of internal resistance of a cell depends on:
 1
1) The distance between the plates (r  d ) 2) The area and size of the plates  r  
 A
3) Nature of electrolyte, 4) Strength of the electrolyte or concentration (r  c)

5) Area of the cross-section of the electrolyte through which current flows.


6) Internal resistance depends on temperature. It decreases with increase of temperature.
The internal resistance of a cell can be accurately measured using a potentiometer.
(d) Relation among E, V and r
If a resistance R is connected between positive and
negative terminals of a cell of emf E and internal
resistance r as shown in fig., the current in the circuit by
Ohm’s law will be

Dr.K.K.R Gowtham 58 Current Electricity


E
I or E = IR + Ir …(1)
r  R 
So in terms of ‘terminal voltage’ V (= IR) and ‘internal drop’ X(= Ir) above equation reduces to
E  X  Ir 
E = V + X, i.e.,  1    1  
V  V   IR 
E  r E 
or,  V  1  R i.e., r  R   1
V 
….(2)

This is the required result and from this it is clear that :


(1) When the cell is discharging, i.e., current inside the cell is from cathode to anode; in this
situation terminal voltage.
E
V = (E – X) = (E – Ir) with I  ….(3)
R  r
i.e., when current is drawn from a cell, potential difference across it is lesser than emf of the cell
and greater the current drawn from the cell lesser will be the terminal voltage. This is why when
a heavy current is drawn from a battery, its terminal voltage drops appreciably.
Formulae related with cells

E V  E V 
i
E V
..........(A) r ..........(B) r      E  V  R   E  1 R .... (C)
r i  V   V  V 
 R 
Lost volts : It is the difference between emf and P.D. of a cell.It is used in driving the current
between terminals of the cell.
Lost volts E - V = i r
(2) When the cell is charging, i.e., current inside the cell is from anode to cathode; In this situation
circuit current

I
V  E , i.e., V  E  Ir ….(4)
r
i.e., when a cell is being charged potential difference across its terminals is greater than emf of
the cell and in charging, positive terminal of the charger is connected to the anode of the cell
while negative to cathode.
(3) When the cell is in open-circuit, i.e., external
resistance between anode and cathode is
infinite,
Open circuit resistance is infinity.i.e.,
R   ; In this situation as :

E
I  0, X  Ir  0
 r  
So V=E–X=E …(5)
i.e., open-circuit voltage of a cell is equal to its emf and is the maximum potential difference
which a cell can provide.
(4) When the cell is short-circuited, i.e., external resistance between anode and cathode is zero,
i.e., R 0; In this situation
E E
I  ; So V  1 0  0
 r  0 r
i.e., short-circuit current of a cell is maximum (=E/r) while terminal voltage is zero.

Dr.K.K.R Gowtham 59 Current Electricity


BACK EMF OF A CELL :
e
i) Due to the flow of current, the electrolyte decompose into ions.
These ions travels towards the opposite electrodes and produce an +Cu -Zn
emf in opposite direction of the emf that maintains current. This
opposing emf is called backemf and the phenomena is called Normal
electrolytic polarization. emf
eb
Back
ii) When current flows through the electrolyte solution, due to Polarisation emf

electrolysis, the electrodes get covered with a layer of hydrogen in


the form of gas bubbles. This is called polarization
Electrolyte H2SO4
iii) It resists flow of positive charge carriers and concentration of ions
get, altered at electrodes. This results in an emf acting in a direction opposite to the emf of the cell.
This is called backemf.
iv) To prevent the polarisation in Voltaic cell and Leclanche cell, the depolarisers like potassium
dichromate (or) manganese dioxide is to be added to electrolyte.
v) Depolariser is essentially an oxidising agent that neutralises hydrogen and there by prevents the
covering of the copper plate and nullifies backemf.
Electric cells are of two kinds (i) Primary cells and (2) Secondary cells.
Primary cells: These are the cells which provide current as a result of chemical reaction, but
cannot be recharged i.e, chemical reaction is irreversible. Their emf is less and internal resistance is
more. Simple voltaic cell, Laclanche cell, Daniel cell, dry cell ,cadmium cell etc., are the primary
cells
Secondary cells: These are the cells which can be recharged after use. Thus, as a result of
reversible reaction, the electrical energy can be stored in it. Their emf is more and internal
resistance is less. Lead accumulator, Ni-Fe (Edison) cell or alkali accumulator etc., are the
secondary cells.
Note:
1. The current inside a cell is always due to to motion of both positive and negative ions while out side
it depends on the circuit element.
2. a cell neither creates nor destroys charge but maintains the flow of charge present in various parts of
the circuit by supplying energy needed for their orgnised motion.
3. if the resistance in a circuit containing a cell changes the current will also change but emf of the cell
remains unchanged, i.e., cell is a source of constant emf.
4. Higher the emf of a cell and smaller the internal resistance, greater will be the current which can be
drawn from the cell.
5. With the use of a cell internal resistance increases appreciable while emf falls slightly. This is why a
fresh cell gives more current as compared to old one foe the same load.
6. In reference to a battery ‘capacity’ refers to the product of current (in ampere) and time(in hour) for
which is cell can operate, e.g., if the capacity of a Flashlight battery is 6Ah at 2 hr rating the cell
can deliver a current of 3 amp for 2 hr or of 1amp for 6 hr. This is why cell works for longer period
if small current drawn from it. The capacity of a cell depends on the amount of electrolyte and
hence its size. A biggest cell contains more electrolyte also has greater capacity than a smaller one
of same type.
GROUPING OF CELLS
From a single cell, only a limited amount of current can be drawn. In many experiments, the current
required may be much more than that can be supplied by a single cell. In order to have a large value
of current, the cells are usually connected in series or in parallel or the cells are mixed grouped.
There are three types of grouping of cells.
1) Cells in Series , 2) Cells in Parallel,
3) Cells in Mixed Grouping Group of cells is called a ‘battery’.

Dr.K.K.R Gowtham 60 Current Electricity


(a) Series Grouping
If the negative terminal of a cell is connected to the positive terminal of the second and negative
terminal of second to the positive terminal of third and so on leaving the anode of first and cathode of
the last, the cells are said to be in series
Cells of different emf in series aiding :
The two cells are said to be connected in series aiding between E1, r1 B E2, r2
two points A and C if positive terminal of one cell is joined A C
a
negative terminal of another cell.
E eq , rep
(Two cells in series)
Let E1 , E2 be the emfs of the two cells. Let r 1, r2 be their internal
b
resistances respectively. Let the cells be sending the current in a circuit not shown in Fig. Let VA,
VB and VC be the potential at points A, B and C, and I be the current flowing through them. Potential
difference betweeen positive and negative terminals of the first cell,
VAB  VA  VB  E1  Ir1
Potential difference between positive and negative terminals of the second cell.
VBC = VB – VC = E2  Ir2 P.D. between A & C of the two cells,
VAC  VA  VC  (VA  VB)  (VB  VC)  (E1  Ir1)  (E2  Ir2)  (E1  E2)  I(r1  r2) ... (i)\
Let the series combination of two cells be replaced by a single cell between A and C of emf
Eeq and internal resistance r eq then
VAC  Eeq  Ireq ....... (ii)
Comparing (i) and (ii), we get Eeq  E1  E2 and req  r1  r2
If n cells of emfs E1, E 2 .......En and of internal resistances r1, r2 ................rn respectively, are
connected in series between A and C, then equivalent emf, Eeq  E1  E2  .....  En
equivalent internal resistance req  r1  r2  ....  rn.
Rules for series combination of cells are as follows :
(i) The equivalent emf of a series combination of cells is equal to the sum of their individual emfs.
(ii) The equivalent internal resistace of a series combination of cells is equal to sum of their
individual internal resistances.
Cells of identical emf in series aiding :

In this situation if all the s cells are identical


each having emf E and internal resistance r
Eeq = sE and req = sr
So current in the load resistance R :
E eq sE
I 
 req  R  R  sr
….(1)
This is the required result and from this it is clear that :
(1) If sr << R, I = s(E/R), i.e., if effective internal resistance is lesser than external resistance, current
in the circuit is s times that of the circuit current due to single cell (E/R)

Dr.K.K.R Gowtham 61 Current Electricity


(2) If sr >> R, I = (E/r), i.e., if effective internal resistance is greater than external resistance, current
in the circuit is equal to that of a single short circuited cell (E/r).
(3) If in series grouping of s cells ‘n’ cells are reversed Eeq = (s – n) E – nE = (s – 2n) E and req = sr

So, I
 s  2n  E
R  sr
(4) If the cells in series are not identical (E, r) (E, r) (E, r) (E, r) (E, r)

>
Eeq = E1 + E2 + … = Ei and req = r1 + r2 + … = ri I

>
E i
So, I
R  ri > >
R
Cells in series opposition:
In the series combination of two cells, if negative terminal of first cell is connected to the negative
terminal of the second cell between points A and C, as shown in Fig.
Let E1 , E2 be the emfs of the two cells. Let r1, r2 be their internal resistances respectively. Let the
cells be delivering the current in a circuit .

Let VA, VB and VC be the potential at points A, B and C, and I be E1 r1 E2 r2


the current flowing through them. Potential difference
A I B I I C
betweeen positive and negative terminals of the first cell,
Two cells in series opposing
VAB  VA  VB  E1  Ir1
Potential difference between positive and negative terminals of the second cell.
VBC = VC - VB = E2 + Ir2
P.D. between A & C of the two cells,
VAC  VA  VC  (VA  VB)  (VB  VC)  E1 - Ir1  E2 - Ir2  (E1  E2) - I(r1  r2) ... (i)
Let the series combination of two cells be replaced by a single cell between A and C of emf
Eeq and internal resistance r eq then
VAC  Eeq  Ireq ------------------(ii)
Comparing (i) and (ii), we get Eeq  E1  E2 and req  r1  r2
Then equivalent emf of the two cells is Eeq = E1 – E2
But equivalent internal resistance is req = r1 + r2

EXAMPLE: In the circuit shown in figure, E1  10V , E2  4V , r1  r2  1 E1 r1 E2 r2


and R  2. Find the potential difference across battery 1 and
battery 2.
E1 r E2 r2
1

Solution
Net emf of the circuit  E1  E2  6V I V1 V2
Total resistance of the circuit  R  r1  r2  4

Dr.K.K.R Gowtham 62 Current Electricity


net emf 6
 Current in the circuit I    1.5 A
total resist ance 4

Now, V1  E1  Ir1  10  1.51  8.5V


and V2  E2  Ir2  4  1.51  5.5V
Ex: n identical cells each of emf 6V are connected in series with an
 
external resistor of 5 and carry a current of 10 emp. If two
cells are connected wrongly in series with the same external r r
resistor the current flowing through the cells will be 6.45 amp.
Find the value of n and internal resistance of each cell.
For series connection   
 eff  n  6n reff  nr
n 6n
or, i   10 n  2cell
nr  R nr  5  eff  n  2
For wrong connection of two cells the effective emf decreases
by 2 because emf of two identical cells counteracts with each other.
Then,  'eff  n  2   n  2    n  2 6
And r 'eff  nr

or, i' 
 n  2     n  2  6  6.45
nr  R
nr  5
By solving eqs. 9i) and (ii), we have n  10 and r  0.1
(b) Parallel grouping
suppose the batteries are connected in parallel as shown in figure.The currents are also shown in
the figure. Applying Kirchhoff’s loop law in the loop containing E 1r1 and R
Ri  ri
1 1  E1  0 ---(1)
Similarly, Applying Kirchhoff’s loop law in the loop containing E2r2 and R,
Ri  r2 (i  i1 )  E2  0 ---(2)
Multiply (1) by r2. (2) by r1 and add. This gives
Ri (r1  r2 )  r1r2i  E1r2  E2 r1  0
E1r2  E2 r1
E1r2  E2 r1 r1  r2 E0
or i   
R(r1  r2 )  r1r2 R  r1r2 R  r0
r1  r2
Ei
E1r2  E2 r1
r
We see that the combination acts as a battery of e mf E0  E0  i

r1  r2 1
r
i
r1r2 1 1 1
And internal resistance r0  or  
r1  r2 r0 ri r2

Dr.K.K.R Gowtham 63 Current Electricity


n
Ei
r 1 n
1
For n cells E0  i 1
n
i
and 
1
r
r0 i 1 ri
i 1 i
Parallel grouping (identical cells)
If positive terminals of all cells are connected at
one junction while negative at the other, the cells
are said to be in parallel.

In this situation if all the p cells are identical,


Eeq = E and req = (r/p)
And so current in the circuit,
Eeq E
I  …(2)
R  req R  r / p
This is the required result and from this it is clear that :
(1) If (r/p) < < R, I = (E/R), i.e., if effective internal resistance (= r/p) is lesser than external
resistance R, current in the circuit is equal to the circuit current due to a single cell (= E/R).
(2) If (r/p) > > R, I = p(E/r), i.e., if effective internal resistance is greater than external resistance,
current in the circuit is equal to p times that of single short circuited cell (E/r)
(3) If the cells are not identical the problem is solved by using Kirchhoff’s laws and in this situation
the potential difference V across the external resistance R is such that
Emin < V < Emax
Where Emin and Emax respectively mean the minimum and maximum emf of the cells to be
arranged in parallel.
(4) A comparison of case (a) and (b) reveals that to get maximum current, cells must be connected in
series if effective internal resistance is lesser than external and in parallel if effective internal
resistance is greater then external.

(c) Mixed Grouping


The situation is shown in fig. If there are s
identical cells in a row and there are p rows,
total cells will be s  p = n

Treating each row as a single cell of emf sE


and internal resistance sr, by concept of grouping of cells in parallel,
Eeq = sE and req = (sr/p)
sE E
So, I  ….(3)
R  (sr / p)  R / s    r / p 
From this expression it is clear that :
(1) Current in the circuit will be maximum when
R r  d  pR r   n
 s  p   min, i.e., dp  n  p   0 as s  p 
     
R r R r
or,  2  0, i.e.,   as i  s  p  ….(4)
p p s p

Dr.K.K.R Gowtham 64 Current Electricity


so in case of mixed grouping of cells, current in the circuit will be maximum when (R/s) = (r/p)
sE pE
(with s  p = n) and Imax   …(5)
2R 2r
(2) Power transferred to the load will be maximum when in accordance with maximum power
transfer theorem load resistance is equal to internal resistance, i.e.,
s R r
R r or 
p s p
Which is Eq. (4), So in mixed grouping of cells power transfer to the load is maximum when
current in the circuit is maximum and as here
E2R s2 E 2 p2 E 2
P ; Pmax   R …(6)
 R / s   (r / p) 
2
4R 4r 2
(3) If in a problem of mixed grouping of cells for maximum current (or power) p and s come out to
be fractional, then as physically neither rows nor cells in a row can be fractional, taking proper
integral values of p and s nearest to fractional values such that s  p = n, calculate I. Proper
choice will be the integral combination of p and s which provides the maximum current. (or
power).
EXAMPLE: 10V
Find the emf and internal resistance of a single battery which is 6V 2
equivalent to a combination of three batteries as shown in figure.
1 2

4V
Solution
The given combination consists of two batteries in parallel and
resultant of these two in series with the third. For parallel 6V 3V
combination we can apply,
1 1
E1 E2 10 4
 
r1 r2
Eeq   2 2  3V
1 1 1 1
 
r1 r2 2 2
E  3V

r  2 Further,
1 1 1 1 1
    1  req  1
req r1 r2 2 2
Now this resistance is in series with the third resistance so the equivalent emf of these two is
 6  3 V  3V and internal resistance will be 1  1  2 . 3V 1
Ex:Three cells of emf 3V, and 6V are connected in parallel. If their internal
resistances are 2,1 and 4.1 find the  a   eff  b  reff  c  current in 2
the external load R  4.1 .
4V 2

Solution: The cells are conceted in parallel. 6V


The equivalent cell emf is eeff 
  / ri i

1/ ri 1.6

Dr.K.K.R Gowtham 65 Current Electricity


3  4 6
 
1  2 1
  2.8V
1 1 1
 
1 2 1
The internal resistance of the cell is
1 1 1 1 1 2
      2.5 or, reff 
reff ri 1 2 1 5
The current in the circuit is E  2.8V r  1.5
 2.8
i   1.4amp
Rr 8 2

5 5
R  4.1
WHEATSTONE BRIDGE B
I3

<
A resistance can be measured by Ohm`s law using a voltmeter and an P
IG Q

>
ammeter in an electrical circuit. But this mesurement is not accurate. To I1
measure it more accurately Kristie devised and Wheatstone popularized A C
<
> >
a special network design called Wheatstone Bridge. It is an arrangement I2
of four resistances which can be used to measure one of them in terms R S
I4
<
of the rest.
D
I

>
>
( )
E K
Four resistors P, Q, R, and S are connected as four sides of a quadrilateral ABCD as shown.
A Galvanometer of resistance G is connected across one set of diagonally opposite corners B and D.
A source of emf E with plug key K is connected across another set of diagonally opposite corners A
and C. When key K is closed current I is drawn from source. It spilts as I1 and I2 at junction A and
flows through P and R resistors. Current I1 spilts at junction B and flows as IG and I3 through
galvanometer and resistor Q respectively. IG and I2 combine at junction D and flows as I4 through
the resistor S.
Condition for balancing of bridge :
Applying Kirchhoff`s first law at junction B and D we get I1 – I3 – IG = 0 ; and I2 + IG – I4 = 0
Applying Kirchhoff’s second law for closed loop ABDA, - I1P - IGG + I 2 R = 0
Applying Kirchhoff’s second law for closed loop BCDB , - I3Q + I 4 S + I GG = 0
The values of P, Q, R, S are adjusted such that I G becomes zero. At this stage the bridge is said to
be in balance condition.
i.e., In balanced condition of bridge IG = 0
In balanced condition the above equations respectively become
I1 = I3 ........(1)
and I2 = I4 ........(2)

Dr.K.K.R Gowtham 66 Current Electricity


I1P = I 2 R ...........(3)
I3Q = I 4 S ............(4)
Dividing equation (3) by equation (4)
I1P I 2 R

I 3Q I 4 S
P R
Using equations (1) and (2) we get  .......(5)
Q S
This is the balancing condition for Wheat stone bridge.
USES OF WHEATSTONE BRIDGE :
P R
1) We can compare two unknown resistances R and S from 
Q S
2) If the value of S is also known, we can calculate the value of unknown resistance R.
3) The small strains produced in hard materials can be measured accurately with a Wheatstone bridge
using a strain gauge. (A strain gauge is a small, flat coil of wire that is firmly attached to the
material under strain. The strain produced gives rise to a change in resistance of the coil, which can
be measured with the Wheatstone bridge. From this, the strain can be calculated).
Note :
1.The balance condition given by eq (5) at null position is independent of the applied voltage E. In
other words if we change the e.m.f. of the cell, the balance will not change.
2.The measurement of resistance does not depend on the accuracy of calibration of the
galvanometer. Galvanometer is used only as a null indicator.
3.The main factor affecting the accuracy of measurement by Wheatstone Bridge is its sensitivity
with which the changes in the null condition can be detected.
4.It has been found that the bridge has the greatest sensitivity when the resistances are as nearly
equal as possible.
The bridge is most sensitive if P = Q = R = S.
Note :
a) The condition for balance is not effected when positions of battery and Galvanometer are inter
changed.
b) Equivalent resistance of balanced bridge across the ends of battery when the bridge is balanced is
given by
 P  Q  R  S 
PQ R S
c) There are seven closed meshes in Wheatstone’s bridge
Three other common forms of balanced Whatstone’s Bridge are shown here.

Dr.K.K.R Gowtham 67 Current Electricity


P Q
I  I1 R E
P
I1 I G G  I  I1  I G  G Q R
B and S
I S I A
P
R B
 I1  IG  Q S G
B
P Q
Direction of current in an unbalanced Wheatstone’s bridge : i A
i1
C
R  S P >

>
i2 >

>
VAB  VA  VB  i1P  i
PQ R S R
S

VAD  VA  VD  i2 R  i
 P  Q R D

PQ R S
i
 P  Q  R   R  S  P 
PQ R S 
i
 QR  PS 
PQ R S

if QR  PS , VB  VD  current flows from B to D

QR  PS ,VB  VD  current flows from D to B


B
QR  PS , VB  VD  Balanced bridge
20 10
Ex: Find the value of R in Fig. so that there is no current in the
resistor. A C
> >
15

40
R

40  10  D
R   20 
20
Sol : This is the Wheatstone bridge with the galvanometer replaced by resistor. The bridge is balanced
because there is no current in resistor, hence, R6
20 / 10 = 40 / R
R1 R3
Ex:In the given circuit it is observed that the current I is independent R5
of the value of theresistance R5.Then what is the relation the resistance
R2 R4
values must satisfy.
Sol : R1 R4  R3 R2
R3 R1
HInt :  ,  R1R4  R3 R2
R4 R2

Dr.K.K.R Gowtham 68 Current Electricity


METER BRIDGE : R S
RB
Meter bridge is a practical application of Wheatstone bridge. B
Figure shows the meter bridge. Meter bridge consists of a wire of
length 1 m and of uniform cross sectional area stretched taut and
clamped between two thick copper strips bent at right angles. A D C
As shown in fig. The copper strips has two gaps. In between > 100 1
the gaps, the resistors can be connected. The wire is clamped 1 HR
at the end . The circuits are connected to a cell through a G
key. One end of a galvanometer is connected to the copper E K
strip midway between the two gaps. The other end of the galvanometer
is connected to a jockey ‘which is essentially a metallic rod whose one end has a knife-edge and can
slide over the wire to make electrical connection. R is an unknown resistance, whose value we want
to determine which is connected across one of the gaps. Across the other gap, we connect a standard
known resistance S. The jockey is connected to some point D on the wire, a distance l cm from the
end A. The jockey can be moved along the wire. The portion AD of the wire has a resistance l ,
where  is the resistance of the wire per unit centimeter. The portion DC of the wire has a
resistance  (100  l ) . The four arms AB, BC, DA and CD [with resistances R, S, l
and  (100  l ) ] obviously form a Wheatstone bridge with AC as the battery arm and BD the
galvanometer arm. The galvanometer shows no current at one position, if we move the jockey along
the wire. Let the distance of the jockey from the end A at the balance point be l= l1. The four
resistances of the bridge at the balance point are R, S,  l1 and  (100–l1). The balance condition,
R l1 l1
gives   If we find l1 then the unknown resistance R is known in terms of
S  (100  l1 ) 100  l1
l1
the standard known resistance S by R  S ........(1) We would get different values of l1 by
100  l1
choosing different values of S and calculate R each time. If you get an error in measurement of l1,
then we will get an error in R. The percentage error in R can be minimized by adjusting the balance
point near the middle of the bridge.
Uses of Meter bridge
a) to compare resistances of two resistors
b) to measure resistance of unknown wire
c) to determine specific resistance of a resistor
d) to verify laws of resistances in series and parallel combination
Note :
a) A high resistance is connected in series to galvanometer to protect it from high currents.
b) By any reason, if resistance in the left gap increases or resistance in the right gap decreases,
balancing point shifts towards right side. By any reason, if resistance in the left gap decreases or
resistance in the right gap increases, balancing point shifts towards left.
c) With a metre bridge it is advised to obtain. a balancing point in the middle one third of the wire for
more accuracy.
d) In a meter bridge experiment,metallic conductor is placed in the left gap and it is heated, then the
balancing point will shifts towards right gap (balancing length increases).
e) In a meter bridge experiment, semi conductor is placed in the left gap and it is heated, then the
balancing point will shifts towards left gap (balancing length decreases).

Dr.K.K.R Gowtham 69 Current Electricity


Drawbacks with Metre Bridge :
The metre bridge is not a suitable device for the determination of high or low resistences. This is
because (i) there will be certain unknown resistances of contact at the ends of the wire AB that are in
contact with the terminals (binding screws). Hence we cannot measure low resistances. (ii) The metre
bridge wire may not be of exactly one metre length. This gives rise to an error in the reading of the
remaining length .
Ex: In a metre bridge, the null point is found at a distance of 33.7 cm from A. If now a resistance of
is connected in parallel with S, the null point occurs at 51.9 cm. Determine the values of R and S.
R 33.7
Sol : From the first balance point,  ........ (i)
S 66.3
After S is connected in parallel with a resistance of 12 , the resistance across the gap changes from
12S 51.9 R R S+12 
S to Seq, where Seq  and hence the new balance condition gives  
S+12 48.1 Seq 12S
51.9 S+12 33.7
Substituting the value of R/S from equation (i), we get  . which gives S  13.5  .
48.1 12 66.3
Using the value of R/S above, we get R  6.86 
Ex: A resistance of 2W is connected across one gap of a metre-bridge (the length of the wire is 100
cm) and an unknown resistance, greater than 2W, is connected across the other gap. When these
resistances are interchanged, the balance point shifts by 20 cm. Neglecting any corrections, the
unknown resistance is
Sol : Refer to the diagram
Apply the conditions of the balanced Wheatstone’s bridge for the two cases.
2 l
 ..................................(i)
x 100  l
x l  20
 ....................................(ii) G
2 80  l
Equations (i) and (ii) give x = 3W 100-l
l
POTENTIOMETER
Potentiometer is an instrument which can measure accurately the emf of a source or the potential
difference across any part of an electric circuit without drawing any current.
Principle : The principle of potentiometer states that when a constant current is passed through a
wire of uniform area of cross-section, the potential drop across any portion of the wire is directly
proportional to the length of that portion.
A. CIRCUIT DIAGRAM
Potentiometer consists of a long resistive wire AB of length L (about 6m to 10 m long) made up of
mangnin or constantan and a battery of known voltage e and internal resistance r called supplier
battery or drier cell. A I
Connection of these two forms primary circuit. > Copper
>

I strip
E
One terminal of another cell (Whose emf E is to be r measured) is
connected at one end of the main circuit and the other terminal at any point
RS
on the resistive wire through a galvanometer G. This forms the secondary
circuit. Other details are as follows:
>

The principle of potentiometer require that <


B I

Dr.K.K.R Gowtham 70 Current Electricity


a) The specific resistance  of potentiometer wire e,r K Rh
must be high but its temperature coefficient of
resistance   must be low. Primary
circuit J
A
b) All higher potential points (terminals) of primary and
Secondary
secondary circuits must be connected together at point circuit E G
A and all lower potential points must be connected to
point B or jockey. J = Jockey
c) The value of known potential difference must be K = Key
greater than the value of unknown potential difference R = Resistance of potentiometer wire
to be measured.   Specific resistance of potentiometer wire
d) The potential gradient must remain constant. For this Rh = Variable resistance which controls
the current in the primary circuit must remain the current through the wire AB
constant and the jockey must not slide while in
contact with the wire.
e) The diameter of potentiometer wire must be uniform everywhere.
B. POTENTIAL GRADIENT (x)
Potential difference (or fall in potential) per unit length of wire is called potential gradient i.e.,
V volt  e  V IR l  e R
x where V  IR   R So, x    
L m  R  Rh  r  L L A  R  Rh  r  L
a) The potential gradient directly depends upon the
R
(i) resistance per unit length   of potentiometer wire.
L
(ii) radius of potentiometer wire (i.e., Area of cross-section)
(iii) specific resistance of the material of potentiometer wire (i.e.,  )
(iv) current flowing through potentiometer wire (I)
b) Potential gradient indirectly depends upon the
(i) emf of battery in the primary circuit (i.e., e)
(ii) resistance of rheostat in the primary circuit (i.e., Rh ) Rh
e,r K
C.WORKING
Suppose the jockey is made to touch a point J on wire. Then
potential difference between A and J will be V  xl . At this A
J1 J2 J3
B
length  l  , two potential difference values are obtained.
G G G
a) V due to battery e and b) E due to unknown cell E
If V > E, then current will flow in galvanometer circuit in one

direction, shown as If V < E, then current will flow in

galvanometer circuit in opposite direction, shown as


If V = E, then no current will flow in galvanometer circuit this condition to known as null deflection

position, length l is known as balancing length, shown as


In balanced condition  E  xl
V IR  e R
 E  xl  l   l
L L  R  Rh  r  L
x1 L1 l1
If V is constant then L  l   
x2 L2 l2

Dr.K.K.R Gowtham 71 Current Electricity


D. STANDARDIZATION OF POTENTIOMETER Rh
e,r K
The process of determining potential gradient experimentally is
known as standardization of potentiometer.
Let the balancing length for the standard emf E0 is l0 , then by the A J
B
E
principle of potentiometer we have E0  xl0  x 0 E0 G
l0
E
E. SENSITIVITY OF POTENTIOMETER
Potentiometer is said to be more sensitive if it measures less potential
difference more accurately i.e. lower the potential gradient higher will be the sensitivity and vice
versa.
1 L L LRT
Sa a a a Here L is length of potentiometer wire,
pot.gradiant V iR p ERP
R p is resistance of wire, E is emf of primary cell
RT is total resistance of primary circuit. i is current in the primary circuit.
Sensitivity can be increased by
1. Increasing length of the wire ( L ) 2. Decreasing Potential gradient
3. Decreasing the current in the primary circuit
4. Decreasing resistance of the potentiometer wire
5. Decreasing the emf of the cell in primary circuit
6. Increasing the total resistance of the primary circuit.
Note:If x is the length on the wire at a position, l is a balancing length then
a. If x = l – no current flows in secondary circuit.
b. if x< l – Current flows from primary to secondary
c. If x > l – Current flows from secondary to primary
APPLICATIONS OF POTENTIOMETER
A. TO CETERMINE THE INTERNAL RESISTANCE OF A PRIMARY CELL
a) Initially, in the secondary circuit the key K ' remains open and balancing length  l1  is obtained.
Since cell E is in open circuit so it’s emf balances for a length l1 i.e.,
E  xl1 …(1)
b) Now key K ' is closed so cell E comes in closed circuit. If the process of balancing repeated
again then potential difference V balances on length l2 i.e.,. V  xl2 …(2)
c) By using the formula for internal resistance Rh
K
E  e,r
given by r    1 R ' , we get
V 
J
 l l  A B
r   1 2 R'
 l2  E
G

K'
R'

Dr.K.K.R Gowtham 72 Current Electricity


B. COMPARION OF EMF’S OF TWO CELL
Let l1 and l2 be the balancing lengths with the cells E1 and E2 Rh
e,r K
respectively then E1  xl1 and E2  xl2
E1 l1
 
E2 l2 A J
B
Let E1  E2 and both are connected in series . If balancing length is E1 1 G
l1 , when cells assist each other and is l2 , when they oppose
each other ( as shown) , then E2
2
E1 E2 E1 E2

 E1  E2   xl1  E1  E2   xl2
E1  E2 l1 E l l Rh
   1  1 2 K
E1  E2 l2 E2 l1  l2
C. COMPARISON OF RESISTANACES
Let the balancing length for resistance R1 ( when XY is connected) A J
B
is l1 and let balancing length for resistance R1  R2 (when
X YG Z
YZ is connected) is l2 .

i R1 R2
Then IR1  xl1 and I  R1  R2   xl2
R2 l2  l1 Rh1
  K1
R1 l1
D. CALIBRATION OF AMMETER
Checking the correctness of ammeter readings with the help of potentiometer is called calibration
of ammeter.
a) In the process of calibration of an ammeter the current flowing in a circuit is measured by an
ammeter and the same current is also measured with the help of potentiometer . By comparing both
the values , the error(s) in the ammeter readings can be determined.
b) For the calibration of an ammeter , 1 standard resistance coil is e K1
specifically used in the secondary circuit of the potentiometer,
because the potential difference across 1 is equal to the current
flowing through it. So, from Ohm’s Law we get V  l
A
c) If the balancing length for the emf E0 is l0 then E0  xl0 E1 B
1
E0
 x (Process of standardization) 2 G
l0 1 3
A
d) Let I ' be the current that flows through 1 resistance , thus giving
the potential difference as V '  I ' 1  xl1 where l1 is the
balancing length. So error can be found as K2
E 
I  I  I '  I  xl2  I   0  l1
 l0 

Dr.K.K.R Gowtham 73 Current Electricity


E. CALIBRATION OF VOLTMETER e K1
a) Practical voltmeters are not ideal, because these do not have infinite
resistance . The error of such practical voltmeter can be found by
comparing the voltmeter reading with calculated value of potential
A B
difference by potentiometer. E0 1 C
b) If l0 is balancing length for E0 , the emf of standard cell( by 2 G
E v
connecting 1 and 2 of bi-directional key), then x  0 RB
3
l0
c) The balancing length l1 for unknown potential difference V ' is
 E0  K2 Rh
given by ( by closing 2 and 3), then V '  xl1    l1
 l0 
If the voltmeter reading is V then the error will be V  V ' which may be positive, negative or
zero.
DIFFERENCE BETWEEN VOLTMETER AND POTENTIOMETER

Voltmeter Potentiometer
It’s resistance is high but finite It’s resistance is infinite
It draws some current from source of emf It does not draw any current from the source of
unknown emf
The potential difference measured by it is lesser The potential difference measured by it is equal
than the actual potential difference to actual potential difference
Its sensitivity is low Its sensitivity is high
It is a versatile instrument It measures only emf or potential difference
It is based on deflection method It is based on null deflection method

EXAMPLE: A cell having a steady emf of 2V is connected across the potentiometer wire of
1
length 10m. The potentiometer wire is of manganin and having resistance of 11.5m . An
another cell gives a null point at 6.9m. If a resistance of 5 is put in series with the
potentiometer wire, find the new position of the null point.
Solution
Length of potentiometer wire =10m
2
So, fall in potential per meter (also known as potential gradient)   0.2Vm 
10
Since the cell gives a null point at 6.9m, hence its emf is,
E   0.2 6.9  1.38V
Resistance of potentiometer wire  11.5 10  115
When a resistance of 5 is connected , total resistance becomes Rnet  115  5  120 and
2
Current, I  A
120
So, new potential gradient=(current) resistance per unit length)
 2  23
 Potential Gradient =   11.5  Vm 1
 120  120
Let l ' be the position of new null point, then
 23  1.38 120
  l '  E  1.38  l'  7.2m
 120  23

Dr.K.K.R Gowtham 74 Current Electricity


EXAMPLE: A potentiometer wire of length 1 m has a resistance o 10 ohm. It is connected in series
with a resistance R and a cell of emf 3V and negligible internal resistance. A source of emf 10 mV
is balanced against a length of 60 m of the potentiometer wire. Find the value of R.
Solution : Following the theory of potentiometer, VAB  iRAB
  
  RAB ,
 R  RAB  10mV
  3V , RAB  10, VAB  10 103V 60cm BH4
AB 60 A C
And RAC  RAB   10  6,
AC 100 100cm
 3 
We have 10 103   6
 R  10 
or, R  1790  R
HEATING EFFECTS OF CURRENT
JOULE’S LAW:
whenever electric current is passed through a conductor, it becomes hot after some time. This
indicates that the electrical energy is being converted into heat energy. This effect is known as
heating effect of current or joule heating effect. If we reverse the direction of flow of current, heat is
still produced in the conductor. Joule heating is an irreversible phenomenon and it cannot be used to
convert heat into electrical energy.
According to Joule’s law, the current passing through a conductor produces heat.
The quantity of heat produced ‘Q’ is equal to the work done by the electric field on the free
electrons. If constant current ‘i’ passes through a conductor for time ‘t’ under a potential difference ‘V’,
then work done (W) in the time ‘t’ is W = Vit
But according to Ohm’s law, V = iR where ‘R’ is the resistance of the conductor.
2
Now, work done, W = (iR) i t = i2 R t = V t = V i t
R
This work is converted into energy in the conductor. It means the electric current through a
conductor produces thermal energy in the conductor and the conductor gets heated.
i 2 Rt
Thermal energy produced, Q = i Rt ( in joule) Q 
2 (in calories)
J
From the above equation, we can say that
i) The heat produced in a given conductor in a given time is proportional to the square of the current
passing through it. i.e. Q  i 2 ,
ii) The heat produced in a given conductor and current is proportional to the time for which the current
passing through it. i.e. Q  t ,
iii) The heat produced in a given conductor by the current in a given time is proportional to the
resistance of the conductor. i.e. Q  R ,
These laws are known as Joule’s laws.
Electric heater, electric iron, electric bulb, electric stove etc., are some of the instruments which
work on the Joule’s law and convert electrical energy into heat energy.

Dr.K.K.R Gowtham 75 Current Electricity


ELECTRIC POWER
The rate at which electrical work is done by the source of e.m.f in maintaining the current in electric
circuit is called electric power of the circuit.
If a current “i” ampere flows through a conductor for a time ‘t’ second under a potential difference
of ‘V’ volt, then electric work done to maintain the current is given by W = Vit joule.
Electric power, P  W  Vit  Vi ----- (1) SI unit--- watt or Js–1
t t
It V = 1volt and i = 1 ampere then P = 1 watt
Thus the power of an electric current is said to be one watt if one ampere current flows in it
against a potential difference of 1 volt.
The biggest units of electric power are kilo watt (kW) and mega watt (MW)
1kW = 1000 W and 1MW = 106 W
Commercial unit of power is horse power (H.P) where 1H.P = 746 watt
Expression for electric power in terms of i and R
According to ohm’s law, V = iR
Now equation (1) becomes P = i2R ----- (2)
Expression for electric power in terms of V and R
V
We know that, V = iR  i 
R
V2 V2
Now equation (1) becomes, P P  VI  I 2 R
R R
Note :
a) If resistances are connected in series , i.e.., I is same
P  R with V R [ as V  IR ]
i.e.., in series potential difference and power consumed will be more in larger resistance.
However, if resistances are connected in parallel, i.e.., V is same
1 1
P with I  [ as V  IR ]
R R
i.e.., in parallel current and power consumed will be more in smaller resistance. This in turn implies
that more power is consumed in larger resistance if resistances are in series and in smaller resistance if
resistances are in parallel.
b) A resistance R under a potential difference V dissipates power. P  V 2 / R 
So If the resistance is changed from R to (R/n) keeping V same, the power consumed will be
V2 V2
P 
1
n  nP
( R / n) R
i.e.,if for a given voltage, resistance is changed from R to (R/n),power consumed changes from P to nP.
c) If n equal resistances are connected in series with a voltage source, the power dissipated will be
V2
Ps  [ as Rs  nR ]
nR
And if the same resistances are connected in parallel with the same voltage source

Dr.K.K.R Gowtham 76 Current Electricity


V2 nV 2
Pp   [as Rp  ( R / n)]
( R / n) R
Pp
So,  n 2 i.e.., PP  n 2 PS .
Ps
i.e.., power consumed by n equal resistors in parallel is n 2 times that of power consumed in series if
V remains same.
d) Where VS and W are the voltage and wattage specified(rated) on the appliance.
An electric appliance consumes the specified power W(called rated wattage) only if it runs at the
specified voltage VS (called rated voltage).
As resistance of a given electric appliance ( e.g.., bulb , heater, geyser or press ) is constant and is
given by,
VS VS V2 W
R   s [as I  ]
I (W / VS ) W V

If the applied voltage VA is greater than the rated voltage the appliance will get damaged (fused) as
in this situation I=VA/R will exceed it current capacity I C = VS/R.
So if the applied voltage is less than the specified, the ‘actual power consumption’ is less than rated
2
V 2 V  VS 2
power consumption. actual power consumption’ will be P  A   A   W [ as R  ].
R  VS  W
ELECTRICAL ENERGY:
Our household consumption of electrical energy is measured in kilowatt-hours. One kilowatt-hour
is defined as the electrical energy consumed at the rate of one kilowatt ( one thousand watts) for one
hour.
1 kilowatt-hour = ( 1000 watts) (1 hour) = ( 1000 watts) (3600 seconds) = 36 105 joules

1 kilowatt-hour = 36 105 joules


Usually we call this 1 kWh as one “unit” and our house hold electrical consumption is charged
accordingly. The number of units mentioned in our monthly electrical bill refers to the number of
kilowatt- hours of energy consumed over the month.
If number of appliances of wattage W1 , W2 ... etc .., are operating for hours W1 , W2 ... etc..,
respectively at the specified voltage ‘ electrical energy consumed’ in their operation
E = [W1h1 + W2 h2 + ...] = å Wh
[ as P = ( E / t)]
W1h1 W1h1
However the practical unit of electrical energy is kWh, So, E  kWh  units
1000 1000
And if each unit costs ( C Rs,) the cost of consumed electric energy(i.e.., electric bill) will be Rs ( C  E).
Ex: A 1 kW heater is meant to operate at 200 V. (a) What is its resistance ? (b) How much power will
it consume if the line voltage drops to 100 V ? (c) How many units of electrical energy will it
consume in a month ( of 30 days ) if it operates 10 hr daily at the specified voltage ?

Sol : a) The resistance of an electric appliance is given by R 


VS 2
, so R 
 200 2

 40 ,
W 1000

Dr.K.K.R Gowtham 77 Current Electricity


2
 VA 
b) The ‘ actual power ‘ consumed by an electric appliance is given by P     W , so,
 VS 
2
 100 
P  1000  250W
 200 
c) The total electrical energy consumed by an electric appliance in a specified time is given by,
W1h1 1000  (10  30)
E kWh so, E   300kWh =300 unit
1000 1000
BULBS IN SERIES
Every electrical appliance will have rated or design values like wattage, voltage printed on it. These
values give information about resistance and allowable current etc., Let P and V be the power and
V2
voltage ratings on a bulb. Resistance of filament of the bulb R
P
1
If V is constant, R
P
So, if we compare (100 W – 230 V) and (60W – 230 V) bulbs filament of 100 W bulb will have
P
less resistance compared to that of 60 W bulb. Allowable current in the bulb i 
V
So, i  P if V is constant. So, in the above case, 100 W bulb draws more current.
If two bulbs with power ratings P1 and P2 are connected in series with a voltage source V.
If P1' and P2' are consumed powers of the two bulbs,

P1 i 2 R1 R1
 2  If P1 > P2 then R1 < R2  P1' < P2'
P2 i R2 R2
So, in series bulb with low wattage rating glows brighter than high wattage rated bulb.
Now total power consumed is
V2 V2 V2 V2 1 1 1
P  But R1  and R2    
R R1  R2 P1 P2 P P1 P2
BULBS IN PARALLEL
If the two bulbs are connected in parallel, voltage across them is same.
P1 R2 1
So,  (as P  ) If P1 > P2 then R1 < R2  P1' > P2'
P2 R1 R
So, in parallel high wattage bulb glows brighter than less wattage bulb.
V2 1 1 1
Now total power consumed is P  and   P = P1 + P2
R R R1 R2
Note:
1) In series grouping of bulbs, bulb of greater wattage gives less bright and will have less resistance
1
and potential difference across it, but same current Pa V a Ra
W
2) In series if one bulb gets fused the other will not work
3) In parallel grouping of bulbs, the bulb of greater wattage will give more bright light and will pass
greater current through it, but will have lesser resistance and same potential difference
1
Pa I a W a
R
4) In parallel if one bulb gets fused the other will work

Dr.K.K.R Gowtham 78 Current Electricity


Ex: Three equal resistors connected in series across a source of emf together dissipate 10 watt of
power. What would be the power dissipated if the same resistances are connected in parallel
acrsoss the same source of emf ?
Sol : The power consumed by a resistance R when connected across a source of emf V is given by
P  V 2 / R  Now, if r is the resistance of each resistor, the resistance of combination, in series will

1 1 1 1 r
be RS  r  r  r i.e.. RS  3r , while in parallel,    i.e.. RP  ,
Rp r r r 3
V2
So power consumption in series will be, PS  [ as RS =3r]
3r
V2 V 2  r
while in parallel will be, Pp   3  [ as R p  ]
(r / 3)  r  3
Pp V 2   3r 
i.e..  3   2   9 , And as here PS  10W PP  9  ( PS )  9 10  90W
Ps  r  V 
Ex:An electric tea kettle has two heating coils. When one of the coils is switched on, boiling begins in
6 min. When the other coil is switched on, the boiling begins in 8 min. In what time, will the
boiling begin if both coils are switched on simultaneously (i) in series and (ii) in parallel.
Sol : Let power of firsl coil is P1 and that of second coil is P2.
Let H is the amount of heat required to oil water.
Then H = P1t1 = p2t2 where t1 = 6 min, t2 = 8 min.
i) When the coils are connected in series :
P1 P2 H 1 1  t t 
P t H  H  2  t1  t2  6  8  14 min.
P1  P2 P  P1 P2  H H 
ii) When the coils are connected in parallel
H H H t1t2 68
P = P1 + P2 t      3.43 min .
P P1  P2 H  H t1  t2 6  8
t1 t2
Ex: If two bulbs of 25W and 100W rated at 220V are connected in series across 440V supply. Will
both the bulbs fuse ? If not which one !
V æ 2ö R25 W100 100
Sol : For an electrical appliance R = çç s ÷
÷
÷ = = = 4
çè W ø
÷ R100 W25 25
V ´ R25 4
If , R100 = R R25 = 4 R in series, V25 = = 440´ = 352 V
R25 + R100 5
= 440 - 352 = 88 V
From this it is clear that voltage across 100W bulb is lesser than specified (220V). While across
25W bulb is greater than (220V) so 25W will fuse.
If the bulbs are connected in parallel to 440V supply, both bulbs will be fused because, applied voltage
across each bulb is 440V greater than specified voltage (220V)
Ex. A current i varies with time in a coil of resistance R as shown in i
the graph. Find the (a) total charge flown(b) average current(c) heat
dissipated in the resistor. i0

Solution : (a) The total charge flow is q  idt T


t

Dr.K.K.R Gowtham 79 Current Electricity


1 iT
 Area under i  t graph  Ti0  0
2 2
q iT i
b) The average current  iav   0 / T  0
T 2 2
 t  T  t  5 2
 
c) The heat dissipated  P dt  i 2 R dt , where i  i0 1 
 T
  R 0 i0 1    dt  i0 RT
   T  6
MAXIMUM POWER TRANSFER THEOREM:
Power transfer to the load by the cell will be i

r
E2R  E  E R
P  I2 R  as I  
 R  r 2   r  R  
From this equation its is clear that P = (min) = 0, if R = 0 or  and so will be maximum when
dP d  E2R 
 0, i.e.,  0
dR dR   R  r 2 
 
d 
R R  r   0
2
i.e., [as E 0]
dR  

i.e., (R + r)-2 + R(- 2) (R + r)-3 = 0


i.e., (R +r)-3 [r – R) = 0
i.e., R = r as (R + r)-3  0 for finite R
i.e., power transfer to the load by a cell is maximum when R = r [and (P 1)max = (E2/4r)]
This statement in generalized form is called “maximum power transfer theorem”
Note:
1. Total power consumed in the circuit is E2/(R+r) and not E2 R/(R+r)2 and will be maximum (=E2
/r) when R = min =0 with I = E/r =max. If R= r, P= E2 /2r with I=E/2r.
2. It is common misconception that current in the circuit will be maximum when power consumed
by the total load is maximum. Actually currentI=E/(R+r) is maximum (=E/r) when R=min=0
with PL= (E2 /r2 )  0 =0=min. while power consumed by the total load PL =E2 R/(R+r)2 is
maximum = E2/4r when R=r and I=E/2r  max(= E/r)short circuit current.
3. The % of total energy which is used in driving the current (or) the efficiency of the cell is,
V IR R V R100
  and % =
E I (R  r ) R  r E R+ r
4. The % of initial energy lost at internal resistance
E- V r E- V r100
 and %=
E Rr E R+ r
R 2R
Ex: A battery if internal resistance 4 is connected to the network of
6R
resistances as shown. What must be the value of R so that maximum
power is delivered to the network? Find the maximum power? 2R 4R

Sol : i) According to maximum power transfer theorem Rext  Rint E 4

2
3R  6 R 4  E  E2
 4  R   2  ii) Pmax  i 2 Rext    4 
9R 2  44 16

Dr.K.K.R Gowtham 80 Current Electricity


LEVEL-A
SINGLE CORRECT CHOICE TYPE QUESTIONS:
Electric current ;Current density and Drift velocity
1. A charge in motion produces
1)electric field 2)magnetic field 3)Both electric & magnetic field 4)none
2. Electric current is a
1)scalar quantity 2)vector quantity
3)some times scalar and sometimes vector 4)number only
3. Which of the following is vector quantity
1) Current density 2) Current 3) Wattless current 4) Power
4. Charge carriers in
1)metals are electrons 2)semiconductor are ions
3)electrolytes are holes 4)none of the above
5. Through a semiconductor, an electric current is due to drift of
1) Free electrons 2) Free electrons and holes
3) Positive and negative ions 4) Protons
6. The area of current –time graph gives
1)current 2)charge 3)potential 4)resistance
7. If a steady current is passing through a conducting wire of non uniform cross section the net
charge passing through it per second is
1) more at large area of cross section 2) more at small area of cross section
3) same at any cross section 4) none of the above
8. If an electric current is passed through a nerve of a man, then man
1)Begins to laugh 2) Begins to weep 3) Is excited 4)Becomes insensitive to pain
ur
9. When electric field ( E ) is applied on the ends of a conductor ,the free electrons starts moving
in direction
1)similar to E 2)opposite to E 3)perpendicular to E 4)cannot be predicated
10. When a current flows through a conductor its temperature
1) May increase or decrease 2) Remains same 3) Decreases 4)Increases
11. A steady current i is flowing through a conductor of uniform cross-section. Any segment of the
conductor has
1) Zero charge 2) Only positive charge
3) Only negative charge 4) Charge proportional to current i
12. Given a current carrying wire of none uniform cross-section .Which of the following quantity or
quantities constant throughout the length of the wire?
1)current ,electric field and drift speed 2)drift speed only
3)current and drift speed 4)current only
13. A steady current flows in a metallic conductor of non-uniform cross-section. Which one of the
following is constant throughout the length of wire :
1) current only 2) current and drift speed
3) drift speed only 4) current, electric field and drift speed
14. When there is an electric current through a conducting wire along its length, then an electric
field must exist

Dr.K.K.R Gowtham 81 Current Electricity


1) Outside the wire but normal to it 2) Outside the wire but parallel to it
3) Inside the wire but parallel to it 4) Inside the wire but normal to it
15. Drift velocity v d varies with the intensity of electric field as per the relation
1
1) v d  E 2) v d  3) v d  constant 4) v d  E 2
E
16. We are able to obtain fairly large currents in a conductor because
1) The electron drift speed is usually very large
2) The number density of free electrons is very high and this can compensate for the low values
of the electron drift speed and the very small magnitude of the electron charge
3)The number density of free electrons as well as the electron drift speeds are very large and
these compensate for the very small magnitude of the electron charge
4)The very small magnitude of the electron charge has to be divided by the still smaller product
of the number density and drift speed to get the electric current
17. A current I is passing through a wire having two sections P and Q of uniform diameters d and d/2
respectively. If the mean drift velocity of electrons in sections P and Q is denoted by vP and vQ
respectively, then
1 1
1) vP = vQ 2) vP = vQ 3) vP = vQ 4) vP = 2 vQ
2 4
18. The drift velocity does not depend upon
1) Cross-section of the wire 2) Length of the wire
3) Number of free electrons 4) Magnitude of the current
19. The driff speed of an electron in a metal is of the order of ......
1) 10–13 cm/s 2) 10–3 mm/s 3) 10–3 m/s 4) 10–30 m/s
20. A wire has a non-uniform cross-section as shown in figure. A
steady current flows through it. The drift speed of electrons at
points P and q is vp and vQ.
1) vp = vQ 2) vp < vQ
3) vp > vQ 4)Data insufficient
21. A conductor of variable cross-section is connected across a
battery. Let us consider two cross-sections A and a. Let V and v
be drift speeds of electrons at those cross-sections respectively
then
1) AV > av 2) AV = av
3) Av = Av 4) AV < av
22. The area of cross section of a current carrying conductor is
Ao and Ao/4 at section (1) and (2) respectively. If v 1 , v  2
and E1, E2 be the drift velocity and electric field at sections
1 and 2 respectively then 1 2
1) v1 : v 2  1: 4 2) v1 : v2  4:1

3) E1 : E2 = 4 : 1 4) E1 : E2 = 1 : 4
23. Figure shows four situations in which positive and
negative charges move horizontally through a region
and gives the rate at which each charge moves. The

Dr.K.K.R Gowtham 82 Current Electricity


situations according to the effective current through
the region, greatest first:
1) A>B>C>D 2)(A=B)>C>D 3) A<B<C<D 4) (A=C)>D>B
24. The figure here shows conduction electrons moving leftward
through a wire. Select the quantity/(ies) which have
direction rightward?
1) i, vd 2) i,j 3) j, E, vd 4) i, j and E
Ohm’s law and resistance
25. Ohms law is applicable to
1) ohmic conductors only 2) non Ohmic conductors only
3) both ohmic & Non ohmic conductors 4) vaccum tubes only
26. The formula V = IR is applicable to
1) ohmic conductor only 2) non ohmic conductor only
3) both Ohmic & Non ohmic conductors 4) neither ohmic nor nonohmic conductor
27. Ohms law is not applicable for
1) insulators 2) semiconductors 3) arc lamp 4) all the above
28. For a metallic wire, the ratio V / i (V  the applied potential difference, i = current flowing) is
1) Independent of temperature 2) Increases as the temperature rises
3) Decreases as the temperature rises
4) Increases or decreases as temperature rises, depending upon the metal
29. All of the following statements are true except
1) Conductance is the reciprocal of resistance and is measured in Siemens
2) Ohm's law is not applicable at very low and very high temperatures
3) Ohm's law is applicable to semiconductors
4) Ohm's law is not applicable to electron tubes, discharge tubes and electrolytes
30. v–i graph for a metal at temperatures t1, t2, t3 are as shown. The highest temperature is
1) t1 2) t2 3) t3 4) All are equal
31. The electric intensity E , current density j and specific resistance k are related to each other
by the relation
1) E  j / k 2) E  jk 3) E  k / j 4) k  jE
32. The electric field E, current density J and conductivity  of a conductor are related as
1)   E / j 2)   j / E 3)   jE 4)   1 / jE

33. Express which of the following setups can be used to verify Ohm’s law
A

1) V 2) A
V

A V

V A
3) 4)

Dr.K.K.R Gowtham 83 Current Electricity


34. For making standard resistance, wire of following material is used
1) Nichrome 2) Copper 3) Silver 4) Manganin
35. Material used for heating coils is
1) Nichrome 2) copper 3) Silver 4) Manganin
36. S.I. Unit of electrical conductivity
1) siemen 2) siemen / metre 3) siemen metre 4) ohm metre
37. A piece of silver and another of silicon are heated from room temperature. The resistance of
1) each of them increases 2) each of them decreases
3) silver increases and silicon decreases 4) silver decreases and silicon increases
38. With the increase of temperature, the ratio of conductivity to resistivity of a metal conductor
1) decreases 2) remains same
3) increases 4) may increase or decrease
39. The conductivity of a super conductor, in the superconducting state is
1) zero 2) infinity 3) depends on temp 4) depends on free election
40. Choose the correct option using the following statements
1)coefficient of resistance of carbon is positive
2)at constant temperature, V–i graph of aluminium is a straight line
c)resistance of mercury increases with increasing temperature
d)with increase in temperature the conductivity of silicon decreases
1) a and b are correct 2) b and c are correct 3) c and d are correct 4) a and d are correct
41. For a chosen non-zero value of voltage, there can be more than one value of current in :
1) copper wire 2) thermistor 3) zener diode 4) manganin wire
42. The product of resistivity and conductivity of a cylindrical conductor depends on :
1) temperature 2) material 3) area of cross-section 4) none of these
43. A metallic resistor is connected across a battery. If the number of collisions of the free electrons
with the lattice is somehow decreased in the resistor(for example, by cooling it), the current will :
1) increase 2) decrease 3) remain constant 4) become zero
44. Fuse wire is a wire of :
1) low resistance and low melting point 2) low resistance and high melting point
3) high resistance and high melting point 4) high resistance and low melting point
45. When the length and area of cross-section both are doubled, then its resistance
1) Will become half 2) Will be doubled 3) Will remain the same 4) Will become four times
46. The resistivity of a wire
1) Increases with the length of the wire 2) Decreases with the area of cross-section
3) Decreases with the length and increases with the cross-section of wire
4) None of the above statement is correct
47. On increasing the temperature of a conductor, its resistance increases because
1) Relaxation time decreases 2) Mass of the electrons increases
3) Electron density decreases 4) None of the above
48. The specific resistance of all metals is most affected by
1) Temperature 2) Pressure
3) Degree of illumination 4) Applied magnetic field

Dr.K.K.R Gowtham 84 Current Electricity


49. The fact that the conductance of some metals rises to infinity at some temperature below a few
Kelvin is called
1) Thermal conductivity 2) Optical conductivity
3) Magnetic conductivity 4) Superconductivity
50. Two wires of the same material are given. The first wire is twice as long as the second and has
twice the diameter of the second. The resistance of the first will be
1) Twice of the second 2) Half of the second
3) Equal to the second 4) Four times of the second
51. The relaxation time in conductors
1) Increases with the increase of temperature 2) Decreases with the increase of temperature
3) It does not depend on temperature 4) All of sudden changes at 400 K
52. Which of the following statement is correct
1) Liquids obey fully the ohm's law 2) Liquids obey partially the ohm's law
3) There is no relation between current and p.d. for liquids 4) None of the above
53. A potential difference of V is applied at the ends of a copper wire of length l and diameter d. On
doubling only d, drift velocity
1) Becomes two times 2) Becomes half 3) Does not change 4) Becomes one fourth
54. The resistance of a discharge tube is
1) Ohmic 2) Non-ohmic 3) Both (1) and (2) 4) Zero
55. The resistivity of a wire depends on its
1) Length 2) Area of cross-section 3) Shape 4) Material
56. The resistivity of alloys  Ralloy ; the resistivity of constituent metals Rmetal . Then, usually
1) Ralloy  Rmetal 2) Ralloy  Rmetal
3) There is no simple relation between Ralloy and Rmetal 4) Ralloy  Rmetal
57.  1 and  2 are the electrical conductivities of Ge and Na respectively. If these substances are
heated, then
1) Both  1 and  2 increase 2)  1 increases and  2 decreases
3)  1 decreases and  2 increases 4) Both  1 and  2 decrease
58. A uniform wire of resistance R is uniformly compressed along its length, until its radius
becomes n times the original radius. Now resistance of the wire becomes
R R R
1) 2) 3) 4) nR
n4 n2 n
59. By increasing the temperature, the specific resistance of a conductor and a semiconductor
1) Increases for both 2) Decreases for both 3) Increases, decreases 4)Decreases, increases
60. The lead wires should have
1) Larger diameter and low resistance 2) Smaller diameter and high resistance
3) Smaller diameter and low resistance 4) Larger diameter and high resistance
61. The alloys constantan and manganin are used to make standard resistance due to they have
1) Low resistivity 2) High resistivity
3) Low temperature coefficient of resistance 4) Both (2) and (3)
62. When a potential difference is applied across the ends of a linear metallic conductor
1) The free electrons are accelerated continuously from the lower potential end to the higher
potential end of the conductor

Dr.K.K.R Gowtham 85 Current Electricity


2) The free electrons are accelerated continuously from the higher potential end to the lower
potential end of the conductor
3) The free electrons acquire a constant drift velocity from the lower potential end to the higher
potential end of the conductor
4) The free electrons are set in motion from their position of rest
63. The electric resistance of a certain wire of iron is R. If its length and radius are both doubled,
then
1) The resistance will be doubled and the specific resistance will be halved
2) The resistance will be halved and the specific resistance will remain unchanged
3) The resistance will be halved and the specific resistance will be doubled
4)The resistance and the specific resistance, will both remain unchanged

E.m.f; Terminal potential difference and Internal resistance


64. Choose the correct statement
1) The difference of potential between the terminals of a cell in closed circuit is called emf of the cell
2) electromotive force and accelerating force have the same dimensions
3) The internal resistance of an ideal cell is infinity
4)The difference between the emf of a cell and potential difference across the ends of the cell is
called ‘lost volt’
65. The internal resistance of an ideal cell is
1) low 2) zero 3) high 4) infinity
66. For a cell, which is discharging, in a circuit (V is T.P.D and E is emf) then
1) V = E 2) V < E 3) V > E 4) E = 2V
67. For a cell which is charging in a circuit
1) V = E 2) V < E 3) V > E 4) E > 2V
68. If two identical cells of emf E & internal resistance r are connected in parallel`, the terminal
voltage of battery is
E E
1) E 2) 3) 2E 4)
2 r
69. The internal resistance is
1) Large for primary cells 2) Small for secondary cells
3) Both 1 & 2 4) None
70. The internal resistance of a cell depends on
1) Concentration of electrolyte 2) Distance between the electrodes
3) Area of electrode 4) All of the above
71. The direction of current in a cell is from
1) (–) ve pole to (+) ve pole during discharging inside the cell
2) (+) ve pole to (–) ve pole during charging
3) Always (–) ve pole to (+) ve pole inside 4) Both 1 & 2
72. To supply maximum current, cells should be arranged in
1) Series 2) Parallel
3) Mixed grouping 4) Depends on the internal and external resistance
73. To reduce the electrolytic polarization, the depolarizer used is
1) Copper sulphate solution 2) Manganese dioxide

Dr.K.K.R Gowtham 86 Current Electricity


3) Dilute Sulphuric acid 4) Zinc
74. When an electric cell drives current through load resistance, its Back emf,
1) Supports the original emf 2) Opposes the original emf
3) Supports if internal resistance is low 4) opposes if load resistance large
75. When cells are connected in parallel, then
1) Current decreases 2) Current increases 3) Emf increases 4) Emf decreases
76. The terminal pd of a cell is equal to its emf if
1) External resistance is infinity 2) Internal resistance is zero
3) Both 1 & 2 4) None
77. The electric power transferred by a cell to an external resistance is maximum when the external
resistance is equal to – (r - internal resistance)
1) r/2 2) 2r 3) r 4) r2
78. A cell of e.m.f. E is connected with an external resistance R , then p.d. across cell is V . The
internal resistance of cell will be
( E  V )R ( E  V )R R  R 
1) 2) 3)   4)  
E V r  Rr
79. The terminal potential difference of a cell is greater than its e.m.f. when it is
1) Being discharged 2) In open circuit
3) Being charged 4) Being either charged or discharged
80. When cells are connected in parallel, then
1) The current decreases 2) The current increases
3) The e.m.f. increases 4) The e.m.f. decreases
81. n identical cells each of e.m.f. E and internal resistance r are connected in series. An
external resistance R is connected in series to this combination. The current through R is
nE nE E nE
1) 2) 3) 4)
R  nr nR  r R  nr Rr
82. A cell of internal resistance r is connected to an external resistance R. The current will be
maximum in R, if
1) R  r 2) R  r 3) R  r 4) R  r / 2
83. To get the maximum current from a parallel combination of n identical cells each of internal
resistance r in an external resistance R, when
1) R  r 2) R  r 3) R  r 4) None of these
84. A battery of e.m.f. E and internal resistance r is connected to a variable E r
resistor R as shown here. Which one of the following is true
1) Potential difference across the terminals of the battery is maximum
when R = r R

2) Power delivered to the resistor is maximum when R = r


3) Current in the circuit is maximum when R = r
4) Current in the circuit is maximum when R  r
85. Two non-ideal identical batteries are connected in parallel. Consider the following statements
(i) The equivalent e.m.f. is smaller than either of the two e.m.f.s
(ii) The equivalent internal resistance is smaller than either of the two internal resistances
1) Both (i) and (ii) are correct 2) (i) is correct but (ii) is wrong

Dr.K.K.R Gowtham 87 Current Electricity


3) (ii) is correct but (i) is wrong 4) Both (i) and (ii) are wrong
86. Emf is most closely related to
1) Mechanical force 2) Potential difference 3) Electric field 4) Magnetic field
87. Electromotive force is the force, which is able to maintain a constant
1) Current 2) Resistance 3) Power 4) Potential difference
88. The internal resistance of a cell is the resistance of
1) Electrodes of the cell 2) Vessel of the cell
3) Electrolyte used in the cell 4) Material used in the cell
89. Current provided by a battery is maximum when
1) Internal resistance equal to external resistance
2) Internal resistance is greater than external resistance
3) Internal resistance is less than external resistance 4) None of these
90. To draw maximum current from a combination of cells, how should the cells be grouped
1) Series 2) Parallel 3) Mixed
4)Depends upon the relative values of external and internal resistance

91. Two sources of equal emf are connected to an external resistance R. The internal resistances of
the two sources are R 1 and R2 (R2  R1 ) . If the potential difference across the source having
internal resistance R 2 is zero, then
1) R  R1 R 2 /(R1  R 2 ) 2) R  R1 R 2 /(R 2  R1 )
3) R  R 2  (R1  R 2 ) /(R 2  R1 ) 4) R  R2  R1

Kirchhoff’s laws
92. Kirchhoff's first law i.e. i  0 at a junction is based on the law of conservation of
1) Charge 2) Energy 3) Momentum 4)Angular momentum
93. Kirchhoff's second law is based on the law of conservation of
1) Charge 2) Energy 3) Momentum 4) Sum of mass and energy
94. Kirchoff’s I law and II law of current, proves the
1) Conservation of charge and energy 2) Conservation of current and energy
3) Conservation of mass and charge 4)None of these
95. The figure below shows currents in a part of electric circuit. The current i is
1amp
2amp
1.3amp

2amp

i
1) 1.7 amp 2) 3.7 amp 3) 1.3 amp 4) 1 amp
15A
96. The figure shows a network of currents. The magnitude 3A
of currents is shown here. The current i will be
1) 3 A 2) 13 A 8A
3) 23 A 4) – 3 A
i
5A

Dr.K.K.R Gowtham 88 Current Electricity


97. The current in the arm CD of the circuit will be
1) i1  i2 B

2) i2  i3 i1
i2
A
3) i1  i3 O i3

4) i1  i2  i3 C
Wheatstones bridge D

98. In a balanced Wheatstone’s network, the resistances in the arms Q and S are interchanged. As a
result of this :
1) galvanometer and the cell must be interchanged to balance
2) galvanometer shows zero deflection
3) network is not balanced 4) network is still balanced
99. If galvanometer and battery are interchanged in balanced wheatstone bridge, then
1) the battery discharges 2) the bridge still balances
3) the balance point is changed 4) the galvanometer is damaged due to flow of high current
100. Wheatstone bridge is most sensitive when the arms ratio is
1) equal to one 2) less than one 3) more than one 4) zero
101. In a balanced Wheatstone’s network, the resistances in the arms Q and S are interchanged. As a
result of this
1) Network is not balanced 2) Network is still balanced
3) Galvanometer shows zero deflection 4) Galvanometer and
Meter Bridge
102. In metrebridge experiment, the known and unknown resistances in the two gaps are interchanged.
The error so removed is
1) end correction 2) index error 3) due to temperature effect 4) random error
103. The purpose of high resistance in meter bridge circuit its
1)to get accurate balance point 2)to save the current
3)to protect the meter from damage 4)to change the direction of current
104. In a metrebridge, metal wire is connected in the left gap, standard resistance is connected in the
right gap and balance point is found. If the metal wire in the left gap is heated, the balance point
1) shifts towards left 2) shifts towards right
3) does not shift 4) depends on thickness of metal wire
105. Metal wire is connected in the left gap, semi conductor is connected in the right gap of meter
bridge and balancing point is found. Both are heated so that change of resistances in them are
same. Then the balancing point
1) will not shift 2) shifts towards left
3) shifts towards right 4) depends on rise of temperature
106. Metal wire is connected in the left gap, Germanium connected in the right gap of metrebridge
and balancing point is found at 50 cm. Metal wire is heated and Germanium is cooled so that
variations of resistances in them are equal. Then the balancing point
1) will not shift 2) shifts towards left
3) shifts towards right 4) depends on change of temperature
107. In the measurement of resistance by a meter bridge the current is necessarily reversed through the
bridge wire to eliminate

Dr.K.K.R Gowtham 89 Current Electricity


1)End error 2)Index error
3)Error due to the electric effect 4)Random errors
Heating Effects:
108. A resistor of resistance R is connected to a cell of internal resistane 5 . The value of R is varied
from 1 to 5 . The power consumed by R :
1) increases continuously 2) decreases continuously
3) first decreases then increases 4) first increases then decreases.
109. Power generated across a uniform wire connected across a supply is H. If the wire is cut into n
equal parts and all the parts are connected in parallel across the same supply, the total power
generated in the wire is
H H
1) 2) n2H 3) nH 4)
n2 n
110. A constant voltage is applied between the two ends of a uniform metallic wire. Some heat is
developed in it. The heat developed is doubled if
1) both the length and the radius of the wire are halved
2) both the length and the radius of the wire are doubled
3) the radius of the wire is doubled 4) the length of the wire is doubled
111. When electric bulbs of same power, but different marked voltage are connected in series across
the power line, their brightness will be
1) proportional to their marked voltage 2) inversely proportional to their marked voltage
3) proportional to the square of their marked voltage
4) inversely proportional to the square of their marked voltage
112. A wire when connected to 220 V mains supply has power dissipation P1. Now the wire is cut
into two equal pieces which are connected in parallel to the same supply. Power dissipation in
this case is P2. Then P2 : P1 is :
1) 1 2) 4 3) 2 4) 3
113. Rate of dissipation of Joule.s heat in resistance per unit volume is (symbols have usual meaning)
1) E 2) J 3) JE 4) None
114. If the length of the filament of a heater is reduced by 10%, the power of the heater will
1) increase by about 9% 2) increase by about 11%
3) increase by about 19% 4) decrease by about 10%
115. If X, Y and Z in figure are identical lamps, which of the following
changes to the brightnesses of the lamps occur when switch S is
closed ?
1) X stays the same, Y decreases
2) X increases, Y decreases
3) X increases, Y stays the same
4) X decreases, Y increases
LEVEL-B
SINGLE CORRECT CHOICE TYPE QUESTIONS:
I. ELECTRIC CURRENT:
1. 62 . 5  10 18
electrons per second are flowing through a wire of area of cross-section 0.1 m 2 , the
value of current flowing will be
1)1 A 2)0.1 A 3)10 A 4)0.11 A

Dr.K.K.R Gowtham 90 Current Electricity


2. Current of 4.8 amperes is flowing through a conductor. The number of electrons per second
will be
1) 3  10 19 2) 7.68  10 21 3) 7.68  10 20 4) 3  10 20

3. In an inert gas discharge tube positive ions (e+) move to the right through a cross-section of the
tube each second, while 2.9 1018 negative ions (e–) move to the left in the same time. The
magnitude of current is :
1) 2.384 A 2) 2.384 10–3 A 3) 2.384 10–6 A 4) 0.66A
4. In a region 10 – particles and 10 protons move to the left while 10 electrons move to the
19 19 19

right per second. The current is


1) 3.2 amp towards left 2) 6.4 amp towards left
3) 9.6 amp towards left 4) 6.4 amp towards right
-15
5. In hydrogen atom, the electron 6.6x10 revolutions/sec around the nucleus in an orbit of
radius0.5A0 . The equivalent current is nearly :
1) 10–2 A 2) 10–3 A 3) 10–4 A 4) 10–5 A
6. If the electron in H2 atom makes 6.25x 1015 rev/sec, the current is

1) A 2) 10–1A 3) 1mA 4) 1A
7. An –particle (mass=4mp, charge 2e) is revolving in a circular orbit of radius 1 Aº, with
speed 3.14 106 m / s . The equivalent current is :
1) 1.6 mA 2) 3.2 mA 3) 6.4 mA 4) 0.8 mA
8. The amount of charge passed in time t through a cross-section of a wire is Q{t) = At2 + Bt + C.
If the numerical values of A, B and C are 5, 3 and 1 respectively in SI units, find the value of the
current at t = 5 s.
1)23A 2)53A 3)43A 4)20A
9. Flow of charge through a cross-section is given as Q=4t2+2t  0  t  10 , Then the current
through the crosection at 5s and the average current for (0 -10sec) are
1)24A&42A 2)42A&42A 3)42A&23A 4)none of these
10. The current through a wire depends on time as i = i0 +  t,
where i0 = 10 A and  = 4 A/s, then the charge crossed through a section of the wire in 10
seconds is
1)100C 2)200C 3)300C 4)400C
II. CURRENT DENSITY:
1. An electron beam has an aperture 1mm2 . A total of 6  1016 electrons go through any
perpendicular cross-section per second ,then the current density in the beam is
1)9.6  10-3A/m2 2) 9.6  103A/m2 3) 9.6  10-6A/m2 4) 9.6  106A/m2
2. In a long wire of round cross-section of side length L current density varies with distance from
one edge of cross-section x as J  Ae Bx A/m2 where A and B are positive constants.The current
flowing in wire is
AL BL AL BL AL  BL AL  BL
1) (e  1) 2) (e  1) 3) (e  1) 4) (e  1)
B B B B

Dr.K.K.R Gowtham 91 Current Electricity


3. In a long wire of round cross-section of radius R current density varies with the radial distance
from axis of wire x as J=Cx2 A/m2 where c is a positive constant .The total current flowing in
wire is
 CR 4  CR 4  CR 2
1) 2) 3) 4)  CR 4
2 3 2
4. The current density across a solid cylindrical conductor of radius R varies according to the
x
relation J  J o (1  ) where J o a constant vector along the radial direction and x is distance
R
from axis .The current through cylinder’s cross-section is
J0   R2 J0   R2 J0   R2
1) J 0   R 2
2) 3) 4)
3 6 2
III. DRIFT SPEED
1. There is a current of 4.0 A in a wire of cross-sectional area 10–6 m2. If the free electron density
in wire is 1028 m–3, the drift velocity is :
1) 2.5 mm/s 2) 25 mm/s 3) 2.5 103 m/s 4) 1.25 mm/s
2. A current of 2.0 A exists in a wire of cross-sectional area 1.0 mm2. If each cubic metre of the
wire contains 6.0x1028 free electrons, find the drift speed.
1) 2.1104 m / s 2) 3.1104 m / s 3) 3.1104 m / s 4) 3.1105 m / s
3. The drift speed of the electron when 1A of current exists in a copper wire of cross section 2mm 2
is (The number of free electron in 1cm2 of copper is 8.5  1022 )
1)0.036mms-1 2) 0.36mms-1 3) 36mms-1 4) 0.0036mms-1
4. A current of 1.0 A exists in a copper wire of cross-section 1.0 mm2. Assuming one free electrons
in the wire. The density of copper is 9000 kg/m3.
1) 0.074mm/s 2) 0.74mm/s 3) 074 mm/s 4) 024 mm/s
5. 2
Consider a wire of length 4m and cross-sectional area 1 mm carrying a current of 2A. If each
cubic metre of the material contains 1029 free electrons, find the average time taken by an
electron to cross the length of the wire.
1) 3.2  10 4 s 2) 3.2 104 s 3) 1.2  104 s 4) 1.2 104 s
6. In a wire of circular cross-section with radius r free electrons travel with a drift velocity V when
a current I flows through the wire. What is the current in another wire of half the radius and of
the same material when the drift velocity is 2 V :
1) 2 I 2) I 3) I/2 4) I/4
7. A uniform copper wire carries a current i amperes and has  carriers per metre3. The length of the
wire is l metres and its cross-section area is s metre2. If the charge on a carrier is q coulombs, the
drift velocity in ms -1 is given by
1) i/ l sq 2) i/psq 3) psq/i 4) i/ps l q
8. A current I flows through a uniform wire of diameter d when the mean electron drift velocity is
V. the same current will flow through a wire of diameter d/2 made of the same material if the
mean drift velocity of the electron is
1) v/4 2) v/2 3) 2v 4) 4v
9. An insulating pipe of cross-section area ‘A’. contains an electrolyte which has two types of ions
 their charges being - e and +2e. A potential difference applied between the ends of the pipe
result in the drifting of the two types of ions, having drift speed = v (- ve ion) and v/4 (+ ve ion).
Both ions have the same number per unit volume = n. The current flowing through the pipe is

Dr.K.K.R Gowtham 92 Current Electricity


1) nev A/2 2) nev A/4 3) 5nev A/2 4) 3nev A/2
10. Mark the correct option regarding drift speed
1)Drift speed is of the order of 10-4m/s 2)Drift speed increases with decrease in temperature
3)Drift speed is depending on applied electric field 4)All the above
IV. OHM’S LAW AND RESISTANCE:
1. An incandescent lamp of resistance 80 draws a current of 0.75A. Find the line voltage
1) 60 V 2) 30V 3) 40V 4) 10V
2. A bulb of resistance 400 is connected to 200V mains. Calculate the magnitude of current
1) 1A 2) 0.5A 3) 0.2A 4) 5A
3. An electric heater draws a current of 5A, when connected to 220V mains. Calculate the
resistance of its filament.
1) 44  2) 32  3) 22 4) 11
4. A conductor of resistance R is connected to a battery. A charge of 36 coulomb passed through
the conductor per 2 min. Then p.d. across the end of the wire is 4V. Then the value of ‘R’ will
be
1) 10.3  2) 11.3 3) 12.3 4) 13.3
5. What should be the length of nichrome wire of resistance 45 , if the length of similar wire is
60cm and resistance 2.5?
1) 108 cm 2) 100 cm 3) 120 cm 4) 106 cm
6. Resistance of conductor of length 75cm is 3.25  . Calculate the length of a similar conductor,
whose resistance is 16.25.
1) 475 cm 2) 375 cm 3) 175cm 4) 75 cm
7. A conductor of length 85cm has a resistance of 3.75 . Calculate the resistance of a similar
conductor of length 510 cm.
1) 2.2  2) 12.2  3) 22.5  4) 12.5 
8. A nicrhome wire has a resistance of 5. Find the resistance of another nichrome wire, whose
length is four times and area of cross-section three times the first wire.
1) 1.1     
9. A resistance wire made from German silver has a resistance of 4.25 . Calculate the resistance
of another wire, made from same material, such that its length increases by 4 times and area of
cross-section decreases by three times.
1) 31        
10. A nichrome wire of length l and area of cross-section a/4 has a resistance R. Another nichrome
wire of length 3l and area of cross-section a/2 has a resistance of R1. Find the ratio of R1:R.
1) 1:2 2) 2:1 3) 3:2 4) 2:3
11. The resistance of a wire of length 80 cm and of uniform area of cross-section 0.025 cm2, is found
to be 1.50 ohm. Calculate sp. Resistance of wire.
1) 0.000469  - cm 2) 0.000269  - cm 3) 0.000369  - cm 4) 0.000569  - cm
12. A wire of resistance 4.5  and length 150 cm, has area of cross-section of 0.04 cm2. calculate
sp. resistance of wire
1) 0.0052-cm 2) 0.0012-cm 3) 0.0032-cm 4) 0.0042-cm
13. A wire of length 40cm and area of cross-section 0.1mm2 has a resistance of 0.8 . Calculate sp.
Resistance of wire.

Dr.K.K.R Gowtham 93 Current Electricity


1) 0.00002 -cm 2) 0.0002 -cm 3) 0.002 -cm 4) 002 -cm
14. Two wires made of same material have their electrical resistance in the ratio 1:4. If their lengths
are in the ratio of 1:2, the ratio of their masses is
1) 1:1 2) 1:2 3) 1:4 4) 2:1
15. Following figure shows cross-sections through three 3a
2a
long conductors of the same length and material, a
with square cross-section of edge lengths as shown.
Conductor B will fit snugly within conductor A, and A B C
conductor C will fit snugly within conductor B.
Relationship between their end to end resistance is
1)RA = RB = RC 2)RA > RB > RC
3)RA < RB < R 4)Information is not sufficient

16. A conductor with rectangular cross section has


dimensions (a × 2a × 4a) as shown in figure.
Resistance across AB is x, across CD is y and across
EF is z. Then

1) x = y = z 2) x > y > z
3) y > z > x 4) x > z > y
17. A rectangular carbon block has dimensions
1.0cm×1.0cm×50 cm. Resistances are measured,
first across two square ends and then across two
rectangular ends, respectively. If resistivity of
carbon is 3.5×10.5-m, then values of measured
resistances respectively are :
35 15
1) 102 , 7 105  2) 7 105 , 102 
2 2
35 15
3) 104 , 7 107  4) , 7 10 2 
2 2
18. In figure shows a rectangular block with dimensions x, 2x and
C
4x. Electrical contacts can be made to the block between 4x
opposite pairs of faces (for example, between the faces B B
labelled A-A, B-B and C-C). Between which two faces would
the maximum electrical resistance be obtained (A-A : Top and C x
2x
bottom faces, B-B : Left and right faces, C-C : Front and rear
faces)
1) A-A 2) B-B 3) C-C 4) Same for all three pairs
19. The potential difference across a wire of 10–3 cm2 cross-sectional area and 50 cm length is 2
volt, when a current of 0.25 A exists in the wire. The conductivity of the material is :
1) 2  5 106 mho/metre 2) 6  25 105 mho/metre
3) 2  5 109 mho/metre 4) 6  25 1010 mho/metre
20. When a metal pipe of 6.28m length has inner diameter of 20cm and wall thickness 2cm is
connected to a 3V battery carries a current of 6A. the resistivity of material of pipe in m is
1) 1.1x10-3 2) 2.2x10-3 3) 4.4x10-3 4) 1.76x10-3

Dr.K.K.R Gowtham 94 Current Electricity


21. Masses of three wires of same material are in the ratio of 3:2:1 and their areas of cross section in
the ratio of 1:2:3. Electrical resistances of these wires are in the ratio of
1) 1:1:1 2) 3:4:3 3) 27:8:1 4) 54:9:2
22. Find the resistance of a copper coil of total wire-length 10m and area of cross-section 1.0mm2.
What would be the resistance of a similar coil of aluminium? The resistivity of copper
= 1.7  108   m and that of aluminium = 2.6 108   m
  0.26    
23. Find the resistance of a hollow cylindrical conductor of length 1.0m and inner and outer radii 1.0
mm and 2.0 mm respectively. The resistivity of the material is
1) 1.0  10 3  2) 2.1 10 3  3) 3.1 10 3  4) 4.1 10 3 
24. A copper rod of length 20cm and cross-sectional area 2mm2 copper
is joined with a similar aluminium rod as shown in figure.
Find the resistance of the combination between the ends.
aluminium
Resistivity of copper = 1.7 108   m and that of aluminium
= 2.6  108   m
1) 1.0m  2) 2.0m  3) 3.0m  4) 0.4m 
25. A uniform wire of resistance 100 is melted and recast in a wire of length double that of the
original. What would be the resistance of the wire?
1) 200       

26. What length of a copper wire of cross-section area 0.01 mm2 will be needed to prepare a
resistance of 1 k ? Resistivity of copper = 1.7  108   m
1) 0.6km 2) 0.4 km 3) 0.3km 4) 0.2 km
27. A carbon filament has resistance of 120at 0ºC. What must be the resistance of a copper filament
connected in series with carbon so that the combination has same resistance at all temperatures.
 carbon = 7x10–4/ºC  copper = 4x10–3/ºC
1) 120 2) 21 3) 60 4) 210
28. The temperature coefficient of resistivity of a material is 0.0004/K. When the temperature of the
material is increased by 50°C the resistivity increases by 2 x 10–8 ohm meter. The initial
resistivity of the material in ohm meter is
1) 50 x 10–8 2) 90 x 10–8 3) 100 x 10–8 4) 200 x 10–8
29. The resistance of an iron wire 10 ohm and temperature coefficient of resistivity is 5 103 / 0 C .
At 20ºC it carries 30 mA of current. Keeping constant potential difference between its ends, the
temperature of the wire is raised to 120ºC. The current in milliamperes that flows in the
wire now is
1) 20 2) 15 3) 10 4) 40
30. The resistances of an iron wire and a copper wire at 20 0C are 3.9 and 4.1 resistively. At
what temperature will the resistances be equal? Temperature coefficient of resistances be equal?
Temperature coefficient of resistivity for iron is 5.0 103 K 1 and for copper it is
4.0  103 K 1 . Neglect any thermal expansion.
1) 84.50C 2) 64..50C 3) 54.50C 4) 44.50C

Dr.K.K.R Gowtham 95 Current Electricity


V. GROUPING OF RESISTANCES:
1. Calculate the equivalent resistance (i) between points A and B (ii) between points A and C.

1) (i) 1 (ii) 2 2) (i) 2 (ii) 3 3) (i) 3 (ii) 1 4) (i) 3 (ii) 3

2. In figure, calculate equivalent resistance between points A and B, B and C, A and C.

1) 743 2) 347 3) 124 4) 247

3. Calculate the equivalent resistance between points A and B, as shown in figure.

1) 9 2) 6  3) 12  4) 3

4. In figure, calculate equivalent resistances between points P and Q, points R and S,


points A and B

1) 442 2) 4127 3) 4122 4) 447

5. The equivalent resistance of circuit diagram 9.34 is 4. Calculate the resistance of x.

1) 1.5 2) 2.5  3) 3.5  4) 5.5


6. Calculate the equivalent resistance between points B and E, A and F

1) 44 2) 28 3) 84 4) 47

Dr.K.K.R Gowtham 96 Current Electricity


7. Equivalent resistance between A and B will be 3 3
1) 2 ohm 2) 18 ohm
3)6 ohm 4) 3.6 ohm
3 3
3 3

A B
3 3
8. The equivalent resistance between A and B if each resistance is equal to 20 ohm is
1) 10 ohm 2) 32 ohm
3) 20 ohm 4) 26.67 ohm

9. If the equivalent resistance between A and B is 6 ohm, the value of r is


1) 18 ohm 2) 12 ohm
3) 10 ohm 4) 6 ohm

10. Six equal resistances are connected between points P, Q and R as shown in the figure. Then the
net resistance will be maximum between P

1) P and Q
2) Q and R
3) P and R
4) Any two points Q R
R
11. In the figure shown, the total resistance between A and B is
1) 12  2) 4 
3) 6  4) 8 

2 C 1 1 1 1 1
12. Effective resistance between A & B in the given circuit is A
1) 1.5  2) 7.5  8 8 4
3) 47  4) 12 
B 2 D 1 1 1 1 1

13. The effective resistance between the points A and B in the figure is D

1) 5  2) 2  3 3

3) 3  4) 4  6
A C

3 3

Dr.K.K.R Gowtham 97 Current Electricity


14. The effective resistance between A and B in the given circuit is
1) 2 2) 3
3) 9 4) 6

15. The equivalent resistance of the arrangement of resistances shown in adjoining figure between
8
the points A and B is
16 20
1) 6 ohm
16
2) 8 ohm
A B
3) 16 ohm 9
6
4) 24 ohm
18

16. Equivalent resistance between A & B in the given circuit is

1) 4  2) 8 3) 6 4) 3
17. For what value of R the net resistance of the circuit will be 18 ohms R
1) 8 
10 10
2) 10 
10
3) 16 
4) 24  10 10 10
A B
18. In the circuit shown here, what is the value of the unknown resistor R so that the total resistance
of the circuit between points P and Q is also equal to R 10

1) 3 ohms 2) 39 ohms P
3
Q
3 R
3) 69 ohms 4)10 ohms
A B
19. What is the equivalent resistance between A and B in
R R
the figure below if R  3  R
1) 9  2) 12  3) 15  4) None of these
R R
2
20. The equivalent resistance of the circuit shown in the figure is 2 2
1) 8  2) 6  2
3) 5  4) 4 

21. In the given figure, the equivalent resistance between the points A and B is R2 = 4 
1) 8  2) 6 
R1 = 2 R4 = 2 
3) 4  4) 2  A B
R3 = 4 
22. If all the resistors shown have the value 2 ohm A B
each, the equivalent resistance over AB is
2 2
1) 2 ohm 2) 4 ohm 3) 1 ohm 4) 2 ohm
3 3

Dr.K.K.R Gowtham 98 Current Electricity


23. The equivalent resistance between the points P and Q of the circuit given is
R R
1) 2) R R R
4 3 P Q
3) 4 R 4)2 R

24. Equivalent resistance between the points A and B is (in )


1 1
1) 2) 1
5 4 A 1 1 1 1 1 B
1 1
3) 2 4) 3
3 2
25. Equivalent resistance across A & B is
1) 4 2) 2
3) 6 4) 8/3

26. A wire of resistance 18 ohm is bent to form equilateral triangle. The resistance across any two
vertices of the triangle is(in ohm)
1) 12 2) 6 3) 4 4) 9
27. A wire of resistance 12 ohm is bent in the form of a circle. The effective resistance between the
ends of any diameter(in ohm)
1) 3 2) 6 3) 4 4) 24
28. Four conductors of same resistance connected to form a square. If the resistance between
diagonally opposite corners is 8 ohm, the resistance between any two adjacent corners is
1) 32 ohm 2) 8 ohm 3) 1/6 ohm 4) 6 ohm
29. Six conductors each of resistance R are connected to form a hexagon. The resistance between
the two corners of any side is
1) 6R 2) R/6 3) 2R/3 4) 5R/6
30. A piece of wire of resistance is bent through 180º at its mid-point and the two halves are twisted
together. Then the resistance is :
1) 8  2) 1  3) 2  4) 5 
31. The total current supplied to the circuit by the battery is
1) 1A 2) 6A
3) 4A 4) 2A

32. The current in the adjoining circuit will be i


1 1
1) ampere 2) ampere 30
45 15 2V 30
1 1
3) ampere 4) ampere 30
10 5
33. The reading of the ammeter as per figure shown is 2
1 3
1) A 2) A 2 2V
8 4 A
1 2
3) A 4) 2 A
2 2

Dr.K.K.R Gowtham 99 Current Electricity


34. Resistances of 6 ohm each are connected in the manner shown in adjoining figure. With the
current 0.5 ampere as shown in figure, the potential difference VP  VQ is
6 6 6
1) 3.6 V 2) 6.0 V
3) 3.0 V 4) 7.2 V P 6 Q
0.5 A
6 6
35. Four resistances are connected in a circuit in the given figure. The electric current
4 6
flowing through 4 ohm and 6 ohm resistance is respectively
1) 2 amp and 4 amp 2) 1 amp and 2 amp 4 6
3) 1 amp and 1 amp 4) 2 amp and 2 amp

20V
36. The current from the battery in circuit diagram shown is
2 A 7
1) 1 A
15V
2) 2 A
0.5 6 1
3) 1.5 A
4) 3 A 8 B 10
37. In the given figure, when key K is opened, the reading of the ammeter A will be
10V
1) 50 A
+ –
2) 2 A
5
3) 0.5 A E A D
10
4) A 4 A
9
B C
K
6
38. In the figure given below, the current passing through 6  resistor is
1) 0.40 ampere 1.2 A
2) 0.48 ampere
4
3) 0.72 ampere
4) 0.80 ampere
i R2 = 2
39. What is the current (i) in the circuit as shown in figure
1) 2 A
R3 = 2

3V
2) 1.2 A R1 = 2

3) 1 A
4) 0.5 A R4 = 2

40. In the figure, current through the 3  resistor is 0.8 ampere, then potential drop through 4 
3
resistor is
1) 9.6 V
4
2) 2.6 V 6
3) 4.8 V
3
4) 1.2 V + –
4
41. The potential drop across the 3 resistor is
6
1) 1 V 2) 1.5 V
3) 2 V 4) 3 V
3V

Dr.K.K.R Gowtham 100 Current Electricity


42. What will be the resistance of a semicircle shown in fig. between its two
end faces. Given that radial thickness =3cm, axial thickness=4cm, inner
radius =6cm and resistivity=
1) 24.15  106  2) 7.85 107 
3) 7.85 106  4) 7.85 105 
43. A wire of resistor R is bent into a circular ring of radius r. Equivalent resistance between two
points X and Y on its circumference, when angle XOY is , can be given by
X
Ra R
1) (2p - a ) 2) (2p - a )
4p 2 2p W  O Z
4p
3) R (2 – ) 4) (2p - a )
Ra Y

44. A uniform wire of resistance 36 is bent in the form of a circle.


The effective resistance across the points A and B is
1) 5 2) 6
3) 7.2 4) 30
45. The effective resistance of a number of resistors in parallel is x.
When one of the resistors is burnt, the effective resistance is y. Resistance of
the burnt part is
1) (xy/x+y) 2) x - y 3) ( x+y/xy) 4) (xy/y-x)
46. Two wires of the same dimensions but resistivities 1 and  2 are connected in series. The
equivalent resistivity of the combination is
1   2
1) 1   2 2) 3) 1  2 4) 2(1   2 )
2
47. You are given several identical resistors each of value 10 and each capable of carrying a
maximum current of 1A. It is required to make a suitable combination of these resistances to
produce a resistance of 5 which can carry a current of 4A. The minimum number of resistors
required for this job is
1) 4 2) 8 3) 10 4) 20
48. Find the equivalent resistance between the points a and b of the infinite ladder shown in figure.
1 5 1 5
1) ( )r 2) ( )r
2 2
1 3 1 3
3) ( )r 4) ( )r
2 2
49. The equivalent resistance between the terminals
A and B of infinite network as shown is
the figure is
1) 
R 2) 1  3 R  
3) 1  3 R  4) infinete
50. The equivalent resistance of the following infinite network of resistances is
1) Less than 4  2) 4  2 2 2

3) More than 4  but less than 12  2 2 2


4) 12 
2 2 2

Dr.K.K.R Gowtham 101 Current Electricity


VI. KIRCHOFF’S LAW AND CELLS:
4A
1. In the given current distribution what is the value of I
I 2A
1) 3A 2) 8 A
3A
3) 2A 4) 5A
5A

2. The figure shows a network of currents. The magnitude of currents is shown here. The current I
will be 1A

1) 3 A
10 A I
2) 9 A
3) 13 A
6A
4)19 A
2A
3. Current i in the given branch of circuits is
1) 3.4A
2) 2.6A
3) 6A
4) 5.8A
4. The figure here shows a portion of a circuit. What are the magnitude and direction of the current i
in the lower right-hand wire 1A
1) 7 A 2A
2A
2) 8 A 2A

3) 6 A
3A 4A
4) 2 A i

5. The given four terminal net work is a part of larger circuit. The points A, B, C are at the same
potential. The P.D. between any one of A,B or C and D is 40V.
The P.D. between A& O is
1) 10v
2) 15v
3) 18v
4) 20v

6. The e.m.f. of a cell is E volts and internal resistance is r ohm. The resistance in external circuit is
also r ohm. The p.d. across the cell will be
1) E/2 2) 2E 3) 4E 4) E/4
7. A cell of e.m.f. E is connected with an external resistance R , then p.d. across cell is V . The
internal resistance of cell will be
( E  V )R ( E  V )R (V  E)R (V  E)R
1) 2) 3) 4)
E V V E
8. By a cell a current of 0.9 A flows through 2 ohm resistor and 0.3 A through 7 ohm resistor. The
internal resistance of the cell is
1) 0.5  2) 1.0  3) 1.2  4) 2.0 
9. A cell of e.m.f 1.5V, records a p.d of 1.2V, when connected to an external resistance R, such that
current flowing through circuit is 0.5A. Calculate the value of R and internal resistance of cell.

Dr.K.K.R Gowtham 102 Current Electricity


      
10. A cell, when connected to an external resistance of 2, shows a p.d of 1.2V. If the 2
resistance is replaced by 1 resistance, p.d. drops to 1 volt. Calculate e.m.f, internal resistance
of cell.
1) 1.4V,  2) 1.5V,  3) 2.4V,    2V, 
11. A cell of emf 2V and internal resistance 0.5 is charged with a current 3A. During charging, the
terminal P.d of the cell is
1) 3V 2) 0.5 V 3) 3.5 V 4) Zero
12. Two cells of emf 2V and 6V having internal resistance of 0.25 and 1 are connected so that +ve of
one cell to +ve of another. Then the terminal Voltage is –
1) 2V 2) 6V 3) 2.8V 4) 4V
13. The potential difference across the terminals of a batterry is 50 V when 11A are drawn and 60 V
when 1A is drawn. The emf and the internal resistance of the battery are
1) 62V,2  2) 63V,1 3) 61V,1 4) 64V,2
14. A 6V cell with 0.5 internal resistance, a 10 V cell with 1 internal resistance and a 12 external
resistance are connected in parallel. The current (in amperes) through the 10V cell is :
1) 0.60 2) 2.27 3) 2.87 4) 5.14
15. The magnitude and direction of the current in the circuit shown will be
7 1 e 2
1) A from a to b through e a b
3 10V 4V
7
2) A from b to a through e
3
3
3) 1A from b to a through e d c
4) 1A from a to b through e 60
16. The magnitude of i in ampere unit is i
1) 0.1 15 5
2) 0.3 1A 1A
3) 0.6
4) None of these 10
17. In the circuit shown, potential difference between X and Y & potential difference across the
40  resistance will be 40 X Y

1) Zero, 80V
2) 20 V, 40V
3) 60 V, 120V 20
4) 120 V, zero 120V
18. The internal resistances of two cells shown are 0.1  and 0.3  . If R  0.2  , the potential
difference across the cell 2V, 0.1 2V, 0.3
1) B will be zero
A B
2) A will be zero
3) A and B will be 2V
4) A will be  2V and B will be  2V 0.2

Dr.K.K.R Gowtham 103 Current Electricity


28 54
19. Consider the circuit shown in the figure. The current I3 is equal to
1) 5 amp 6V
2) 3 amp I3
3) 3 amp
8V 12 V
4) 5 / 6 amp
10 5V
20. If VAB  4 V in the given figure, then resistance X will be
B
1) 5  2) 10  A

3) 15  4) 20  2V X

21. Two batteries of e.m.f. 4V and 8 V with internal resistances


1 4V 8V 2
1  and 2  are connected in a circuit with a resistance of P Q
9  as shown in figure. The current and potential r1 r2
difference between the points P and Q are
1 1 9
1) A and 3 V 2) A and 4 V
3 6
1 1
3) A and 9 V 4) A and 12 V
9 2
22. In the shown circuit, what is the potential difference across A and B 20 V

1) 50 V
2) 45 V
3) 30 V
A B
4) 20 V
23. In the given circuit the current I1 is 30 
1) 0.4 A
I1
2) – 0.4 A 40 
3) 0.8 A I3
I2 40V
4) – 0.8 A 40 

80V
24. The current in the given circuit is
10 
5V
1) 0.1 A
2) 0.2 A A B
3) 0.3 A
4) 0.4 A 20 
2V

25. Two batteries A and B each of e.m.f. 2 V are connected in series to an external resistance R =
1 ohm. If the internal resistance of battery A is 1.9 ohms and that of B is 0.9 ohm, what is the
potential difference between the terminals of battery A A B
1) 2 V
2) 3.8 V
3) Zero
R
4) None of the above

Dr.K.K.R Gowtham 104 Current Electricity


26. Find out the value of current through 2 resistance for the given circuit
1)5 A
2) 2 A 5 10 20V
10V
3) Zero
2
4) 4 A
27. The potential difference across A & B in the given circuit i.e. (VA–VB) is
1) 12V 2) 24V
3) 6V 4) 18V
28. An ammeter A is connected as shown in the diagram. Ammeter reading
is
1) 2E/r
2) 5E/r
3) E/r
4) E/2r

29. In the circuit R1 = 400 ohm and R2= 200 ohm. If the resistance of the voltmeter R = 10,000 ohm,
voltmeter reading is
1) 2V
2) greater
3) greater than 3V
4) less than 2V

30. In the given circuit the potential difference between X and Y


1) 6.7V
2) 3.7V
3) 4V
4) 10V
31. Three conductors draw respectively currents of 1A, 2A and 4A when connected in turn across a
battery. If they are connected in series across the same battery, the current drawn will be :
1) 2/7 A 2) 3/7 A 3) 4/7 A 4) 5/7 A
32. In the circuit, the galvanometer 'G' shows zero deflection. If the batteries A & B have negligible
internal resistance, the value of resistor 'R' is ______
1) 1000 2) 100
3) 500 4) 200

33. If the current through 1 ohm resistor is 2A, the ammeter reading is
1) 1A 2) 2A
3) 1.5A 4) 2.5A

Dr.K.K.R Gowtham 105 Current Electricity


34. In the given circuit, the current is
1) 1A from A to B through e
2) 1A from B to A through e
3) 3A from A to B through e
4) 3A from B to A through e
35. In the given circuit if Ammeter reads 5A, then emf of the cell is
1) 12V
2) 18V
3) 24V
4) 36V
36. At 500C the resistance of a metallic wire is 8  and its temperature coefficient of resistivity is
4x10-3/0C. Under a given potential difference it carries a current of 25mA. Keeping P.d. constant,
if the temperature of wire is increased to 1250C, the current that flows through the wire in mA is
1) 62.5 2) 20 3) 15 4) 25

37. An uniform conducting wire of 16 resistance is stretched uniformly so that its length increases
by 50 percent. Then it is bent in the form of a square. A battery of e.m.f 12 volt, internal
resistance one ohm is connected between two opposite corners. A resistance of 10 is connected
between other two opposite corners, current flowing through one of the sides of square (in
ampere) is
1) 1.2A 2) 0.6A 3) 0.3A 4) 6A
38. An electric iron draws 5A, a TV set draws 3A and a refrigerator draws 2A from a 200 V main
line. The three appliances are connected in parallel. If all the three are operating at the same
level, the fuse used may be of :
1) 20 A 2) 11.1 A 3) 15 A 4) 5 A
39. 20 Cells each of 6V are connected in series. The emf of the combination is found to be 96V.How
many cells are connected wrong?
1) 2 2) 4 3) 6 4) 12
40. Four identical cells each having an electromotive force (e.m.f.) of 12V, are connected in parallel.
The resultant electromotive force (e.m.f.) of the combination is
1) 48 V 2) 12 V 3) 4 V 4) 3 V
41. In the given circuit, each cell has 5V and internal resistance of 0.2. The reading of ideal
voltmeter is ––––
1) Zero 2) 5V
3) 10V 4) 20V
42. Three resistors of 5; 8 and 11 are connected in series to a
battery of e.m.f 24V and negligible internal resistance. Calculate
current drawn from cell, p.d at the ends of 5ohm resistor.
1) 1A, 5 V 2) 2A, 3V
3) 3A, 2V 4) 5A, 1V
43. Carefully study the circuit diagram shown in figure
and calculate the value of resistor x in ohms.
1) 20    
   

Dr.K.K.R Gowtham 106 Current Electricity


 Three resistors of 6; 3 and 2 are connected in
parallel. The combination of above resistors is
connected in series to a resistance of 4 and then to a
cell of e.m.f. 1.5V. If the internal resistance of cell is
negligible Calculate the current in main circuit and
Calculate the current in 3 ohm resistor.
1) 0.3A, 0.1A 2) 0.2A, 0.3A 3) 0.4, 0.2A 4) 0.2, 0.2 A
45. Figure shows a circuit diagram, containing eight cells, each
of e.m.f 1.5V and internal resistance 0.25. Calculate: Total
external resistance and drop in potential across the terminals
of cell.
1) 4 V V 2V 3V
46. Four cells, each of e.m.f 2V and internal resistance 2 are connected in parallel, to form battery.
The battery is connected to external resistance of 2.5 and a set of two resistances of 6 and 3
in parallel. Draw a circuit diagram and answer the
following: What is the current in main circuit, What is
the current in 6 wire, What is the drop in potential?
1) 0.1A, 0.13A, 0.2V 2) 0.2A, 0.4V, 0.1V
3) 0.4A, 0.2V, 0.1V 4) 0.2A, 0.1V, 0.2V

47. Eels are able to generate current with biological cells


called electroplaques. The electroplaques in an eel are
arranged in 100 rows, each row stretching horizontally
along the body of the fish containing 5000 0.15 V
+ – + – + – 0.25 
electroplaques. The arrangement is suggestively shown
below. Each electroplaques has an emf of 0.15 V and + – + – + –
internal resistance of 0.25  5000 electroplaques per row
100 rows
The water surrounding the eel completes a circuit
between the head and its tail. If the water surrounding it
has a resistance of 500, the current an eel can produce + – + – + –
in water is about
1) 1.5 A 2) 3.0 A 3) 15 A 4) 30 A
500 
48. In the given circuit, choose the correct answers
I) Current in the circuit is 0.5 A
II) E is at zero potential
III) D is at 5V potential
IV) B is at 9V
1) I, II are correct 2) II, III are correct 3) I, II, III are correct 4) I, III, IV are correct
SHORT CIRCUIT
49. Two cells of emfs E1 & E2 are connected to two resistors R1 & R2 as shown. If E2 is short
circuited, then currents through R1 & R2 are–
E1 E1 E1 E1
1) ;O 2) O; 3) ;O 4) O;
R2 R2 R1 R1
50. Find the currents through the three resistors shown in figure
1)zero in the upper 4resistor and 0.2A in the rest two

Dr.K.K.R Gowtham 107 Current Electricity


2)zero in the upper 4resistor and 0.1A in the rest two
3)1A in the upper 4 resistor and 0.2A in the rest two
4)2A in the upper 4resistor and 0.2A in the rest two

VII. WHEASTONE BRIDGE:


1. Four resistances P,Q,R,X form a Wheatstone bridge. The bridge is balanced with R=100. If P and
Q are interchanged the bridge balanced for R = 121. The value of X is
1) 100  2) 200 3) 300 4) 110
2. If the deflection in the galvanometer is zero, then the current flowing through the 5 resistor is
1) 0.5A
2) 0.6A
3) 0.9A
4) 1.5A

3. Five equal resistances each of value R are connected in a form shown alongside. The
equivalent resistance of the network
1) Between the points B and D is R B

R R R
2) Between the points B and D is
2
A R C
3) Between the points A and C is R
R R R
4) Between the points A and C is
2 D
4. Effective resistance between A and B in the given circuit is
1) r 2) 3r
3) 5r 4) 4r

5. Effective resistance between A and B in the given circuit is


1) 6r
2) r
3) 2r
4) r/2
6. If the reading of Galvanometer is zero
effective resistance between A & B in is
1) 4
2) 7
3) 14
4) 8

7. Effective resistance between A and B in the given circuit is


1) 4  2) 4.5 
3) 2  4) 2.25 

Dr.K.K.R Gowtham 108 Current Electricity


8. Effective resistance between A and B in the given circuit is
1) 3.5 R 2) 3 R
7R
3) 8 R 4)
6
9. Effective resistance between A and B in the given circuit is
1) 12  2) 4.8 
3) 6  4) 4 

10. In the circuit shown in the adjoining figure, the current between B and D is zero, the
unknown resistance is of B

1) 4  4 X

2) 2  12
A C
1
3) 3  1

4) em.f. of a cell is required to find the value of X 1


3
D
11. In the circuit shown in the figure, the current flowing in 2  resistance
1) 1.4 A
10 2
2) 1.2 A
1.4A
3) 0.4 A G
4) 1.0 A
25 5

12. In the figure given the value of X resistance will be, when the p.d. between B and D is zero
1) 4 ohm 2) 6 ohm B
X
3) 8 ohm 4) 9 ohm 6

8 3
13. For the network shown in the figure the value of the current i is 15
A C
9V 4
1) 2
35 15 6

5V 4 4
2) 4
3 6 4
18
D
5V 6
3)
9
i
18 V
4) V
5
VIII. METER BRIDGE: R 80 
1. AB is a wire of uniform resistance. The galvanometer G shows
no current when the length AC=20cm and CB = 80 cm.
G
The resistance R is equal to
A B
1) 2  2) 8  C

3) 20  4) 40 

Dr.K.K.R Gowtham 109 Current Electricity


2. In the circuit shown, a meter bridge is in its balanced state. The meter bridge wire has a
resistance 0.1 ohm/cm. The value of unknown resistance X and the current drawn from the
battery of negligible resistance is X 6
1) 6 , 5 amp
2) 10 , 0.1 amp G
40 cm 60 cm
3) 4 , 1.0 amp A B

4) 12 , 0.5 amp
5V
3. In the figure shown for gives values of R1 and R2 the balance point for
Jockey is at 40 cm from A. When R2 is shunted by a resistance of 10
, balance shifts to 50 cm. R1 and R2 are (AB = 1m)
10
1)  ,5 2) 20, 30
3
15
4) 10, 15 3) 5, 
2
4. The figure shows a metre- bridge circuit, with AB = 100 cm, X
= 12 and R = 18, and the jockey J in the position of
balance. If R is now made 8 , through what distance will J
have to be moved to obtain balance ?
1) 10 cm 2) 20 cm 3) 30 cm 4) 40 cm
5. A meter bridge is set-up as shown, to determine an unknown
resistance ’X’ using a standard 10 ohm resistor. The
galvanometer shows null point when tapping- key is at 52
cm mark. The end-corrections are 1 cm and 2 cm
respectively for the ends A and B. The determined value of
’X’ is
1) 10.2 ohm 2) 10.6 ohm
3) 10.8 ohm 4) 11.1 ohm
6. 6 ohm coil is connected in the left gap, 3 ohm and 6 ohm coils in parallel is connected in right
gap. The balancing point is
1) 25cm 2) 50cm 3) 66.6 cm 4) 75cm
7. The balancing point in a meter bridge is 44 cm. If the resistances in the gaps are interchanged,
the new balance point is
1) 44 cm 2) 56 cm 3) 50 cm 4) 22 cm
8. Two equal resistances are connected in the gaps of a metre bridge. If the resistance in the left gap
is increased by 10%, the balancing point shift
1) 10% to right 2) 10% to left 3) 9.6 to right 4) 4.8% to right
9. A constant resistance is kept in the left gap of a metre bridge. For two different resistances
separately in right gap, the balancing lengths are 40 & 60cm. If these two are connected in series
in right gap, the balancing length is
1) 20 cm 2) 30 cm 3) 40cm 4) 45 cm
10. Two resistances X & Y are in the left and right gaps of a metre bridge. The balancing point is
40cm from left. Two resistances of 10 each are connected in series with X & Y separately. The
balance point is 45cm. Then X and Y are
1) 2&8 Ohms 2) 4 & 6 3) 8  & 12 4) 12 & 16

Dr.K.K.R Gowtham 110 Current Electricity


11. In a metre bridge expt, when the resistances in the gaps are interchanged the balance point is
shifted by 10cm. The ratio of the resistances is
1) 15:5 2) 12:8 3) 11:9 4) 10:9
12. When an unknown resistance and a resistance of 6  are connected in the left and right gaps of a
meter bridge, the balancing point is obtained at 50cm. If 3 resistance is connected in parallel to
resistance in right gap, the balance point
1) decrease by 25cm 2) increases by 25 cm
3) decrease by 16.7 cm 4) increases by 16.7 cm
13. In a meter bridge, the balancing point is obtained at 40 cm. If a resistance equal to that in the left
gap in shunted across it, the new balancing point is
1) 55cm 2) 50 cm 3) 25 cm 4) 33.3 cm
14. In a meter bridge, the left and right gaps are closed by resistances 2 ohm and 3 ohm respectively.
The value of shunt to be connected to 3 ohm resistor to shift the balancing point by 22.5 cm is
1) 3 ohm 2) 1.7 ohm 3) 1 ohm 4) 2.5 ohm
0
15. A metallic conductor at 10 C connected in the left gap of Metre Bridge gives balancing length
40cm. When the conductor is at 600C, the balance point shifts by 5cm. The temperature
coefficient of resistance of the material of the wire is
1) (1/210) /0C 2) (1/220) / 0C 3) (1/200) /0C 4) (1/201) /0C
IX. HEATING EFFECT OF ELECTRIC CURRENT
1. A 220 V, 100W bulb is connected to a 110 V source. The power consumed by the bulb is
1) 25W 2) 50W 3) 75W 4) 100W
2. If the current in a heater increases by 20% the percentage increase in power consumption by the
heater will be
1) 10% 2) 22% 3) 12% 4) 44%
3. Two resistance filaments of same length are connected first in series and then in parallel. Find the
ratio of power dissipated in both cases assuming that equal current flows in the main circuit
1) 4:1 2) 1:4 3) 1:2 4) 2:1
4. 1 KW heater of capacity 1 litre is used for heating water. The time required to raise the temperature
of water by is (specific heat of water = 1 cal g 1 0C 1 and J  4.2 Jcal 1 )
1) 21s 2) 42s 3) 84s 4) none
5. A heater coil rated at 1000 watt is connected to a 110V mains. How much time will it take to melt
625 gm of ice at at (for ice L = 80 cal/gm)
1) 120 sec 2) 100 sec 3) 150 sec 4) 210 sec
6. A wire of resistance 20  is covered with ice and a voltage of 210V, is applied across the wire, then
rate of melting the ice is
1) 0.85 g/s 2) 1.92 g/s 3) 6.56 g/s 4) 7.56 g/s
7. A coil of resistance R is connected between the terminals for a battery. The quantity of heat
produced in 5 minutes is . Now that coil is cut into two equal parts and are connected between the
same terminals in parallel. This time heat produced in 5 minutes is. Then the ratio of is
1) 1 : 4 2) 4 : 1 3) 1 : 2 4) 2 : 1
8. A coil of resistance is immersed in 1kg of water and is connected to the terminals of the battery of
internal resistance and e.m.f 6 volt for 3 minutes. The increase in temperature of water is
1) 0.930C 2) 0.0850C 3) 1.920C 4) 4.310C
9 A 2KW heater of capacity 10 litres is used for heating water. The time required tc raise the tempera
ture of water by 10K is (mass of 1 litre of water =1Kg, specific heat of water = 4200J Kg-1 K-1)

Dr.K.K.R Gowtham 111 Current Electricity


1) 120 s 2) 210 s 3) 3600s 4) 30s
10. A heater coil working on mains is cut into three equal parts and one part is only used for
heating purpose. The quantity of heat produced is
1) 3 times the original heat 2) 2 times the original value
1 1
3) times the original value 4) times original value
3 2
11. According to Joule’s law, if potential difference across a conductor having a material of specific
resistance  remains constant, then heat produced in the conductor is directly proportional to
1)  2) 2 3) 1/  4) 1/
12. An electric bulb rated for 500W at 100V is used in a circuit having a 200V supply.
Calculate the resistance that must be put in series with the bulb so that bulb delivers 500W
1) 10  2) 20  3) 30 4) 40
13. Two heater wires of equal leangth are first connected in series and then in parallel. The ratio of
heat produced in two cases is
1) 2 : 1 2) 1 : 2 3) 4 : 1 4) 1 : 4
14. Two bulbs rated (25 W- 220 V) and (100 W- 220 V) are connected in series to a 440 V line.
Which one is likely to fuse ?
1) 25 W bulb 2) 100 W bulb 3) both bulbs 4) none
15. The charge flowing through a resistance R varies with time as Q = 2t - 8t2. The total heat
 1
produced in the resistance is  for 0  t  
 8
R R R
1) joules 2) joules 3) joules 4) R joules
6 3 2
16. A heater A gives out 300 W of heat when connected to a 200 V d.c. supply. A second heater `
B gives out 600 W when connected to a 200 v d.c. supply. If a series combination of the two
heaters is connected to a 200 V d.c. supply the heat output will be
1) 100 W 2) 450 W 3) 300 W 4) 200 W
17. Two bulbs one of 200 volts, 60 watts & the other of 200 volts, 100 watts are connected in
series to a 200 volt supply. The power consumed will be
1) 37.5 watt 2) 160 watt 3) 62.5 watt 4) 110 watt
18. The ratio of powers dissipated respectively in R and 3R, as shown is
1) 9 2) 27/4
3) 4/9 4) 4/27

19. Which arrangement of your identical resistances should be used to draw maximum energy from a
cell of voltage ?

Dr.K.K.R Gowtham 112 Current Electricity


20. If in the circuit, power dissipation is 150 W then R is
1) 2  2) 6 
3) 5  4) 4 

MULTIPLE CORRECT CHOICE TYPE QUESTIONS:


1. When no current is passed through a conductor,
A) the free electron do not move
B) the average speed of a free electron over a large period of time is zero
C) the average velocity of a free electron over a large period of time is zero
D) the average of the velocities of all the free electron at an instant is zero
2. Which of the following quantities do not change when a resistor connected to a battery is heated
due to the current?
A) Drift speed B) Resistivity C) Resistance D) Number of free electrons
3. Mark the correct statement (s).
A) Current is the flow of charge. B) Current is the ordered flow of charge.
C) Direction of current merely represents the direction of flow of charge.
D) Current is a scalar quantity.
1 2 1 3
4. The charge flowing in a conductor varies with time as Q = at  bt + ct , Where a, b, c are
2 6
positive constants. Then, the current
A) has an initial value i = a
b
B) reaches a minimum value after a time period t =
c
b
C) reaches a maximum value after a time period t =
c
b2
D) has either a maximum or a minimum value i = a 
2c
5. The charge flowing through a conductor varies with time as Q = at – bt2. Then, the current
A) Decreases linearly with time B) Reaches a maximum and then decreases
a
C) Falls to zero after time t  D) Changes at a rate of –2b
2b
6. Which of the following statement is true?
A) Conductance is the reciprocal of resistance and is measured in siemens.
B) Ohm’s law is applicable at constant temperatures.
C) Ohm’s law is applicable to semiconductors.
D) Ohm’s law is not applicable to electron tubes, discharge tubes and electrolytes.

Dr.K.K.R Gowtham 113 Current Electricity


7. Two identical dimension resistors having resistance R1, R2 at 0C and temperature co-efficient of
resistance 1, 2 respectively are connected in series. If their resistivities are respectively 1 and 2
then
α1R1 + α 2 R 2
A) Their effective combined temperature co-efficient of resistance is
R1 + R 2

   
1
B) Their effective combined resistivity is
2 1 2
R1R 2
C)Their effective combined resistance is
R1 + R 2
D) Resistance of combination doesn't change with temperature if 1R1 + 2R2 = 0

8. The fig. shown below is a balanced Wheaton’s bridge and ammeter is ideal. The reading of
ammeter is initially I1 . Now 9 resistor is removed and the final reading of ammeter is I 2 . Then
the select the correct statement(s)
9 15
>
>

>
>
>

>
>

>

>
> 5
>
>
>
>

>
>
>

>
>

>

18 30

A
240 V

A) I1  15amp B) I1  8amp C) I2  15amp D) I2  8amp .

9. In the given circuit, choose the correct answers


A) current in the circuit is 0.5A B) E is at zero potential
C) D is at 5V potential D) B is at 9V

10. When some potential difference is maintained between A and B, current I enter the network at A
and leaves at B.
5
A) The equivalent resistance between A and B is 8  20  C

B) C and D are at the same potential.


A B
C) No current flows between C and D. I

I 5 20 
D) Current flows from D to C.
5
11. Two bulbs consume same power when operated at 200V and 300 V respectively when the bulbs
are connected in series across a D.C source of 500 V, then
A) ratio of potential difference across them is 3 : 2 respectively
B) ratio of potential difference across them is 4 : 9 respectively

Dr.K.K.R Gowtham 114 Current Electricity


C) ratio of power consumed by them is 4 : 9 respectively
D) ratio of power consumed by them is 2 : 3 respectively
12. In the network shown, points A,B and C are at potentials of 70V, 0V and 10V respectively.

A) Point D is at a potential of 40V


B) The currents in the sections AD, DB, DC are in the ratio 3 : 2 : 1
C) The currents in the sections AD,DB,DC are in the ratio 1 : 2 : 3
D) The network draws a total power of 200 W
13. Which of the following are wrong?
If in the wheat stone bridge the battery and the galvanometer are interchanged, the condition for
balance
A) is disturbed B) is not disturbed
C) depends on the internal resistances of the battery
D) depends on the values of resistances in the bridge
14. In a balanced wheat stone bridge, current in the galvanometer is zero. It remains zero when
A) battery emf is increased
B) all resistance are increased by 10 

C) all resistance are made five time


D) the battery and the galvanometer are interchanged

LINKED COMPREHENSION TYPE QUESTIONS


PASSAGE 1: An electrical conductor designed to carry large currents has a circular cross section
2.50 mm in diameter and is 14.0 m long. The resistance between its ends is 0.104
1. What is the resistivity of the material?
A)3.6510-8-m B) 2.3510-8-m C) 1.5510-8-m D) 4.2510-8-m
2. If the electric-field magnitude in the conductor is 1.28 V/m, what is the total current?
A)150A B)172A C)235A D)100A
28
3. If the material has 8.510 free electrons per cubic meter, find the average drift sped.
A)2.58mm/s B)1.28mm/s C)3.26mm/s D)4.12mm/s
I
PASSAGE 2: Current density (J) =
A
1. A current of 1.8 A flows through a wire of area of cross section 0.5mm2. Find the current density
in the wire
A)3.6106Am-2 B) 4.2106Am-2 C) 2.1106Am-2 D)1.3106Am-2
2. A silver wire 1mm in diameter carries a charge of 90 coulombs in one hour and 15 minutes.
Silver contains 5.81022 free electrons cm-3. Find the current density in the wire
A)2.6910-6A/m2 B) 1.3610-5A/m2 C) 2.55104A/m2 D)3.5510-4A/m2

Dr.K.K.R Gowtham 115 Current Electricity


3. If two roads A,B of radii 2cm and 4cm connected on by one respectively, the current flowing
through them is same, the ratio of current densities is
A) 4:1 B)1:4 C)1:2 D)2:1
PASSAGE 3: Rt=R0(1+t)
1. The resistance of a thin silver wire is 1.0 . What is the temperature of the bath? α for silver is
3.8  10-3K-1. The temperature that it should be heated to get a resistance of 3 is
A)583.40c B) 72.60c C) 64.50c D) 81.70c
2. The resistance of a wire is 2 at 3000K. Its temperature coefficient of résistance is 1.2510-3K-1.
The temperature that it should be heated to get a resistance of 3 is
A) 1400K B)600K C) 1000K D) 850K
3. The resistance of bulb filament is 100  at a temperature of 100 c. If its temperature Coefficient
0

of resistance be 0.005 0C-1, Its resistance will become 200 at a Temperature of


A)500 0C B)2000C C)3000C D)4000C
PASSAGE 4:
A circuit is comprised of eight identical batteries and a resistor R = 0.8ohm. Each battery has an
emf of 1.0 V and internal resistance of 0.2ohm. What is the voltage difference across any of the
battery There are n cells, each of emf  internal résistance r. They are connected in parallel.
1. The total emf and internal resistance of n cells is
A)  , nr B)  , r/n C) n  ,nr D) n  , r/n
2. In parallel combination of cells, we get
A) more current B) more voltage C) more power D) less efficiency
3 If n cells are connected in series and two cells are wrongly connected, the total emf and internal
resistance of cells will be
A)  n  2  , nr B)  n  4  , nr C) n , nr D)   2 / n  , r / n

PASSAGE 5: The figure shows a network of five resistances and two batteries

1. The current through the 30V battery is


A) 3A B) 1A C) 2A D) none
2. The current through the 15V battery is
A) zero B) 1A C) 3A D) none
3 Which of the batteries is getting charged.
A) 30V B) 15V C) both D) none

Dr.K.K.R Gowtham 116 Current Electricity


MATRIX MATCHING TYPE QUESTIONS
1 Column –I Column -2
RA
a) rate of follow of charge p)
l
l
b) potential difference q)
A
c) Resistance r) iR
W
d) Resistivity s)
Q
Q
t)
t
2 Column – I Column – II
A. The unit of electrical resistivity is p) m 2 s 1V 1
B. The unit of current density is q)  1m 1
C. The unit of electrical conductivity is r) Am 2
D. The unit of electric mobility is s) m
3. A potential difference V is applied across a copper wire of diameter d and length l. Changes of
different parameters are given in column I and the corresponding changes in the drift velocities
are given in column II. Pick out the correct combinations.
Column – I Column – II (Drift velocity)
(A) Doubling l (p) Increases two times
(B) Doubling V (q) Decreases to half
(C) Doubling d (r) Remains same
(D) Increasing temperature (s) Decreases

4. Column –I Column -2
a) Ohm’s [ ] p) Conservation of charge
b) Joule’s Law [ ] q) Conservation of energy
c) Kirclihoff’s I law [ ] r) V=IR
d) Kirclihoff’s II law [ ] s) H= i 2 Rt
5. Match of the following
Four bulbs of 25W, 40W, 60W and 100W are connected in series and this combination is
connected across the mains. Match the bulbs in column –I with the potential difference across
each in column-II., writing the bulb across which the potential difference is the highest
against(p), the next highest against (q) and so on; the lowest against (s).

Dr.K.K.R Gowtham 117 Current Electricity


Column – I Column – II
(A) 25W (P) highest p.d.
(B) 100W (Q) second highest p.d.
(C) 60W (R) third highest p.d.
(D) 40W (S) lowest p.d.

INTEGER ANSWER TYPE QUESTIONS


1. Find the resistance of hollow cylindrical conductor of length 1.0m and inner and outer radii 1.0
mm and 2.0 mm respectively. The resistivity of the material is 2.0 108   m is n times
2.1 103  then n is
2. An aluminium wire having a cross-sectional area of 4 106 m2 carries a current of 5 A. The
drift speed of the electrons in the wire, Is 26n  ms 1 . The density of aluminium is 2.7 gcm-3.
Assume that one conduction electron is supplied by each atom. Find n.
3. When two resistances 3 and 6 are connected in series, their equivalent resistance is R1
and then they are correct in parallel, their equivalent resistance is R 2 . What is the value of
R1  R 2 ? (in  ).
1V
B
n
4. The potential difference VA  VB for the circuit shown in fig is  V .Find
3 >

>>
> 1 >

>>
the value of n.4 1 >
> >
1 >
>

>>
>
> >
>
5. For the network shown in figure, show that the resistance
 5n 
Rab  1    .Find n.
 17  A
1V

6. The meter-bridge wire AB shown in figure is 50cm long.


When AD=30cm, no deflection occurs in the galvanometer.
Then R is 2n  is then n is
R
7. If in the circuit, power dissipation is 150W , then R is ____ 
2

15V
8. The heat generated in 5 resistor due to current flowing
through it is 10 cal per sec. find the heat generated in 4 resistor (in cal/sec).
4 6

5

Dr.K.K.R Gowtham 118 Current Electricity


KEY
LEVEL - A
1) 3 2) 1 3) 1 4) 1 5) 2 6) 2 7) 3 8) 3 9) 2 10) 4
11) 1 12) 4 13) 2 14) 3 15) 1 16) 3 17) 3 18) 2 19) 3 20) 3
21) 2 22) 1 23) 4 24) 4 25) 1 26) 3 27) 4 28) 2 29) 3 30) 1
31) 2 32) 2 33) 1 34) 4 35) 1 36) 2 37) 3 38) 1 39) 2 40) 2
41) 2 42) 4 43) 1 44) 4 45) 3 46) 4 47) 1 48) 1 49) 4 50) 2
51) 2 52) 2 53) 3 54) 2 55) 4 56) 4 57) 2 58) 1 59) 3 60) 1
61) 4 62) 3 63) 2 64) 4 65) 2 66) 2 67) 3 68) 1 69) 3 70) 4
71) 4 72) 2 73) 2 74) 2 75) 2 76) 2 77) 3 78) 2 79) 3 80) 3
81) 1 82) 2 83) 2 84) 2 85) 3 86) 2 87) 4 88) 3 89) 1 90) 4
91) 2 92) 1 93) 2 94) 1 95) 1 96) 3 97) 2 98) 3 99) 2 100) 1
101) 1 102) 1 103) 3 104) 2 105) 3 106) 1 107) 3 108) 1 109) 2 110) 2
111) 3 112) 2 113) 3 114) 2 115) 2
LEVEL - B
I. ELECTRIC CURRENT:
1) 4 2) 1 3) 4 4) 2 5) 2 6) 3 7) 1 8) 2 9) 2 10) 3
II. CURRENT DENSITY:
1) 2 2) 2 3) 1 4) 2
III. DRIFT SPEED

1) 1 2) 1 3) 1 4) 1 5) 2 6) 3 7) 2 8) 4 9) 4 10) 4

IV. OHM’S LAW AND RESISTANCE:


1) 1 2) 2 3) 1 4) 4 5) 1 6) 2 7) 3 8) 4 9) 3 10) 3
11) 1 12) 2 13) 1 14) 1 15) 1 16) 4 17) 1 18) 3 19) 2 20) 1
21) 4 22) 2 23) 2 24) 4 25) 1 26) 1 27) 2 28) 3 29) 1 30) 1

V. GROUPING OF RESISTANCES:
1) 1 2) 2 3) 4 4) 3 5) 1 6) 2 7) 4 8) 2 9) 3 10) 1
11) 4 12) 2 13) 2 14) 2 15) 2 16) 2 17) 3 18) 3 19) 4 20) 3
21) 2 22) 4 23) 2 24) 3 25) 4 26) 1 27) 1 28) 4 29) 4 30) 2
31) 3 32) 3 33) 2 34) 3 35) 4 36) 1 37) 2 38) 2 39) 1 40) 3

Dr.K.K.R Gowtham 119 Current Electricity


41) 1 42) 3 43) 1 44) 1 45) 4 46) 1 47) 2 48) 1 49) 2 50) 3

VI. KIRCHOFF’S LAW AND CELLS:


1) 3 2) 3 3) 4 4) 2 5) 1 6) 1 7) 2 8) 1 9) 3 10) 2
11) 3 12) 3 13) 3 14) 3 15) 4 16) 3 17) 4 18) 1 19) 4 20) 4
21) 1 22) 4 23) 2 24) 1 25) 3 26) 4 27) 1 28) 1 29) 4 30) 2
31) 3 32) 2 33) 1 34) 1 35) 4 36) 2 37) 2 38) 4 39) 1 40) 2
41) 2 42) 1 43) 2 44) 1 45) 2 46) 1 47) 1 48) 4 49) 3 50) 1

VII. WHEASTONE BRIDGE:


1) 4 2) 2 3) 3 4) 1 5) 4 6) 1 7) 2 8) 1 9) 4 10) 2
11) 4 12) 3 13) 2

VIII. METER BRIDGE:


1) 3 2) 3 3) 1 4) 2 5) 2 6) 4 7) 2 8) 4 9) 2 10) 3
11) 3 12) 2 13) 3 14) 2 15) 1

IX. HEATING EFFECT OF ELECTRIC CURRENT


1) 1 2) 4 3) 1 4) 2 5) 4 6) 3 7) 1 8) 2 9) 2 10) 1
11) 4 12) 2 13) 4 14) 1 15) 1 16) 4 17) 1 18) 4 19) 2 20) 2
ONE OR MORE THAN ONE ANSWER TYPE QUESTIONS
1) C,D 2) D 3) B,C,D 4) A 5) A,C,D
6) A,B,D 7) A,B,D 8) A 9) A,C,D 10) A,B,D
11) D,C 12) A,B,D 13) A,C,D 14) A,C,D
LINKED COMPREHENSION TYPE QUESTIONS
PASSAGE – I 1 –A, 2 – B, 3 - A PASSAGE – II 1 –A, 2 – C, 3 – D
PASSAGE – III 1 –B, 2- D, 3- C PASSAGE – IV 1 –B, 2- A, 3- D
PASSAGE – V 1 –A, 2- B, 3- D
MATRIX MATCHING TYPE QUESTIONS
1) a – t ; b – r,s ; c – q ; d - p 2) a –s ; b – r ; c – q ; d – p
3) a – q ; b – p ; c – r ; d – s 4) a – r ; b – s ; c – p ; d – q
5) a – p ; b – s ; c – r ; d – q
INTEGER ANSWER TYPE QUESTIONS
1) 1 2) 5 3) 7 4) 4 5) 2 6) 2 7) 6 8) 2

Dr.K.K.R Gowtham 120 Current Electricity


LEVEL – C
POTENTIOMETER:
Potentiometer
1. A series high resistance is preferable than shunt resistance in the galvanometer circuit of
potentiometer. Because
1) shunt resistances are costly 2) shunt resistance damages the galvanometer
3) series resistance reduces the current through galvanometer in an unbalanced circuit
4) high resistances are easily available
2. The value of potential gradient in a potentiometer experiment in balanced state is due to
1) primary and secondary circuits 2) only by primary circuit
3) only by secondary circuit 4) nothing can be said
3. The sensitivity of potentiometer wire can be increased by
1) decreasing the length of potentiometer wire 2) increasing potential gradient on its wire
3) increasing emf of battery in the primary circuit 4) decreasing the potential gradient on its wire
4. A cell of emf ‘E’ and internal resistance ‘r’ connected in the secondary gets balanced against
length ‘’ of potentiometer wire. If a resistance ‘R’ is connected in parallel with the cell, then the
new balancing length for the cell will be
 R   Rr  R  R 
1)  l 2)  l 3)  4)  l
 RR  R  r Rr
5. Potentiometer is an ideal instrument, because
1) no current is drawn from the source of unkown emf
2)current is drawn from the source of unknown emf
3) it gives deflection even at null point 4) it has variable potential gradient
6. On increasing the resistance of the primary circuit of potentiometer, its potential gradient will
1) become more 2) become less 3) not change 4) become infinite
7. In potentiometer experiment, the unknown potential difference is compared with
1) unknown emf 2) known emf 3) known standard emf 4) internal resistance of cell
8. If the value of potential gradient on potentiometer wire is decreased, then the new null point will
be obtained at
1) lower length 2) higher length 3) same length 4) nothing can be said
9. A cell of negligible internal resistance is connected to a potentiometer wire and potential gradient
is found. Keeping the length as constant, if the radius of potentiometer wire is increased four
times, the potential gradient will become (no series resistance in primary)
1) 4 times 2) 2 times 3) half 4) constant
10. A potentiometer is used
1) to measure thermo emf 2) to calibrate voltmeter
3) to determine internal resistance of cell 4) all of the above
11. The material of wire of potentiometer is
1) Copper 2) Steel 3) Manganin 4) Aluminium
12. If the resistivity of a potentiometer wire be  and area of cross-section be A, then what will be
potential gradient along the wire
I I IA
1) 2) 3) 4) IA
A A 

Dr.K.K.R Gowtham 121 Current Electricity


13. For the working of potentiometer, the emf of cell in the primary circuit (E) compared to the emf
of the cell in the secondary circuit (E1) is
1) E > E1 2) E < E1 3) Both the above 4) none of the above
14. The balancing lengths of potentiometer wire are l1 and l2 when two cells of emf E1 and E2 are
connected in the secondary circuit in series first to help each other and next to oppose each other
is equal to (E1>E2).
l1 l1  l 2 l1  l 2 l2
1) 2) 3) 4)
l2 l1  l 2 l1  l 2 l1
15. At the moment when the potentiometer is balanced,
1) Current flows only in the primary circuit
2) Current flows only in the secondary circuit
3) Current flows both in primary and secondary circuits
4) current does not flow in any circuit
16. Temperature coefficient of resistance and resistivity of a potentiometer wire must be
1) high and low 2) low and high 3) low and low 4) high and high
17. Sensitivity of potentiometer
1) Increases with increase of length of the wire
2) Decreases with the increase of length of the wire
3) Increases with the decrease of radius of the wire 4) None of the above
18. The potential gradients on the potentio meter wire are V1 and V2 with an ideal cell and a real cell
of same emf in the primary circuit then
1) V1 = V2 2) V1 > V2 3) V1 < V2 4) None
19. On increasing the resistance of the primary circuit of potentiometer, its potential gradient will
become
1) More 2) Less 3) Do not charge 4) Infinite
20. If the current in the primary circuit is decreased, then balancing length is obtained at
1) Lower length 2) Higher length 3) Same length 4) None of the above
21. Sensitivity of potentiometer can be increased by
1)increasing series resistance in the primary circuit
b)decreasing the length of potentiometer wire
2)using thin and high resistivity wire as potentiometer wire
d)increasing the length of the wire
1) a and c are correct 2) b and d are correct 3) b and c are correct 4) a and d are correct
22. Choose the correct statements from the following
a)in potentiometer experiment balancing length gives the potential difference across the
secondary cell
b)the shunt resistance between the terminals of cell in the secondary circuit of potentiometer
increases, the balancing length increases
c)in potentiometer experiment the e.m.f of the cell in the secondary circuit may be greater than
the e.m.f of cell in the primary circuit
d) potentiometer is used to measure thermo e.m.f. also
1) b and c are correct 2) c and d are correct 3) a and d are correct 4) a and b are correct

Dr.K.K.R Gowtham 122 Current Electricity


23. Choose the correct option from the following statements
1)ohm's law is applicable at constant temperature
b)when temperature is raised, resistance of copper increases and resistance of germanium
decreases
c) potentiometer is used to measure e.m.f of a cell
d)closed mesh law obeys the law of conservation of charge
1) a, b and c 2) a, b and d 3) c and d 4) a and c
24. The circuit shown here is used to compare the e.m.f. of two cells E1 and E2 (E1  E2 ) . The null
point is at C when the galvanometer is connected to E1 . When the galvanometer is connected to
B
E 2 , the null point will be
1) To the left of C
C
2) To the right of C
A B
3) At C itself E1
4) Nowhere on AB E2 G

25. A potentiometer circuit shown in the figure is set up to measure e.m.f. of a cell E. As the point P
moves from X to Y the galvanometer G shows deflection always in one direction, but the
deflection decreases continuously until Y is reached. In order to obtain balance point between X
and Y it is necessary to V R
1) Decreases the resistance R
2) Increase the resistance R P
X Y
3) Reverse the terminals of battery V
4) Reverse the terminals of cell E E
G

R 2V
26. AB is a potentiometer wire of length 100 cm and its resistance is 10
ohms. It is connected in series with a resistance R = 40 ohms and a
40 cm
battery of e.m.f. 2 V and negligible internal resistance. If a source A B
of unknown e.m.f. E is balanced by 40 cm length of the
potentiometer wire, the value of E is
1)0.8 V 2)1.6 V E
3) 0.08 V 4) 0.16 V
27. In the figure, the potentiometer wire AB of length L and resistance 9r
is joined to the cell D of emf  and internal resistance r. The cell C.s
emf is /2 and its internal resistance is 2r. The galvanometer G will
show no deflection when the length AJ is
4L 5L 7L 11L
1) 2) 3) 4)
9 9 18 18
28. In given figure, the potentiometer wire AB has a resistance of 5 
and length 10 m. The balancing length AM for the emf of 0.4 V is R=45
1) 0.4 m 2) 4 m 5V
3) 0.8 m 4) 8 m M
A B
0.4V

Dr.K.K.R Gowtham 123 Current Electricity


29. In the given figure, battery E is balanced on 55 cm length of potentiometer wire but when a
resistance of 10  is connected in parallel with the battery then it balances on 50 cm length of the
potentiometer wire then internal resistance r of the battery is 2V

1) 1  1m
2) 3  B
3) 10  A
E r
4) 5 
30. A resistance of 4  and a wire of length 5 metres and resistance 4 10V

5  are joined in series and connected to a cell of e.m.f. 10 V and 1

internal resistance 1  . A parallel combination of two identical 3m


cells is balanced across 300 cm of the wire. The e.m.f. E of each 5, 5m
cell is E
G
1) 1.5 V 2) 3.0 V E
3) 0.67 V 4) 1.33 V
31. A battery of emf E0 = 12 V is connected across a 4m long uniform
wire having resistance 4/m. The cells of small emfs 1 = 2V and
2 = 4V having internal resistance 2 and 6 respectively, are
connected as shown in the figure. If galvanometer shows no
deflection at the point N, the distance of point N from the point A is
equal to
1 1
1) m 2) m 3) 25 cm 4) 50 cm
6 3
32. In the arrangement shown in figure when the switch S2 is
open, the galvanometer shows no deflection for l = L/2.
When the switch S2 is closed, the galvanometer shows no
deflection for l = 5L/12. The internal resistance (r) of 6 V
cell, and the emf E of the other battery are respectively.
1) 3, 8V 2) 2, 12V
3) 2, 24V 4) 3, 12V

33. In a potentiometer experiment the balancing with a cell is at


length 240 cm. On shunting the cell with a resistance of 2 ,
the balancing length becomes 120 cm. The internal resistance of the cell is
1)4  2) 2  3)1  4) 0.5 
34. Potentiometer wire of length 1 m is connected in series with 490  resistance and 2V battery. If
0.2 mV/cm is the potential gradient, then resistance of the potentiometer wire is
1)4.9  2) 7.9  3) 5.9  4) 6.9 
35. In a potentiometer experiment, when two cells are joined in series to support each other and then
joined to oppose each other balancing points are obtained at 6m and 2m respectively. The ratio of
their e.m.f. is
1) 1:1 2) 2:1 3) 3:1 4) 4:1
36. In a potentiometer experiment, two cells of emf’s E1 and E2 balances for a length of 800cm
when they are in series. If the terminals of the cell of E 2 in reversed then the balancing length is
200cm. If E1>E2, the ratio E1:E2 is
1) 5 : 3 2) 2 : 1 3) 3 : 2 4) 4 : 1

Dr.K.K.R Gowtham 124 Current Electricity


37. The emf of a cell is 2V and its internal resistance is 2 ohm. A resistance of 8 ohm is joined to
battery in parallel. This is contacted in secondary circuit of potentio meter. If 1V standard cell
balances for 100cm of potentiometer wire, the balance point of above cell is
1) 120cm 2) 240cm 3) 160cm 4) 116cm
38. A 10 m long wire of resistance 15 ohm is connected in series with a battery of emf 2V (no internal
resistance) and a resistance of 5 ohm. The potential gradient along the wire is
1) 0.15 Vm–1 2) 0.45 Vm–1 3) 1.5 Vm–1 4) 4.5 Vm–1
39. In an experiment to determine the internal resistance of a cell with potentiometer, the balancing
length is 260 cm. When a resistance of 3 ohm is joined in parallel with the cell, the balancing length
is 150 cm. The internal resistance of the cell is
1) 2.2 ohm 2) 1.1 ohm 3) 3.3 ohm 4) 3 ohm
40. A potentiometer wire consists of two wires of length L 1 and L2 of resistances R1 and R2 in series.
If a current ‘i’ is passed through it, the ratio of potential gradients will be
R1 R2 R1 R2 R1  R2 R1 R2 R1  R2
1) : 2) : 3) : 4)
L2 L1 L1 L2 2 L1 L2 R1  R2
41. A cell in the secondary circuit gives balancing length for 1.5 m length in potentiometer of length
10m. If the length of potentiometer wire is increased by 1m without changing the cells in primary or
secondary, balancing length becomes
1) 1.5 m 2) 1.65m 3) 3m 4) 3.5 m
42. A standard resistance of 1 ohm and a small resistance ‘r’ are connected in series. The balancing
length on potentiometer wire corresponding to this combination is 630 cm where as the balancing
length corresponding to 1 ohm resistance for same current through it is 600cm. The value of ‘r’ will
be
1) 0.5  2) 1 3) 2 4) 0.05
43. A potentiometer wire of length 10m and resistance 30 ohm is connected in series with a battery of
emf 2.5V, internal resistance 5 ohm and external resistance R. If the fall of potential along the
potentiometer wire is 50mV/m, the value of R in ohms is
1) 115 2) 80 3) 50 4) 100
44. In potentiometer experiment, a standard cell of 1.2 V emf gets balanced at 260 cm length of
potentiometer wire. If a current of 0.2 A flows through 3 ohm resistance, the balancing length for
this p.d. across 3 ohm will be
1) 80cm 2) 130 cm 3) 520 cm 4) 260 cm
45. A potentiometer has a wire of 100 cm length and its resistance is 10 ohms. It is connected in series
with a resistance of 40 ohms and a battery of emf 2V and negligible internal resistance. If a source
of unknown emf connected in the secondary is balanced by 40cm length of potentiometer wire, the
value of ‘E’ is
1) 0.8V 2) 1.6V 3) 0.08V 4) 0.16 V
46. The balancing length for a cell in secondary is 560 cm in a potentiometer experiment. when an
external resistance of 10 is connected in parallel to that cell, the balancing length changes by 60 cm.
The internal resistance of the cell is ohms is
1) 1.6 2) 1.4 3) 1.2 4) 0.12
47. A student finds the balancing length as’’ with a cell of constant emf in the secondary circuit.
Another student connects the same cell in the secondary circuit of potentiometer of half the length
but with a cell of double the emf in the primary circuit than used in the primary of circuit of first
case. Then the balancing length will be (No series resistance in primary)
1) l /4 2) l /2 3) 4 l 4) l

Dr.K.K.R Gowtham 125 Current Electricity


48. In an experiment for calibration of voltmeter, a standard cell of emf 1.5V is balanced at 300cm
length of potentiometer wire. The P.D. across a resistance in the circuit is balanced at 1.25m. If a
voltmeter is connected across the same resistance, it reads 0.65V. The error in the voltmeter is
1) 0.05V 2) 0.025V 3) 0.5 V 4) 0.25V
49. A potentiometer wire of length 10m and resistance 9.8 is connected in series with a battery of emf
2V and internal resistance 0.2. The balancing length of 1st cell of emf E in potentiometer is 4m.
When 2ohm resistance is connected in series with the potentiometer wire, the change in balancing
length is
1) Decreases by 0.8m 2) Increases by 1m 3) Decreases by 1m 4) Increase by 0.8 m
50. Twelve cells, each having e.mf. ‘E’ volts are connected in series and are kept in a closed box. Some
of these cells are wrongly connected with positive and negative terminals reversed. This 12 cell
battery is connected in series with an ammeter, an external resistance ‘R’ohms and a two cell battery
(two cells of the same type used earlier, connected perfectly in series). The current in the circuit
when the 12-cell battery and 2-cell battery aid each other is 12E/R. Then the number of cells in 12-
cell battery that are connected wrongly is :
1) 4 2) 3 3) 2 4) 1
POTENTIOMETER:
1) 3 2) 2 3) 4 4) 4 5) 1 6) 2 7) 3 8) 2 9) 4 10) 4
11) 3 12) 1 13) 1 14) 15) 1 16) 2 17) 1 18) 2 19) 2 20) 2
21) 4 22) 3 23) 1 24) 1 25) 1 26) 4 27) 2 28) 4 29) 1 30) 2
31) 3 32) 2 33) 2 34) 1 35) 2 36) 1 37) 3 38) 1 39) 1 40) 2
41) 2 42) 4 43) 1 44) 2 45) 4 46) 3 47) 1 48) 2 49) 4 50) 4

I. SINGLE CORRECT CHOICE TYPE QUESTIONS:


1. Current in the wire varies according to equation I  15sin50t where I is in ampere, t is in
second and  50t  is in radian. How much net charge has passed a cross-section of the wire in the
interval t  0 to t  40 ms :
A) 0.080 C B) 0.80 C C) 0.030 C D) Zero
 X  R
 J 0   1 for 0  x 

Current density in a cylindrical wire of radius R is given as  
R  2
2. .
J X R
for  x  R

 R
0
2
The current flowing in the wire is
7 1 7 5
A)  J0 R2 B)  J0 R2 C)  J0 R2 D)  J0 R2
24 6 12 12

Dr.K.K.R Gowtham 126 Current Electricity


3. A current passing through a long solid cylindrical conductor of radius R. The current in the
conductor is distributed in a non-uniform manner such that current density being given as:
I   r 2 
J  02 1     i for r  R
R   R  
0 for r  R
Here r is the radial distance from the axis, I 0 is a constant which has unit of ampere. It is also
assumed that the conductor is placed with its axis in the x-direction. Total current passing
through the entire cross section of the conductor is
A) I 0 B) I 0 / 2 C) I 0 / 4 D) I 0 / 8

4. In order to increase the resistance of a given wire of uniform cross section to four times its value,
a fraction of its length is stretched uniformly till the full length of the wire becomes times the
original length what is the value of this fraction?
1 1 1 1
A) B) C) C)
4 8 6 6
5. A wire of cross-section area A, length L1, resistivity 1 and temperature coefficient of resistivity
1 is connected to a second wire of length L2, resistivity 2, temperature coefficient of
resistivity 2 and the same area A, so that wire carries same current. Total resistance R is
independent of temperature for small temperature change if (Thermal expansion effect is
negligible)
A)  1 = –  2 B) 1L1  1 + 2L2  2 = 0

C) L1  1 + L2  2 = 0 D) None

6. An infinite ladder is constructed with 1 and 2 resistors as shown in figure. Find the effective
resistance between the points A and B, Find the current that passes through the 2 resistor nearest
to the battery.

A) 2 B) 1  C) 2  D3


7. The equivalent resistance across XY is

A) r B) 2r C) r/2 D) r/4

Dr.K.K.R Gowtham 127 Current Electricity


8. In the circuit each resistance is equal to 12 ohm.
The equivalent resistance between A and B is
A) 6 ohm B) 1 ohm
C) 8 ohm D) 4 ohm

9. If R1 = R3 = 10 ohm R2 = 100 ohm and R4 = R5 = 30 ohm,

the equivalent resistance between A and B is


A) 16.7 ohm B) 50 ohm
C) 30 ohm D)100 ohm

10. In the circuit shown in the figure, the current through


A) the 3 resistor is 0.50 A
B) the 3 resistor is 0.25 A
C) 4 resistor is 0.50 A
D) the 4 resistor is 0.25 A

11. In the circuit shown, P  R, the reading of the galvanometer is same with
switch S open or closed. Then
A) IR = IG B) IP = IG C) IQ = IG D) IQ = IR

12. Equivalent resistance across A and B in the given cicuit is


A) 7r B) 8r/7
C) 4r/7 D) 7r/8

13. Find the equivalent resistance of the circuits shown in figure


between the points a and b. Each resistor has a resistance r.
A) 4r/5 B) 3r/5
C) r/2 D) 5r/4
14. Twelve wires, each having equal resistance r, are joined to
form a cube as shown in figure.
A)the equivalent resistance between the diagonally opposite points
a and f is 5r/6
B) the equivalent resistance between the ends of a face diagonal
such as a and c is 3r/4
C)the equivalent resistance between the ends of an edge such as a
& b is 7r/12 D) all the above

Dr.K.K.R Gowtham 128 Current Electricity


15. In the box shown current i enters at H and leaves at C.
1 2i i i 1
If iAB  , iDC  , iHA  , iGF  , iHE  ,
6 3 2 6 6
choose the branch in which current is zero

A) BG B) FC C) ED D) none
16. What is the equivalent resistance between A and B
A) 3/7R
B) 4/9R
C) 5/6R
D) 6/5 R

17. In fig each resistance (R) in the


circuit is of 100. The ammeter reading
(A1) will be

A) 1A B) 2A
C) 3A D) 4A

18. Find the effective resistance between A and B in the given figure
A) 1.2r B) 1.4r
C) 1.6r D) 1.8r

19. In the circuit as shown in fig. What is the equivalent resistance


between A and B?
A) 7 B) 14
C) 21 D) 28 
20. What are the effective resistances between A and B and D and E respectively?
10r 5r 10r 7r
A) and B) and
7 7 7 5
7r 8r 17 r 5r
C) and D) and
17 7 7 7
21. In the circuit shown, the cell of emf 6V is ideal. The
resistor in which the power dissipated is greatest is
A) 5 B) 3
C) 9 D) 18

Dr.K.K.R Gowtham 129 Current Electricity


22. The resistance of all the wires between any two adjacent dots is R. Then
equivalent resistance between A and B as shown in figure is :
A) 7/3 R B) 7/6 R
C) 14/8 R D) None of these

23. The effective resistance across P and Q in the figure


A) r/3 B) r/2
C) 2r D) 6r

24. A metal rod of radius a is concentric with a metal cylindrical shell of radius b and length l .
The space between rod and cylinder is tightly packed with a high resistance material of
resistivity  . A battery having a terminal voltage V is connected across the combination as
shown. Neglect resistance of rod and cylinder. If I is the total current in the circuit then,
lV
I
A) 
2 lV
I
 1nb  1na 
B)
4 lV lV
I I
 1nb  1na  4 1nb  1na 
C) D)

25. A resistor is formed by two square plates of edge ‘a’, separated by distance ‘d’. The material of
resistivity 1 and  2 are filled in the gap as shown in the figure. The resistance between the
points M and N is

A)
 1  2  d B)
a2 M

a2  1  2  d C D

C)
 1  2  d D)
2  1  2  d
1
    2
d
a2 ln  1  a2 ln  1 
 2   2 
A
B
26. The resistance of a conductor of temperature toC is R = Ro(1 + at + bt2) where Ro is the resistance
of conductor at 0oC. The temperature coefficient of resistance at temperature
N toC is

a  2bt 1  at  bt 2
A) B) a + 2b C) D) constant
1  at  bt 2 a  2bt

27. Figure shows a potentiometer wire AB of length 100 cm connected with an ideal cell of emf 5
volt. The key k is open. If the key k is closed then the balancing length x obtained on the
potentiometer wire when galvanometer shows zero deflection is :

Dr.K.K.R Gowtham 130 Current Electricity


A) 60 cm B) 50 cm
C) 45 cm D) 40 cm

28. A potentiometer is connected to the circuit as shown in figure. In


case of no deflection in the galvanometer the current flowing in
the wire AB is 2 A. Total length of wire AB is 10 m and the null
point is at 6 m from A and the total resistance of the wire is 10Ω.
The value of resistance X is
A) 15  B) 3
C) 4.5 D) 6
29. In the below circuit, AB is a wire of 100 cm with 5 resistance. If
there is no deflection in the galvanometer, the current flowing in the
wire AB is
A) 0.1 A B) 0.5 A
C) 1.0 A D) 1.4 A
30. The potentiometer wire AB is made of two equal parts AN and
NB of cross sectional radii 2r and r respectively. The material of
both parts is same. A cell is connected between A and B. The
potential gradient in the part AN is 1 V/m. Potential difference
between two points C and D (shown in figure) separated by 20 A C N D B
cm is found to be 0.5V. The separation between C and N.
A) 5 cm B) 10 cm C) 12 cm D) 15 cm

31. If a steady current 1 is flowing through a cylindrical


element ABC. Choose the correct relationship A 2r B
A) VAB = 2VBC r C
B) power across BC is 4 times the power across AB l
C) current densities in AB and BC are equal
D) Electric field due to current inside AB and BC are equal l/2
l/2
32. Arrange the order of power dissipated in the given circuits, if the same current is passing through
all circuit and each resistor is .r.

A) B)

C) D)

1) P2 > P3 > P4 > P1 2) P3 > P2 > P4 > P1


3) P4 > P3 > P2 > P1 4) P1 > P2 > P3 > P4

Dr.K.K.R Gowtham 131 Current Electricity


33. Figure shows three resistor configurations R1, R2 and R3 connected to 3V battery. If the power
dissipated by the configuration R1, R2 and R3 is P1, P2 and P3, respectively, then

(1) (2) (3)

A) P1 > P2 > P3 B) P1 > P3 > P2 C) P2 > P1 > P3 D) P3 > P2 > P1


34. What amount of heat will be generated in a coil of resistance R due to a charge q passing
through it if the current in the coil decreases to zero uniformly during a time intervalt

A)
4 q2 R
B) l n
q2 R
C)
2q 2 R
D) l n
 2t 
3 t 2 t 3t q2 R
35. In the given figure for what value of R, the power dissipated in it will be maximum
R0
3 R0
A) B) 3R0
2
R0 R0
R0
C) D) R0
2
R
36. In the circuit shown in the figure the value of R at which the power transferred to this
resistance will be maximum
20V 5
A) 7 B) 7.4 10V
C) 7.2 D) 2.4 4
6

R
10V
37. In the circuit shown in figure for what value of R. The
power consumed by this resistance is maximum
20 10
A) B)
3 3
C) 10 D) 30
38. How a battery grid can be made by using 24 cells of 1 internal resistance which can be deliver
maximum power to a load of 10 resistance.
A) for M=2, N=12, power supply by a grid will be maximum
B) for M=1, N=24, power supply by a grid will be maximum
C) for M=3, N=9, power supply by a grid will be maximum
D) for M=6, N=4, power supply by a grid will be maximum
(where N cells in series, M rows in parallel)

Dr.K.K.R Gowtham 132 Current Electricity


39. V-1 graph for a conductor at temperatures T 1 and T2 is shown. (T2 – T1) is proportional to :
A) cos 2 B) sin 2 T2
V
C) cot 2 D) tan 2 T1



i
40. E denotes electric field in a uniform conductor, I corresponding current through it, v d drift
velocity of electrons and P denotes thermal power produced in the conductor, then which of the
following graph is incorrect
vd P P P
A) B) C) D)

E vd i
E

WHEASTONE BRIDGE:
41. If a current i enters at P and leaves at Q then the p.d. between P
and Q is
A) 2Rri/(R+r)
B) 8R(R+r)i/3R+r
C) (2r+ rR)i
D) 5Ri/(2+2r)
42. In the given circuit, Current in the main circuit is
A) 0.1 Amp B) 0.2 Amp

C) 0.4 Amp D) 0.8 Amp


43. Equivalent resistance between A and B in the given network is
A) 5  B) 6 
C) 10  D) 12 

44. Thirteen resistances each of resistance R ohm are connected in


the circuit as shown in the figure below. The effective resistance between A and
R B is R
4R
A) 2R  B)  R R
3 R R

2R
C)  D) R  A R B
3
R
R R R

45. Effective resistance between A and B in the given circuit is


R R

(last junction ‘J’ before B is J  (J infinity)


A) zero B) 2R C) R 2 D) 3R 2

Dr.K.K.R Gowtham 133 Current Electricity


46. Effective resistance between A and B in the given circuit is
16Rr
A) 7.5 r B)
7R+2r
7Rr
C) 3 R D)
2R+5r

47. In the circuit shown below the resistance of the galvanometer is 20 . In i1


which case of the following alternatives are the currents arranged strictly in 10 ig 100
the decreasing order
i2 G
A) i, i1, i2, ig
B) i, i2, i1, ig 2 20
C) i, i2, ig, i1 i

D) i, i1, ig, i2 2V 0

HEATING EFFECTS:
48. In the circuit shown, the resistances are given in ohms and
the battery is assumed ideal with emf equal to 3.0 volts
The resistor that dissipates the most power is
A) R1 B) R2 C) R3 D) R4

49. What should be the value of resistance R in the circuit shown in


Figure so that the electric bulb consumes the rated power ?
A) 12  B) 4.5 
C) 6 D) 9

50. A 100W bulb B1, and two 60W bulbs B2 and B3, are connected to a 250 V
source, as shown in the figure. Now W 1, W2 and W3 are the output powers of
the bulbs B1, B2 and B3 respectively. Then
A) W1 > W2 = W3 B) W1 > W2 > W3
C) W1 < W2 = W3 D) W1 <W2 < W3

51. Electric bulb 50W – 100V glowing at full power


1
are to be used in parallel with battery 120V, 10. i/
n
Maximum number of bulbs that can be connected i/
2

so that they glow in full power is n


n
i i/n

120 V, 10 
A) 2 B) 8
C) 4 D) 6

Dr.K.K.R Gowtham 134 Current Electricity


52. Assuming all bulbs are identical, rank the brightnesses of the bulbs, from brightest to
dimmest.

A) A > D = H > B = C > E = F = G


B) A > B = C > D = H > E = F = G E
C) A > D = H > E = F = G > B = C
D) all have equal brightness

THE MAXIMUM POWER TRANSFER THEOREM:


53. A dc source with internal resistance R0 is loaded with three
identical resistance R interconnected as shown in figure. At what
value of R will the thermal power generated in this circuit be the R R R
   
highest?
A) 3R0 B) 4R0
C) 5R0 D) 6R0
54. A battery of internal resistance 4 is connected to the network
of resistances as shown. In order to given the maximum power
to the network, the value of R(in ) should be
A) 4/9 B) 8/9
C) 2 D) 18

55. In the figure shown the power generated in y is maximum


when y = 5. Then R is
A) 2  B) 6 
C) 5  D) 3 
GRAPHICAL QUESTIONS:
1. The V-I characteristics of four circuit elements are shown below. Which of these is ohmic :

1) 2) 3) 4)

2. The variation of current with potential difference is as shown in fig. The resistance of the
conductor is
1) 4  2) 2 
3) 3  4) 1 

3. The fig represents the V-I characteristics of a circuit element. The dynamic resistance in the
region AB is
1) 2/3  2) (3/4) 
3) (1/2)  4) 1 

Dr.K.K.R Gowtham 135 Current Electricity


4. Variation of current passing through a conductor as the voltage applied across is ends is varied as
shown in fig. If the resistance is determined at points A, B, C and D. We will find that
1) Resistance at ‘C’ and ‘D’ are equal
2) Resistance at ‘B’ is higher than at A
3) Resistance at ‘C’ is higher than at ‘B’
4) Resistance at ‘A’ is lower than at ‘B’

5. Resistance as shown in fig is negative at


1) A 2) B
3) C 4) None of these

6. The current in a metallic conductor is plotted against voltage at two


different temperatures T 1 and T2. Which is correct
1) T1 > T2 2) T1 < T2 3) T1 = T2 4) none
7. For a metallic wire maintained at two different temperatures,
current (I) – voltage(V) graphs are drawn as shown . Ratio
between the resistances of the wire at 1830C and 00C is
1)1 : 3 2) 2 : 1
3) 3 : 1 4) 5 : 4
8. When a current I is passed through a wire of constant resistance, it produces a potential
difference V across its ends. The graph drawn between log I and log V will be
log I

log I
log I

log I

1) 2) 3) 4)

log V log V log V log V


9. I-V characteristic of a copper wire of length L and area of cross-section A is shown in figure. The
slope of the curve becomes
I
1) More if the experiment is performed at higher temperature
2) More if a wire of steel of same dimension is used
3) More if the length of the wire is increased
4)Less if the length of the wire is increased O
V
10. V-i graphs for parallel and series combination of two identical resistors are as shown in figure.
Which graph represents parallel combination V B

1) A 2) B A
3) A and B both 4) Neither A nor B
i
11. The resistance Rt of a conductor varies with temperature t as shown in the figure. If the
variation is represented by Rt  R0 [1  t  t 2 ] , then Rt

1)  and  are both negative 2)  and  are both positive


3)  is positive and  is negative
4)  is negative and  are positive t

Dr.K.K.R Gowtham 136 Current Electricity


12. A cylindrical conductor has uniform cross-section. Resistivity of its material increase linearly
from left end to right end. If a constant current is flowing through it and at a section distance x
from left end, magnitude of electric field intensity is E, which of the following graphs is correct
E E E

1) 2) 3) 4)

O x O x O x O x

E V(Volts)
2.0
13. For a cell, the graph between the potential difference (V) 1.5
across the terminals of the cell and the current (I) 1.0
drawn from the cell is shown in the figure. The e.m.f. 0.5
and the internal resistance of the cell are 0
1 2 3 4 5 I(amperes)
1) 2V ,0 .5  2) 2V ,0 .4 
4)  2V ,0 .5  3)  2V ,0 .4 
14. The diagram besides shows a circuit used in an
experiment to determine the emf and internal
resistance of the cell C. A graph was plotted of the
potential difference V between the terminals of the
cell against the current I, which was varied by
adjusting the rheostat. The graph is shown on the
right; x and y are the intercepts of the graph with
the axes as shown. What is the internal resistance of the cell ?
1) x 2) y 3) x/y 4) y/x
15. A battery consists of a variable number 'n' of identical cells having internal resistances
connected in series. The terminals of battery are short circuited and the current i is
measured. Which of the graph below shows the relation ship between i and n
i i i i

1) 2) 3) 4)

O O n
O n O n n

16. The two ends of a uniform conductor are joined to a cell of e.m.f. E and some internal resistance.
Starting from the midpoint P of the conductor, we move in the direction of current `and return to
P. The potential V at every point on the path is plotted against the distance covered (x). Which of
the following graphs best represents the resulting curve
V V V V
1) 2) 3) 4)
E <E E
<E

X X X X

17. Current(I)- voltage(V) characteristics of two elements A and B


are as shown [figure (a)and (b)] .Which of the following
graphs represents current voltage characteristics for their
series combination ?

Dr.K.K.R Gowtham 137 Current Electricity


1) 2) 3) 4)
18. A cell of emf E has an internal resistance r & is connected to rheostat. When resistance R of
rheostat is changed, correct graph of potential difference across it is
V V

1) R 2) R 3) 4)

19. A battery is connected to a uniform resistance wire AB and B is earthed. Which one of the graphs
below shows how the current density J varies along AB – +

A B
J J
J J
1) Zero at all 2) 3) 4)
points
0 0
A B A B 0 0
A B A B
20. The variation of current (I) and voltage (V) is as shown in figure A. The
variation of power P with current I is best shown by which of the
following graph

1) 2) 3) 4)

II. REASONING BASED QUESTIONS:


This section contains Reasoning type questions, each having four choice (A) ,(B),(C) and (D) out of
which ONLY ONE IS correct. Each question contains STATEMENT 1 and STATEMENT 2 You have
to mark your answer as
(A) If Statement I is true, Statement II is true; Statement II is the correct explanation of Statement I
(B)If Statement I is true, Statement II is true; Statement II is not a correct explanation of Statement I
(C) If Statement I is true , Statement II is false (D) If Statement I is false , Statement II is true
1. Statement – I : There is no current in the metals in the absence of electric field.
Statement – II : Motion of free electron are randomly.
2. Statement – I : The drift velocity of electrons in a metallic wire will decrease, if the
temperature of the wire is increased.
Statement – II : On increasing temperature, conductivity of metallic wire decreases.
3. Statement – I : The electric bulbs glows immediately when switch is on.

Dr.K.K.R Gowtham 138 Current Electricity


Statement –II : The drift velocity of electrons in a metallic wire is very high.
4. Statement – I : Electric field outside the conducting wire which carries a constant current
is zero.
Statement – II : Net charge on conducting wire is zero.
5. Statement – I : The connecting wires are made of copper.
Statement – II : The electrical conductivity of copper is high.
6. Statement – I : The resistance of super-conductor is zero.
Statement – II : The super-conductors are used for the transmission of electric power.

7. Statement – I : A series combination of cells is used when their internal resistance is much
smaller than the external resistance
nE
Statement – II It follows from the relation I  , where the symbols have their
R  nr
standard meaning, in series connection
8. Statement – I : In meter bridge experiment, a high resistance is always connected in series with
a galvanometer.
Statement – II : As resistance increases current through the circuit increases.
9. Statement – I : A potentiometer of longer length is used for accurate measurement.
Statement – II : The potential gradient for a potentiometer of longer length with a given
source of e.m.f. becomes small.
10. Statement – I : The e.m.f. of the driver cell in the potentiometer experiment should be greater
than the e.m.f. of the cell to be determined.
Statement – II : The fall of potential across the potentiometer wire should not be less than the
e.m.f. of the cell to be determined.

III. MULTIPLE CORRECT CHOICE TYPE QUESTIONS:


1. The resistivity of a cylindrical conductor carrying steady current along its length varies linearly
 x
with the distance from the current carrying end as given by    0  1   where l is the
 l
length of the conductor and x is the distance from the current entry end.  o is a positive
constant.
A) Electric field varies linearly with x
B) Electric potential difference across the length x varies linearly with x
C) Volume charge density in the conductor is zero
D) volume charge density in the conductor is non zero
2. Two cells are connected in series. They have different internal resistance r 1& r2(r1greater than r2 )
with equal EMF. The resistance of connecting wire is R
A)The potential of positive terminal of first cell will not always be higher than potential of
negative terminal

Dr.K.K.R Gowtham 139 Current Electricity


B) The potential of positive terminal of first cell will always be higher than potential of negative
terminal
C)The potential difference across second cell (r 2) can be 0 for a particular value of R
D)The negative terminal of second cell will be at higher potential than its positive terminal for
R=0
3. Consider the following statements which of the following are true
A)By using Ohm’s law we can solve the complex electric circuits having more than one source
of emf
B)Direction of e.m.f depends on the direction of current flows through a cell
C)Jockey can be dragged along the potentiometer wire while measuring potential difference
D)Principle of meter bridge is balanced condition of Wheatstone bridge
4. Sensitivity of potentiometer can be increased by
A) increasing series resistance in the primary circuit
B) decreasing the length of potentiometer wire
C) using thin and high resistivity wire as potentiometer wire
D) increasing the length of the wire
5. In a potentiometer arrangement. E1 is the cell establishing current in primary circuit. E2 is the cell
to be measured. AB is the potentiometer wire and G is a galvanometer. Which of the following are
the essential condition for balance to be obtained.
A) The emf of E1 must be greater than the emf of E 2.
B) Either the positive terminals of both E1 and E2 or the negative terminals of both E1 and E2 must
be joined to one end of potentiometer wire.
C) The positive terminals of E1 and E2 must be joined to one end of potentiometer wire.
D) The resistance of G must be less than the resistance of AB..
6. In the shown circuit, 45J/S power develops across 5 resistance, then
A) Power develops across 6 resistance is 6W.
B) Power develops across 9 resistance is 81W
C) Potential drop across 9 resistance is 9V.
D) Current flowing through 12 resistance is 4A
IV. LINKED COMPREHENSION TYPE QUESTIONS
PASSAGE 1: A battery charger has been
shown in figure.

1. For charging current of 2.5A


A) R = 1.2  B) 1.110 
C) 1.32 D) 1.4

Dr.K.K.R Gowtham 140 Current Electricity


2. For 25W of power to be delivered to 11V voltage source
A) R = 1.23  B) 1.32  C) 1.52  D) 1.44

3. For 25W of power to be delivered to the combination of and 11V source


A) 1.24  B) 1.32 C) 1.46 D) None

PASSAGE 2: Figure shown a balanced meter bridge


X
1. If AC = l mm, =
Y
l l
A) B)
100 - l 1000 - l
l l
C) D)
1- l 1+ l
2. If ‘X’ is heated, ‘C’ shifts
A) towards right B) towards left C) neither right nor left D) it depends on the value of ‘Y’
3. If jockey is slided towards right current through the (G) will be
A) away from P B) towards P C) always zero D) any of the above
PASSAGE- 3: A 6V battery of negligible internal resistance is connected
across a uniform wire AB of length 100cm. The positive terminal of
another battery of emf 4V and internal resistance 1 is joined to the
point A as shown in figure. Take the potential at B to be zero.

1. What are the potentials at points A and C, C is not Touching the wire AB
A) 6V, 2V B)8V, 4V C) 6V, 4V D)8V, 3V
2. At which point D of the wire AB, the potential is equal to the potential at C,C is not Touching the
wire AB
A) A AD=60cm B) B. AD=50cm C)AD=40cm D)None of these
3 If the points C and D are connected by a wire, what will be the current through it?
A) zero B)1A C) 2A D) 3A
Passage 4:
Power supplied by an ideal battery (internal resistance = 0) is E i where E = emf of battery and i is the
current drawn from it. In series, current through resistance are same where as in parallel difference across
V2
resistance are same. : Power consumed by resistance is P = i 2 R = = Vi
R
In the circuit shown, total power supplied by an ideal battery is 80 W, then D A A
1. emf of battery is
A) 20V B) 30V
C) 60V D) 40V

Dr.K.K.R Gowtham 141 Current Electricity


2. Ratio of power developed across 10Ω, 20Ω and 60Ω will be :
A) 2 : 4 : 3 B) 1 : 2 : 6 C) 2 : 4 : 6 D) 1 : 2 : 3
3. If 10 resistance is removed from the circuit, then :
A) Power consumed by 20 will increase B) Power consumed by 60 will decrease
C) Both (A) and (B) are correct D) Both (A) and (B) are wrong
PASSAGE 5:
In the circuit shown in figure points a,b and c are maintained at constant potentials. When a
resistance is connected between a and b no current flows through it. When the same resistance is
connected between b and c current flows from c and b
1 When only three resistances shown in figure are in the circuit ABB
A) Current in resistance 2R is from c to d B) Current in resistance 2R is from d to c
C) Current in resistance 3R is from b to d
D) Current in resistance R is from a to d

2. The ratio of power dissipated across 2R to 3R resistance is


A) 2/3 B) 32/3 C) 16/3 D) 1
3. If Va  Vb  10V and Vc  30V , then value of Vd will be

165 170 154 185


A) V B) V C) V D) V
9 11 8 13
V. MATRIX MATCHING TYPE QUESTIONS
1. Column I gives physical quantities of a situation in which a i
current i passes through two rods I and II of equal length that are
joined in series. The ratio of free electron density (n), resistivity I II

ρ  and cross-section area (A) of both are in ratio A


B C

n1 : n 2  2 :1, ρ1 : ρ2  2 :1 and A1 : A2  1: 2 respectively. Column II gives corresponding


results. Match the ratios in Column I with the values in Column II.
Column – I Column – II
Drift velocity of free electron in rod I
a) p) 0.5
Drif velocity of free electron in rod II
Electric field in rod I
b) q) 1
Electric field in rod II
Potential difference across rod I
c) r) 2
Potential difference across rod II
Average time taken by free electron to move from A to B
d) s) 4
Average time taken by free electron to move from B to C
t) 5

Dr.K.K.R Gowtham 142 Current Electricity


2. Column - i represents a combination of resistors of equal identical resistors of resistance r while
Column - II represents the equivalent resistance across (a) and (b). Match identities in Column - I
to their corresponding match(es) in COLUMN - II.
List – I List – II

1) p) r/4

b) q) r

c) r) r/2

d) s)5/8r
3. A network consisting of three resistors, three batteries, and an identical volt meter is shown in fig
I1 I 2 I2
A B C

3
5 V
4
10V

8V I2
F E D
12V

Column I ColumnII

A. Current in branch EB is P. 0A
B. Current in branch CB is Q. 0.5A
C. Current in branch ED is R. 1.5A

D. Current in branch EC is S. 2A

Dr.K.K.R Gowtham 143 Current Electricity


4. For the circuit given, match the power dissipated in
resistor given in Column I with that in Column II. Match
the Following:

(a) Power in (p) 1/5


(b) Power in (q) 9/10
(c) Power in (r) 4/5

(d) Power in (s) 0

(t) None of these

5. Column I shows the arrangements of bulbs A,B and C having rated powers
PA , PB and PC respectively  PA  PB  PC  , when each bulb is operated at same rated voltage
V. Column II lists information about intensities of bulbs. Match the statements of two columns
Column I Column II
(a) B C (p) A is glowing with minimum
brightness

(b) A (q) A is glowing with maximum


C
brightness
B

(c) A B C (r) B is glowing with minimum


brightness

(d) A (s) C is glowing with minimum


brightness
B

Dr.K.K.R Gowtham 144 Current Electricity


VI. INTEGER ANSWER TYPE QUESTIONS
1. Find equivalent resistance between points A and B in the circuit shown. All resistances are equal
9
to R. (Given R =  ).
5

2. A 20 volt battery with an internal resistance of 5W is connected to a resistor of xW. If an


additional 6W resistor is connected across the battery, the power supplied by the battery in the
30
external resistance remains the same. If x = , find n.
n
3. In the shown circuit, potential difference between points A 4 9V 1 3V 4
and B  VA  VB  is 16 V. The current passing through A B
2  resistor will be 3.5n A then n is
2
4 Figure shows part of a circuit. The potential difference
VC  VB =10n volt. Find n.

5. For the circuit shown in figure. Find the potential difference between points a and bis
1.2nV then n is 2

6. In the circuit given below, all switches are open. What is the voltage across points A and B(in V).

7. If the applied emf in the primary circuit of a potentiometer is increased by 3 times, the value of
potential gradient will become 3n times, then n is

Dr.K.K.R Gowtham 145 Current Electricity


8. When 10 identical cells of no internal resistance are connected in series in the secondary circuit
of a potentiometer, the balancing length is L. If three of them are wrongly connected, the
balancing length becomes 2L/n Then n is
9. A potentiometer is designed to measure emf of two cells. When first cell of emf e1 = 10V is
connected, the balancing length obtained is l1  15cm . When the second cell of emf
 2  20 V is connected, the balancing length obtained is l2 is 10x cm then x is
10. The resistor in which the maximum heat is produced is given by in ohm

11. Resistors P, Q and R in the circuit have equal resistances. If the battery is supplying a total
power of 12W, what is the power dissipated as heat in resistor R in watts
Q
P

12. Three 60W, 120V light bulbs are connected across a 120 V
power source, as shown in figure. The total power delivered to
the three bulbs (in watt) is 5n. Find n. Assume that the
resistance of each bulb is constant.

LEVEL – C
I. SINGLE CORRECT CHOICE TYPE QUESTIONS:
1) D 2) D 3) B 4) B 5) B 6) A 7) C 8) D 9) A 10) D
11) A 12) B 13) A 14) D 15) 16) C 17) B 18) B 19) D 20) D
21) A 22) 23) A 24) B 25) C 26) A 27) D 28) D 29) A 30) B
31) B 32) A 33) C 34) A 35) A 36) B 37) A 38) A 39) C 40) C
41) A 42) A 43) B 44) C 45) B 46) C 47) B 48) A 49) C 50) D
51) C 52) A 53) A 54) C 55) D

Dr.K.K.R Gowtham 146 Current Electricity


GRAPHICAL QUESTIONS:
1) 1 2) 2 3) 4 4) 1 5) 1 6) 2 7) 3 8) 1 9) 4 10) 1
11) 2 12) 2 13) 2 14) 4 15) 4 16) 2 17) 3 18) 4 19) 4 20) 2
II.REASONING TYPE QUESTIONS

1) A 2) B 3) C 4) A 5) A 6) B 7) A 8) C 9) A 10) A

III. ONE OR MORE THAN ONE ANSWER TYPE QUESTIONS


1) A,D 2) A,C,D 3) B,D 4) A,D 5) A,B
6) A,C,D
IV. LINKED COMPREHENSION TYPE QUESTIONS
PASSAGE – I 1 –B, 2- A, 3- A PASSAGE –II 1 –A, 2- A, 3- A
PASSAGE – III 1 –A, 2- D, 3- A PASSAGE – IV 1 –D, 2- A, 3- A
PASSAGE – V 1 –A, 2- B, 3- B
V. MATRIX MATCHING TYPE QUESTIONS
1) a – q ; b – s ; c – s ; d – q 2) a – s ; b – r ; c – p ; d – q
3) a – r ; b – q ; c – q ; d – p 4) a – s ; b – r ; c – q ; d – p
5) a – q, r ; b – r ; c – p ; d – q,s
VI. INTEGER ANSWER TYPE QUESTIONS
1) 1 2) 4 3) 1 4) 5) 2 6) 1 7) 1 8) 5 9) 3 10) 4
11) 2 12) 8
***

Dr.K.K.R Gowtham 147 Current Electricity

You might also like